Вы находитесь на странице: 1из 85

1

Pulmonary Medicine and Critical Care:Question 1


A 57-year-old man with advanced chronic obstructive pulmonary disease (COPD) and
systemic hypertension is evaluated because of a 6-day history of productive cough
and shortness of breath. He uses inhaled albuterol and ipratropium bromide, a long-
acting theophylline preparation, and lisinopril. He uses supplemental oxygen at night
and during ambulation. Ciprofloxacin is prescribed for an exacerbation of CO PD.
Three days later, having had nausea for a day, the man is brought to the emergency
department after he is found nearly unconscious. Arterial oxygen saturation is 89%,
with the patient breathing room air. Electrocardiogram shows normal sinus rhythm
with nonspecific ST-T changes in the lateral chest leads.
Which of the following is likely to have interacted with ciprofloxacin and caused the
symptoms that brought the man to the emergency department?
A. Albuterol
B. Theophylline
C. Ipratropium bromide
D. Lisinopril
E. Oxygen

Pulmonary Medicine and Critical Care:Question 2


A 17-year-old previously healthy man presents with a history of shortness of breath
on exertion, particularly during basketball season, when he sometimes needs to sit
down during practice to catch his breath. He does not notice any shortness of breath
with routine activity. There is no family history of asthma.
On physical examination, he is in no respiratory distress. His lungs are clear, with no
wheezing during either tidal breathing or forced expiration. His heart is normal.
Baseline spirometry is normal.
What is the next diagnostic step?
A. Allergy testing
B. Methacholine challenge testing
C. Exercise testing with postexercise spirometry
D. Overnight oximetry
E. Repeat lung volumes and diffusing capacity

Pulmonary Medicine and Critical Care:Question 3


A 62-year-old woman is evaluated because of abnormal results on chest radiograph.
She smokes 1 pack of cigarettes per day, with a 50-pack-year history. She has a
morning cough productive of small amounts of yellow sputum. She has hypertension,
for which she takes metoprolol, and type 2 diabetes mellitus, which is managed with
diet and metformin therapy.
Her body mass index is 30. Her blood pressure is 145/90 mm Hg. Chest radiograph
shows a 1.5-cm nodule in the left upper lobe; no previous radiographs are available
for comparison.
Which of the following should be measured before sending this patient for a positron
emission tomography scan to evaluate the nodule?
A. Serum sodium
B. Blood pressure
C. Serum creatinine
D. Serum glucose
2

Pulmonary Medicine and Critical Care:Question 4


A 67-year-old man presents with a 6-month history of progressive exertional
dyspnea and nonproductive cough. His weight has been stable. He was treated with
several courses of oral antibiotics for “bronchitis” but his symptoms have not abated.
He has a 40-pack-year cumulative smoking history; he stopped smoking 6 years
ago.
On physical examination, there are no signs of heart failure, but bibasilar coarse
inspiratory crackles are audible, and he has early clubbing of the fingers. Chest
radiograph shows increased reticular markings in the periphery of both lower lobes
that were not evident on radiograph done 3 years ago.
Which of the following additional aspects of the patient’s history are most important
in evaluating him?
A. Travel history
B. Occupational history
C. Allergy history
D. Family history

Pulmonary Medicine and Critical Care:Question 5


A 28-year-old woman is evaluated in February because of a 6-week history of severe
cough. She describes coughing paroxysms often followed by emesis. She has no
postnasal drip.
On physical examination, the patient is afebrile. The chest is clear to auscultation
and chest radiograph is normal. Laboratory studies show a leukocyte count of
12,000/μL, with 30% neutrophils, 65% lymphocytes, 2% monocytes, and 3%
eosinophils.
Which of the following is the most likely diagnosis?
A. Gastroesophageal reflux disease
B. Cough-variant asthma
C. Bordetella pertussis
D. Sinusitis
E. Sarcoidosis

Pulmonary Medicine and Critical Care:Question 6


A 25-year-old man is evaluated because of a 3-month history of episodic dyspnea at
rest. The episodes occur approximately three times per week and are accompanied
by cough. His symptoms awaken him at night approximately three times per month.
He had asthma as a child, which resolved.
His temperature is 36.5 C (97.7 SF), pulse rate 85/min and regular, respiration rate
14/min, and blood pressure 125/75 mm Hg. The only abnormality noted on physical
examination is bilateral wheezing without crackles. Chest radiograph is normal.
Spirometry shows a forced expiratory volume in 1 sec (FEV1) 78% of predicted, and
a forced vital capacity 93% of predicted. He has a 17% (430 mL) improvement in
FEV1 after using bronchodilators.
Which of the following is the most appropriate treatment regimen for this patient?
A. Albuterol
B. Albuterol and a long-acting β-agonist
C. A long-acting β-agonist
D. Albuterol and a low-dose inhaled corticosteroid
3

Pulmonary Medicine and Critical Care:Question 7


A 67-year-old woman is evaluated in the emergency department because of a 2-day
history of fever and a cough productive of purulent sputum. She is intubated for
hypoxic respiratory failure due to pneumonia and hospitalized in the intensive care
unit. Her medical history is remarkable only for hypertension for which she takes a
calcium channel blocker. She has never worked outside the home, has no pets, and
has not traveled recently.
Which of the following would be appropriate initial antibiotic therapy?
A. A second-generation cephalosporin and macrolide
B. A second-generation cephalosporin
C. A third-generation cephalosporin, aminoglycoside, and macrolide
D. An antipseudomonal β-lactam/β-lactamase combination and fluoroquinolone

Pulmonary Medicine and Critical Care:Question 8


A 38-year-old woman who has never smoked is evaluated because of well-
circumscribed nodule in the right lower lobe, which was discovered on a chest
radiograph during a routine physical examination. There is no family history of
cancer, the patient has never had cancer herself, and the lesion is completely
calcified. A CT scan of the chest done 2 years ago showed a 0.8-cm nodule in the
right lower lobe. Another CT scan is ordered, and it shows the nodule is eccentrically
calcified and has grown.
What is the best next step in this patient’s management?
A. Lobectomy
B. Transthoracic needle aspiration
C. Positron emission tomography scan
D. Remove the nodule

Pulmonary Medicine and Critical Care:Question 9


A 67-year-old man with a history of chronic obstructive pulmonary disease is
evaluated because of a 3-month history of progressive dyspnea and wheeze. One
year ago, the patient had five-vessel coronary artery bypass graft surgery with a
prolonged postoperative stay in the intensive care unit because of acute respiratory
distress syndrome. On physical examination the patient has a persistent wheeze. His
jugular venous pressure is normal. The flow-volume loop is shown.
4

Results of pulmonary function tests are as follows:


Forced expiratory volume in 1 sec (FEy1) 2.22 L (64% of predicted)
Forced vital capacity (FVC) 4.96 L (107% of predicted)
FEV1/FVC 45%
Forced expiratory flow (FEF25%-75%) 2.13 L/sec (60% of predicted)
Which of the following is the most likely cause of this patient’s worsening dyspnea?
A. Exacerbation of chronic obstructive pulmonary disease
B. Congestive heart failure
C. Late seq uela of acute respiratory distress syndrome
D. Tracheal stenosis
E. Constrictive pericarditis

Pulmonary Medicine and Critical Care:Question 10


A 59-year-old woman is evaluated because of shortness of breath of acute onset.
She has developed worsening insomnia and fatigue since she was diagnosed with
adenocarcinoma of the breast 2 months ago. There is no known metastatic disease
and therapy is to begin next week. She has no cough or chest pain.
Her temperature is 37.2 C (98.9 SF), pulse rate is 105/min, respiration rate is
22/min, and blood pressure is 158/88 mm Hg. Arterial oxygen saturation measured
by pulse oximetry with the patient breathing room air is 95%. Her lungs are clear to
auscultation. There is no jugular venous distention. Heart sounds are normal without
murmurs, rubs, or gallops. The abdomen is not tender, and there is no edema of the
extremities. Chest radiograph shows no abnormalities, and electrocardiogram shows
only sinus tachycardia. Ventilation/perfusion scan shows two large segmental
perfusion defects; the ventilation scan shows only mildly heterogeneous washout.
Which one of the following is the best next step in the management of this patient?
A. CT scan using an intravenous radiographic contrast agent
B. Intravenous thrombolytic therapy
C. Intravenous heparin therapy
D. Doppler ultrasound imaging of the lower extremities

Pulmonary Medicine and Critical Care:Question 11


A 70-year-old man is referred because of 2-year history of progressive exertional
dyspnea and an abnormal lung examination. He has no significant cough,
5

hemoptysis, or chest pain, and he has never smoked. He worked as an insulation


installer for 30 years and retired 10 years ago.
On physical examination, fine end-inspiratory crackles are auscultated at both lung
bases. He has clubbing of the fingers. Chest radiograph and pulmonary function tests
are ordered.
In addition to bilateral lower lobe reticular interstitial opacities, what radiographic
feature is likely to be found in this patient?
A. Eggshell calcification of lymph nodes
B. Upper lobe pulmonary nodules
C. Calcified pleural plaques
D. Cavitating pulmonary nodules and masses
E. Bilateral hilar adenopathy

Pulmonary Medicine and Critical Care:Question 12


A 46-year-old woman who works as a nurse is evaluated because of a 2-year history
of episodic wheezing and a squeaky voice.” This past spring, her symptoms
worsened, requiring her to seek medical attention; she was placed on a short-acting
β-agonist that did not provide much relief. She has no history of wheezing and says
that these changes began after a severe influenza infection 3 years ago. Currently
she feels well and has had no symptoms for several months; she is not taking any
medications. Physical examination shows no abnormalities, and baseline spirometry
is normal. What is the best test to evaluate this patients condition?
A. Methacholine challenge testing
B. Bronchoscopy to evaluate her trachea
C. Exercise echocardiogram
D. CT scan of the sinuses

Pulmonary Medicine and Critical Care:Question 13


A 43-year-old man is evaluated because he falls asleep while working at his
computer during the day. He typically goes to bed at 11 PM and sleeps until 6AM. He
is uncertain whether he snores, and he has no history of choking arousals. Even
though he sleeps 7 hours per night during the week, he does not feel that his sleep is
restful; he says that he tosses and turns throughout the night. Nearly every morning
he has a headache. He drinks coffee at work to avoid falling asleep. He has not had
any recent automobile accidents. He has no other medical problems, except that he
admits to recent problems with sexual dysfunction. He takes no medicines and drinks
alcohol only occasionally. On physical examination, he is 175 cm (69 in) tall and
weighs 81 .6 kg (180 Ib). His neck is 44.5 cm (17.5 in), and he has a crowed
oropharynx. His blood pressure is 125/70 mm Hg. His palate is low-lying, and his
uvula is elongated. Chest and heart examination reveals no abnormalities. He has no
peripheral edema or clubbing. Which of the following risk factors, considered alone,
is most predictive of obstructive sleep apnea in this patient?
A. Drinking alcohol in the evening
B. Excessive daytime sleepiness
C. Nocturnal sexual dysfunction
D. Neck circumference
6

E. Morning headaches

Pulmonary Medicine and Critical Care:Question 14


A 25-year-old man with cystic fibrosis returns to the emergency department because
of a third episode of abdominal pain with nausea and vomiting within the past 2
years. He takes trimethoprim-sulfamethoxazole for suppression of chronic
respiratory infection and pancrelipase for treatment of pancreatic insufficiency.
On physical examination, he is afebrile, and his blood pressure is normal. Coarse
crackles and short wheezes are heard in both lungs. His abdomen is distended, firm,
and diffusely tender. There is no jugular venous distention or peripheral edema. His
total leukocyte count is 83004±. Liver function studies and serum amylase and lipase
levels are normal. An abdominal radiograph suggests partial small bowel obstruction.

Which of the following is the most likely cause of this patient’s abdominal pain?
A. Exacerbation of chronic intestinal pseudo-obstruction
B. Irritable bowel syndrome
C. Diverticulitis
D. Distal intestinal obstruction syndrome
E. Exacerbation of chronic pancreatitis

Pulmonary Medicine and Critical Care:Question 15


A 43-year-old man is evaluated because he is unable to complete 18 holes of golf
without gasping for air. He has smoked 1 pack of cigarettes per day for 20 years.
On physical examination, he has fine inspiratory crackles in both lower lobes. He has
no wheezing or clubbing but does have trace edema of the lower extremities. Cardiac
examination reveals i and 2 are normal without murmurs. A chest radiograph is
normal. Lung function studies show forced expiratory volume in 1 sec (FEV1) 80% of
predicted, forced vital capacity (FVC) 75% of predicted, and FEV1/FVC ratio of 90%.
Lung volumes are reduced and diffusing lung capacity for carbon monoxide (DLCO) is
55% of predicted. Echocardiography shows normal left ventricular systolic and
diastolic function without valvular abnormalities.
What is the best next step in this patient’s evaluation?
A. Left heart catheterization
B. Cardiac stress test
C. HRCT scan of the chest
D. Bronchoscopy
E. Pulmonary angiography

Pulmonary Medicine and Critical Care:Question 16


A 72-year-old woman with a history of rheumatoid arthritis is evaluated because of
persistent cough, purulent sputum production, and occasional scant hemoptysis. She
reports no fever, but says she has difficulty gaining weight. She has never smoked
cigarettes. She is hospitalized because of tachypnea and hypoxia. Spirometry shows
moderately severe obstructive impairment. Chest radiograph shows tramlines
without infiltrate. Gram stain of the sputum shows numerous leukocytes but no
bacteria; culture results are pending. Previous IgE levels were normal. There is no
eosinophilia. An immediate skin test forAspergillus is negative. A tuberculin skin test
performed last year was nonreactive.
Which of the following is appropriate therapy for this patient?
A. Intravenous fluoroquinolone and aminoglycoside
B. A β-lactam/β-lactamase combination
C. Itraconazole
7

D. A new-generation macrolide

Pulmonary Medicine and Critical Care:Question 17


A 57-year-old man is evaluated because of a 1-year history of dyspnea on exertion.
Examination reveals swan-neck deformities of the second, third, and fourth digits,
and a boutonniere deformity of the fifth digit of both hands. He has 2+ pitting edema
and chronic venous stasis changes of the lower extremities.
The patient weighs 191 kg (421 Ib). Laboratory evaluation reveals a serum
rheumatoid factor of 87 U/mL, and arterial blood gas values, with the patient
breathing room air, are 296 mm Hg, PC0235 mm Hg, and pH 7.44.
Results of pulmonary function tests are as follows.
Forced expiratory volume in 1 sec (FEV1) 2.06 L (84% of predicted)
Forced vital capacity (FVC)2.74 L (83% of predicted)
FEV1/FVC 75%
Forced expiratory flow (FEF25%-75%) 1 .58 L/sec (57% of predicted)
Total lung capacity 4.01 L (76% of predicted)
Functional residual capacity2.13 L (72% of predicted)
Expiratory reserve volume0.16 L (10% of predicted)
Diffusing lung capacity for carbon monoxide (DLCO) 23.65 mL/min/mm Hg (121% of
predicted)

Which of the following is the most likely cause of the patients dyspnea?
A. Bronchiolitis obliterans
B. Interstitial lung disease
C. Congestive heart failure
D. Obesity

Pulmonary Medicine and Critical Care:Question 18


A 59-year-old man with advanced chronic obstructive pulmonary disease is
evaluated because of a daily cough productive of white or yellow sputum, dyspnea
after climbing one flight of stairs, and a recent 4.5-kg (10-Ib) weight loss with no
associated change in appetite or food intake. The patient stopped smoking 4 years
ago. On physical examination, he has diminished breath sounds throughout all lung
fields. Arterial oxygen saturation measured by pulse oximetry with the patient at
rest, breathing room air, is 87%. Chest radiograph suggests hyperinflation of the
lungs but shows no pulmonary infiltrates or abnormalities of the cardiac silhouette.
Pulmonary function studies show a forced expiratory volume in 1 sec (FEy1) 39% of
predicted and forced vital capacity (FVC) 78% of predicted.
Which of the following may prolong life in this patient?
A. Albuterol
B. Ipratropium bromide
C. Theophylline
D. Supplemental oxygen

Pulmonary Medicine and Critical Care:Question 19


A 52-year-old woman is evaluated because of a 1-year history of progressive
dyspnea on exertion. She is short of breath when climbing one flight of stairs and
when dressing. She is a former heavy cigarette smoker and has a long history of
hypertension. On physical examination, she has elevated jugular venous distention,
a pronounced cardiac pulmonic valve component, and mild pitting edema of both
ankles. Results of complete blood count, serum electrolytes, renal function, and liver
function tests are normal. Tests for antinuclear antibodies, rheumatoid factor, HIV,
and anti-Scl-70 antibodies are negative.
8

Chest radiograph shows prominent central pulmonary arteries; the lung fields are
clear.
Echocardiogram shows concentric left ventricular hypertrophy with normal left
ventricular systolic function (ejection fraction 55%) and a mildly dilated right
ventricle with normal function. The aortic, mitral, and tricuspid valves are normal in
structure. The estimated pulmonary artery systolic pressure measured at rest is 59
mm Hg (assuming an estimated right atrial pressure of 10 mm Hg).
Spirometry and lung volumes are normal. Diffusing lung capacity for carbon
monoxide (DLCO) is 40% of predicted. Ventilation/perfusion scan shows normal
ventilation and mild heterogeneity of perfusion but no segmental or larger defects.
Right heart catheterization shows right atrial pressure 10 mm Hg, right ventricular
pressure 50/10 mm Hg, pulmonary artery pressure 50/20 mm Hg, pulmonary
capillary wedge pressure 26, cardiac output 3.1 L/min, cardiac index 2.0 L/min/m2.
Which one of the following is the most likely cause of this patients pulmonary
hypertension?
A. Left ventricular diastolic dysfunction
B. Chronic pulmonary embolism
C. Primary pulmonary hypertension
D. Pulmonary veno-occlusive disease
E. Constrictive pericarditis

Pulmonary Medicine and Critical Care:Question 20


A 45-year-old man is evaluated because of a 6-month history of progressive left
shoulder pain, which has not responded to nonsteroidal medications. He also has
anorexia of recent onset, with a 2.3-kg (5-Ib) weight loss. He currently smokes 2
packs of cigarettes per day and has a 60-pack-year smoking history.
On physical examination, he is in mild distress. He is afebrile; pulse rate is 80/min
and regular, respiration rate is 14/min, and blood pressure is 120/65 mm Hg.
Musculoskeletal examination shows no abnormalities, and he has normal strength
and range of motion in his left arm and shoulder. Radiograph of his left shoulder
shows no abnormalities, but radiography of his chest shows a mass in the left apex.
CT scan of the chest confirms a posterior left upper lobe mass, which appears to be
abutting the vertebral bodies. All mediastinal nodes are <1 cm in greatest
dimension.
Serum sodium, calcium, and alkaline phosphatase levels are normal, as are liver
function studies. Pulmonary function tests show a forced expiratory volume in 1 sec
(FEV1) of 2.5 L and forced vital capacity (FVC) of 4.0 L. Bronchoscopy shows no
endobronchial lesions, but results of transbronchial biopsy of a specimen from the
left upper lobe show squamous cell carcinoma.
What is the best next step in this patient’s management?
A. MRI of the chest
B. HRCT scan of the left upper lobe
C. Resection of the mass
D. CT scan of the brain
E. Radionuclide bone scan

Pulmonary Medicine and Critical Care:Question 21


A 34-year-old woman is evaluated because of a 1-year history of increased dyspnea
on exertion. She has no symptoms at rest but has to stop about 15 minutes into her
aerobics class because of dyspnea and occasional cough. She usually recovers fully
in about an hour. One year ago she was able to do aerobics for 45 minutes without
difficulty.
Her vital signs are normal, and her physical examination is normal, including clear
9

breath sounds. Baseline spirometry is also normal.


Which of the following would be best next step in the management of this patient?
A. Oral leukotriene inhibitors
B. Long-acting theophylline
C. Inhaled ipratropium bromide prior to exercise
D. Inhaled albuterol prior to exercise
E. Inhaled corticosteroids

Pulmonary Medicine and Critical Care:Question 22


A 47-year-old black man is evaluated because of a 2-month history of cough. Three
months ago hypertension was diagnosed, for which he takes hydrochiorothiazide and
benazepril. He attributes his cough to the change in the weather. He has a history of
gastroesophageal reflux disease that is well controlled with a proton-pump inhibitor.
He has no history of asthma.
Which of the following would be the most appropriate next step?
A. CT scan of the sinuses
B. pH probe
C. Methacholine challenge testing
D. Discontinuation of his angiotensin-converting enzyme inhibitor
E. Allergy testing

Pulmonary Medicine and Critical Care:Question 23


A 66-year-old woman is brought to the emergency department after being rescued
from a burning building. Oxygen was administered at the scene, and the oxygen
saturation level was 100% by pulse oximetry on 100% oxygen delivered by face
mask during transport to the emergency department.
At the hospital, oxygen flow is decreased to maintain oxygen saturation at >92%.
The patient is awake but appears somewhat confused. She complains of feeling
dyspneic. On physical examination, she has first- and second-degree burns over
50% of her body. Bilateral crackles are audible in the lower lung fields and coarse
breath sounds bilaterally. Measurement of arterial blood gases shows a partial
oxygen pressure of 45 mm Hg, with oxygen saturation of 94% by pulse oximetry.
Which of the following explains the discrepancy between the arterial blood gas
findings and the oximetry?
A. The carboxyhemoglobin level is elevated.
B. The carbon dioxide level is elevated.
C. The methemoglobin level is elevated.
D. The total hemoglobin is elevated.
E. The patient is anemic.

Pulmonary Medicine and Critical Care:Question 24


A 62-year-old man with chronic obstructive pulmonary disease is interested in lung-
volume-reduction surgery. After a course of pulmonary rehabilitation, his forced
expiratory volume in 1 sec (FEV1) is 17% of predicted and his diffusing lung capacity
for carbon monoxoide (DLCO) is 19% of predicted. A CT scan of the chest shows
heterogeneously distributed emphysema.
Which of the following statements about lung-volume-reduction surgery is true of
this patient?
A. He may be an excellent candidate because of his low FEV1 and DLCO.
B. He is not a candidate because of his heterogeneously distributed emphysema and
his low FEV1 and DLCO.
C. He is not a candidate because of the low FEV1.
10

D. He is not a candidate because of the low DLCO.

Pulmonary Medicine and Critical Care:Question 25


A 45-year-old man is evaluated because he had a tuberculin skin test performed as
part of an employment physical and it was read as positive, with induration of 22
mm. He is asymptomatic.
The patient emigrated from Sri Lanka 15 years ago. He has no recollection of
exposure to tuberculosis, but he remembers being told that he was given a
‘tuberculosis vaccine as a child. A chest radiograph is interpreted as normal.
Which of the following is the most appropriate next step in this patient’s
management?
A. Treatment for active tuberculosis should be initiated.
B. Treatment for latent tuberculosis should be initiated.
C. Further testing is warranted to look for active tuberculosis, and sputum induction
or bronchoscopy should be performed.
D. Skin testing should not have been performed; his reaction is a false-positive
secondary to his earlier vaccination.

Pulmonary Medicine and Critical Care:Question 26


A 53-year-old woman with a history of mild persistent asthma is evaluated because
of a recent increase in her symptoms, with dyspnea and cough occurring daily and a
cough that awakens her once a week. She is currently using low-dose inhaled
corticosteroids. She has no symptoms of rhinitis or gastroesophageal reflux. On
physical examination, she has intermittent wheezing bilaterally.
Which of the following is the most appropriate change in her therapy?
A. Initiate azithromycin therapy.
B. Add a nebulized short-acting β-agonist.
C. Add inhaled ipratropium bromide.
D. Add a long-acting β-agonist.
E. Add a leukotriene inhibitor.

Pulmonary Medicine and Critical Care:Question 27


A 57-year-old man is evaluated because of a 3-month history of chronic cough associated
with occasional streaky hemoptysis. He has no other systemic symptoms and has not
traveled recently. He stopped smoking cigarettes 2 years ago, after a 45-pack-year
smoking history. He has had two episodes of pneumonia in the past year, both in the
right upper lobe, which resolved with antibiotics.
On physical examination, he has decreased breath sounds over the right chest
posteriorly. Chest radiograph shows a 3-cm proximal mass in the right upper lobe.
Bronchoscopy shows an endobronchial mass in the right upper lobe, 3 cm from the
carina. Biopsy of the mass shows it to be squamous cell carcinoma. A CT scan of the
chest with contrast shows a speculated mass in the proximal right upper lobe with distal
atelectasis. Two mediastinal nodes are found, each 1.5 cm in diameter, one in the right
hilum and one in the subcarinal area. Pulmonary function is normal, with a forced
expiratory volume in 1 sec (FEV1) of 3.1 L (95% of predicted).
Which of the following is the best next step in the management of this patient?
A. Resection of the mass
B. Mediastinal node sampling
C. Chemotherapy
11

D. Radiation therapy
E. Palliative care

Pulmonary Medicine and Critical Care:Question 28


A 70-year-old man is evaluated because of shortness of breath. He has noted
progression of his symptoms, primarily with exertion over the past 6 months. He is
unable to walk one flight of stairs or two blocks on level ground without becoming
short of breath. He has no chest pain, paroxysmal nocturnal dyspnea, orthopnea, or
lower extremity edema. He has a 40-pack-year history of cigarette smoking, but
stopped smoking 10 years ago. He worked in a naval shipyard 50 years ago but has
spent most of his working life as a schoolteacher. On physical examination, his
respiration rate is 18/min but he does not appear short of breath. Examination of the
chest shows an increased anterior-posterior diameter. On auscultation he has
diffusely decreased breath sounds and a prolonged expiratory phase with no
wheezing. He has no cyanosis or clubbing. Spirometry shows forced expiratory
volume in 1 sec (FEV1) 55% of predicted and forced vital capacity (FVC) 80% of
predicted with a ratio of FEV1 to FVC 60%. Which of the following is the best test to
evaluate this patients condition?
A. Lung volumes and diffusing capacity
B. Echocardiography
C. Exercise spirometry
D. Methacholine challenging testing

Pulmonary Medicine and Critical Care:Question 29


A 57-year-old woman is evaluated because of a 6-month history of progressive
dyspnea. She has never smoked cigarettes. She has been treated on several
occasions for a “COPD exacerbation” with intravenous corticosteroids with some
relief in symptoms.
She states that symptoms are particularly bothersome in the evening. Pulmonary
function tests are as follows:
Forced expiratory volume in 1 sec (FEV1) 2.06 L (84% of predicted)
Forced vital capacity (FVC)2.74 L (83% of predicted)
FEV1/FVC 75%
Maximum voluntary ventilation63 L/min (65% of predicted)
Maximum inspiratory pressure39 cm H20 (52% of predicted)
Maximum expiratory pressure58 cm H20 (42% of predicted)
Total lung capacity 4.01 L (76% of predicted)
Functional residual capacity 2.13 L (72% of predicted)
Residual volume 1.42 L (73% of predicted)
Diffusing lung capacity for carbon monoxide (DLco) 19.39 mL/min per mm Hg (99%
of predicted)
Which of the following is the most likely cause of this patients dyspnea?
A. Chronic thromboembolic disease
B. Chronic obstructive pulmonary disease
C. Idiopathic pulmonary fibrosis
D. Myasthenia gravis

Pulmonary Medicine and Critical Care:Question 30


A man on a mountain climbing expedition is evaluated because of shortness of
breath and a 2-day history of coughing. He is unable to keep pace with the other
climbers. His cognition appears normal, and he is not drowsy. Present altitude is
5000 m (16,000 feet), and the group has ascended 2500 m (8000 feet) in the past 2
days.
12

Which of the following statements is accurate regarding this patient’s condition?


A. Carbonic anhydrase inhibitors are the treatment of choice.
B. Rate of ascent contributes minimally to this disorder.
C. Dexamethasone is the treatment of choice.
D. Oxygen and descent to lower altitude are the treatment of choice.
E. The patient is not at increased risk for recurrence of this disorder on future climbs.

Pulmonary Medicine and Critical Care:Question 31


A 67-year-old man has a 12-year history of seropositive rheumatoid arthritis
currently well controlled with methotrexate, 12.5 mg orally once weekly, and
prednisone, 5 mgld orally. He smokes 2 packs of cigarettes per day. He has noted
some subtle decrease in exercise tolerance and an occasional dry cough.
On physical examination he has stable deformities of the hands and wrists consistent
with rheumatoid arthritis and several old subcutaneous nodules on the extensor
surfaces of his forearms. He has no cervical adenopathy, no abnormal jugular venous
distention, and no peripheral edema. Coarse crackles are audible at both lung bases,
and there is dullness to percussion with decreased breath sounds on the left. Chest
radiograph shows minimal bilateral reticular infiltrates and a moderate left pleural
effusion. Thoracentesis yields straw-colored fluid in which protein is measured at 4.5
g/dL (serum 5.6 gIdL) and glucose at 30 mg/dL (serum = 106 mgldL).
Pulmonary function studies demonstrate:
Forced expiratory volume in 1 sec (FEy1) 78% of predicted
Forced vital capacity (FVC)75% of predicted
FEV1/FVC 86%,
Total lung capacity 70% of predicted
Residual volume 72% of predicted
Diffusing lung capacity for carbon monoxide (DLco) 86% of predicted
Which of the following is most likely true regarding this patient’s lung disease?
A. The crackles strongly suggest a diagnosis of congestive heart failure.
B. This patient has an empyema in need of immediate surgical drainage.
C. The patient has recurrent aspiration pneumonia.
D. The patient’s signs and symptoms are consistent with rheumatoid arthritis-
associated interstitial lung
disease.

Pulmonary Medicine and Critical Care:Question 32


A 52-year-old man is evaluated in the emergency department because of mild
shortness of breath and chest pain, which began after he carried heavy suitcases
from his car following a 9-hour drive. He has a history of chronic obstructive
pulmonary disease, peptic ulcer disease, and Hodgkin’s lymphoma in remission 4
years after radiation therapy. One year ago, after an automobile accident, he had
deep vein thrombosis of the right leg for which he received warfarin therapy for 3
months.
Lungs are clear to auscultation. Heart sounds are regular and are not accompanied
by gallops or murmurs. There is mild pitting edema at both ankles, which the patient
says has been present since the automobile accident. Chest radiograph and
electrocardiogram show no evidence of acute disease. A ventilation/perfusion nuclear
scintigraphy scan shows several subsegmental matched defects and is interpreted as
indicative of a low probability of pulmonary embolus.
Which of the following is the best next step in the management of this patient?
A. Restart heparin.
B. Measure arterial blood gases.
13

C. Measure the D-dimer level in the serum and re-initiate anticoagulation therapy if it
is elevated.
D. Measure the D-dimer level in the serum and perform Doppler ultrasound imaging
of the lower
extremities if it is elevated.

Pulmonary Medicine and Critical Care:Question 33


A 70-year-old woman resident of a nursing home is evaluated in the emergency
department because of decreasing mental status and hypothermia. She has a history
of stroke and is currently taking only aspirin. She has been able to eat on her own
and there have been no witnessed aspirations. She has not been treated recently
with antibiotics.
Her leukocyte count is 12,000/μL, and her hemoglobin is 12 gIdL. Serum electrolytes
are within normal limits and she has mild chronic renal insufficiency. Chest
radiograph shows a small interstitial infiltrate in the right lower lung field.
She receives traditional empiric treatment for community-acquired pneumonia.
Therapy for which of the following should also be considered?
A. Pseudomonas aeruginosa
B. Anaerobic bacteria
C. Enteric gram-negative organisms
D. Aspergillus fumigatus
E. Mycobacterium tuberculosis

Pulmonary Medicine and Critical Care:Question 34


A 28-year-old man with cystic fibrosis who was hospitalized 3 days ago to receive
antibiotic therapy is evaluated because he has developed hemoptysis. He is a thin
man in mild respiratory distress. Twenty minutes ago, he coughed up approximately
75 mL of clotted blood, but has not had any further bleeding.
His vital signs are stable, and pulse oximetry is 92% with the patient breathing 2 L
oxygen by nasal cannula. Bilateral crackles are audible on chest auscultation. He has
a scaphoid abdomen. The remainder of his examination is normal.
What is the best next step in this patient’s management?
A. Administer &epsilon;-aminocaproic acid as a plasminogen-to-plasmin inhibitor.
B. Arrange pulmonary consultation for urgent bronchoscopy.
C. Change his antibiotics to cover resistantPseudomonas.
D. Arrange an interventional radiology consultation for embolization.
E. Arrange an anesthesia consultation for selective intubation.

Pulmonary Medicine and Critical Care:Question 35


A 34-year-old woman, a nurse, is evaluated after having made a number of trips to
the emergency department because of breathing difficulties that were treated as
exacerbations of asthma. The patient was treated in the emergency department with
nebulized β-agonists and a corticosteroid. The patient has been given prescriptions
for inhaled corticosteroids, but she has never had a documented objective change in
lung function after using them.
Spirometry in an outpatient setting between exacerbations has been normal, and
measurement of arterial blood gases done during one of her visits to the emergency
department was normal. She has never smoked cigarettes. She has no symptoms of
rhinitis, no nocturnal symptoms, no recent viral infections, and no symptoms of
gastroesophageal reflux disease.
On physical examination, she is anxious but in no respiratory distress. She is
afebrile. Her pulse rate is 85/min and regular; respiration rate is 16/min, and blood
14

pressure is 130/75 mm Hg. Her breathing is unlabored, and she has inspiratory and
expiratory wheezes but is otherwise normal.
What is the most likely explanation for this patients lack of response to corticosteroid
therapy?
A. The patient has chronic obstructive pulmonary disease.
B. The patient has vocal cord dysfunction.
C. The patient has corticosteroid-resistant asthma.
D. The patient has asthma with allergic bronchopulmonary aspergillosis.

Pulmonary Medicine and Critical Care:Question 36


An obese 62-year-old man is evaluated because of heartburn and frequent throat
clearing. Gastroesophageal reflux disease (GERD) is diagnosed and therapy with a
proton-pump inhibitor, twice daily, is initiated. He is advised of lifestyle modifications
that help prevent GERD.
Six weeks later, he is re-evaluated because the cough, although somewhat better,
persists. He has no postnasal drip. His heartburn has resolved.
Which of the following is the best next step in this patients management?
A. Change his medication to intranasal corticosteroids and antihistamines.
B. Order 24-hour esophageal pH monitoring.
C. Refer him for fundoplication.
D. Continue the proton-pump inhibitor therapy and re-evaluate him in 6 weeks.

Pulmonary Medicine and Critical Care:Question 37


A 66-year-old woman with chronic obstructive pulmonary disease is evaluated
because of chronic cough and dyspnea. She currently uses a long-acting
bronchodilator twice per day, an inhaled corticosteroid twice per day, ipratropium
four times per day, and albuterol four to six times per day. She smokes 1 pack of
cigarettes per day.
On physical examination, her vitals signs are normal. Her oxygen saturation at rest
and with exertion is 94%. She has diminished breath sounds, a prolonged
expiratory-to-inspiratory phase, and no wheezes. Her heart rate and rhythm are
regular, with a normal i, a physiologically split S2, and no murmurs or rubs. Chest
radiograph reveals hyperinflation, increased retrosternal airspace, and flattened
hemidiaphragms bilaterally.
Which of the following should be initiated at this time to address this patient’s cough
and dyspnea?
A. Increase her use of the long-acting bronchodilator.
B. Prescribe supplemental oxygen.
C. Provide emergency treatment for a tension pneumothorax.
D. Discuss techniques to help her to quit smoking.
E. Increase her dosage of inhaled corticosteroid.

Pulmonary Medicine and Critical Care:Question 38


A 60-year-old man with a history of alcohol abuse is evaluated because of the
insidious onset of dyspnea over the course of 6 weeks. He has no cough, chest or
abdominal pain, or hemoptysis. He smoked 1.5 packs of cigarettes per day for 25
years and stopped smoking 4 years ago.
He has a moderate-sized right pleural effusion. Chest radiograph shows the effusion
with minimal contralateral mediastinal shift and is otherwise normal. Pleural fluid
analysis shows clear yellow fluid with 500 nucleated cells/μL, 10% neutrophils, 25%
15

lymphocytes, 60% macrophages, and 5% mesothelial cells.


Pleural fluid values are as follows:
Total protein 1.1 g/dL
Serum lactate dehydrogenase4l U/L
Serum amylase 20 U/L
Glucose 100 mg/dL
pH 7.45
Ratio of pleural fluid to serum (PF/S) total protein 0.2
Ratio of pleural fluid to serum lactate dehydrogenase (upper limits of normal) 0.35
Other laboratory results show a serum albumin of 2.4 g/dL, an INR of 1 .5, and a
normal urinalysis. Electrocardiogram is normal.
Which of the following is the most likely diagnosis?
A. Congestive heart failure
B. Hepatic hydrothorax
C. Chronic pancreatitis
D. Lung cancer
E. Nephrotic syndrome

Pulmonary Medicine and Critical Care:Question 39


A 45-year-old man alternates day, evening, and night shifts at weekly intervals in a
factory. He drinks six to eight cups of coffee per day to fight off episodes of
sleepiness. His wife reports that he snores and moves his legs often while sleeping.
The accompanying image represents one segment of his overnight polysomnogram.

Which of the following disorders does this polysomnogram show?


A. Obstructive sleep apnea
B. Restless legs syndrome
C. Narcolepsy
16

D. Central sleep apnea


E. Cheyne-Stokes breathing

Pulmonary Medicine and Critical Care:Question 40


A 67-year-old man is evaluated because of a 3-week history of cough productive of
blood-streaked sputum. A chest radiograph shows an infiltrate in the right upper
lobe. He is treated with antibiotics for 2 weeks, but the blood-streaked sputum
persists. A CT scan of the chest shows a mass obstructing the right upper lobe and
evidence of postobstructive pneumonitis.
Examination of the mediastinum shows enlarged lymph nodes in the right
paratracheal space. A positron emission tomography (PET) scan shows uptake in the
mass itself and in the lymph nodes in the right paratracheal space. Fiberoptic
bronchoscopy is performed; an endobronchial lesion is identified and a sample is
taken for biopsy. The biopsy shows squamous cell carcinoma.
Which of the following is the best next step in this patients management?
A. Refer him for radiation therapy.
B. Perform immediate mediastinoscopy.
C. Refer him for surgery.
D. Perform a repeat positron emission tomography scan.

Pulmonary Medicine and Critical Care:Question 41


A 39-year-old woman is evaluated because of an abnormality on a chest radiograph
obtained as a pre-employment screen. She is a lifetime nonsmoker. She has worked
for several years as an assistant in a research laboratory specializing in molecular
genetic screening assays. The patient’s chest radiograph is shown.
17

Which of the following findings would warrant a trial of oral corticosteroid therapy?
A. Bilateral anterior uveitis
B. Hypercalcemia
C. Fever and tender red nodules over the anterior shins
D. Persistent cough

Pulmonary Medicine and Critical Care:Question 42


A 72-year-old woman is evaluated because of morning headaches and swelling in the
lower extremities that worsens as the day progresses. She is able to sleep supine,
using one pillow at night, and notices shortness of breath walking distances greater
than 20 feet.
On physical examination, significant findings include diminished breath sounds,
distant heart sounds, with pulmonic valve component equal in intensity to aortic
valve component, and paradoxical splitting of the S2. Laboratory finding include a
PaO2of 59mm Hg, PaCO2of44 mm Hg, and pH of 7.41. Electrocardiogram shows
right ventricular hypertrophy with cor pulmonale and right axis deviation.
Which of the following is the best rationale for long-term oxygen therapy for this
patient?
A. The PaO2 is less than or equal to 65 mm Hg (Medicare guideline)
B. Her morning headaches
C. Evidence of cor pulmonale and a PaO2 between 55 and 60 mm Hg
D. Dyspnea

Pulmonary Medicine and Critical Care:Question 43


A 57-year-old man with severe persistent asthma is evaluated on routine follow-up.
He states that his asthma has been under good control for the last 3 months on
high-dose inhaled corticosteroids and a long-acting β-agonist. He uses a short-acting
β-agonist only three times per week as a rescue medication, and he has nocturnal
symptoms very rarely. Peak expiratory flows have been stable. His physical
examination is normal, including clear breath sounds, and spirometry is normal. It is
decided that he will keep using the short-acting β-agonist as a rescue medication.
Which of the following is the best next step in this patient’s management?
A. Stop the long-acting β-agonist and decrease the dose of inhaled corticosteroid.
B. Continue current therapy and have the patient return in 6 months.
C. Stop the long-acting β-agonist and maintain the dose of inhaled corticosteroid.
D. Continue the long-acting β-agonist and decrease the dose of inhaled
corticosteroid.

Pulmonary Medicine and Critical Care:Question 44


A 71-year-old woman is evaluated because of progressive fatigue, weakness, and
dyspnea on exertion.
A former smoker, she has a history of advanced emphysema and is on continuous
long-term oxygen therapy. She currently uses a long-acting bronchodilator twice per
day, theophylline and an inhaled corticosteroid twice per day, ipratropium four times
per day, and albuterol four to six times per day.
Physical examination is normal. Heart and lung examinations are consistent with
long-standing, advanced emphysema. Her laboratory and radiographic findings are
unremarkable.
Which of the following is the best next step in this patient’s management?
A. Refer her to a multidisciplinary rehabilitation program.
18

B. Prescribe an antidepressant medication.


C. Prescribe an empiric course of “pulse dose” corticosteroids at 500 mg/d for 3
consecutive days.
D. Explain to her that she is on maximal medical therapy and that nothing more can
be done for her emphysema.

Pulmonary Medicine and Critical Care:Question 45


A 40-year-old worker is rescued from a fire at a fabrics warehouse. He is comatose
and hypotensive. Paramedics administer 100% oxygen by face mask, initiate
intravenous fluids, and transfer him to the emergency department.
Upon arrival in the emergency department, he is still comatose. His pulse rate is
70/min, and respiration rate is 26/min. Mean arterial pressure is 50 mm Hg, and the
monitor shows abundant ventricular ectopy. No burns are noted, and the patient is
not cyanotic. There are no obvious signs of trauma. Pupils are reactive. Scattered
rhonchi are audible. Neurologic examination shows lack of motor response to
application of a painful stimulus.
Laboratory studies:
Serum sodium 143 meq/L
Serum potassium 4 meq/L
Serum chloride 100 meq/L
Serum bicarbonate 10 meq/L
Arterial blood gases
PO2 300 mm Hg
PCO2 25 mm Hg
pH 7.20

Carboxyhemoglobin 15% (normal = 1% to 3%)


Endotracheal intubation is performed, and additional fluid is administered. Treatment
with pressor agents and hyperbaric oxygen is initiated.
What is the best next step in this patient’s management?
A. Initiate intravenous bicarbonate therapy.
B. Administer intravenous sodium thiosulfate.
C. Order a CT scan of the brain and a cervical spine series.
D. Administer intravenous methylene blue.
E. Measure serum osmolarity, and calculate the osmolal gap.

Pulmonary Medicine and Critical Care:Question 46


A 47-year-old man is evaluated because of cough that has persisted for 6 months.
He has no postnasal drip, wheeze, or heartburn. His physical examination, chest
radiograph, and spirometry are normal. He receives no benefit from a 3-month trial
of twice-daily proton-pump inhibitors, intranasal corticosteroids, and antihistamines.
He has a family history of allergies.
Which of the following would suggest the diagnosis of this patients chronic cough?
A. 24-hour esophageal pH monitoring
B. CT scan of the sinuses
C. Bronchoscopy
D. Methacholine challenge testing
E. CT scan of the chest

Pulmonary Medicine and Critical Care:Question 47


A 67-year-old man with chronic obstructive pulmonary disease is evaluated because
of chronic
19

dyspnea, minimally productive cough, and limited exercise tolerance. He thinks his
dyspnea on exertion has worsened. He stopped smoking cigarettes 8 years ago and
is currently using an ipratropium inhaler four times per day and salmeterol discus
twice per day. His body mass index, which 6 months ago was 21, is now 19.
On physical examination, he is afebrile, his pulse rate is 94/min and regular, and
respiration rate is 20/min. His breathing is unlabored at rest. He has signs of chest
hyperinflation and decreased breath sounds without wheezing. He has no peripheral
edema. The remainder of his examination is normal; results of a fecal occult blood
test are negative.
Baseline spirometry is unchanged.
Forced expiratory volume in 1 sec (FEV1) 35% of predicted
Forced vital capacity (FVC) 85% of predicted
FEV1/FVC ratio50%
PO2 62 mm Hg
PCO2 45 mm Hg
pH 7.38 (with the patient breathing room air)
Chest radiograph reveals only hyperinflation.
What is the best way to manage this patient’s weight loss?
A. Provide dietary instructions to increase his caloric intake.
B. Treat him with anabolic steroids.
C. Refer him for pulmonary rehabilitation with exercise and nutritional counseling.
D. Prescribe oxygen supplementation to improve his oxygen consumption.
E. Add inhaled corticosteroids to his medical regimen.

Pulmonary Medicine and Critical Care:Question 48


A 50-year-old man is evaluated in the emergency department because of fever, a
nonproductive cough, and a 2-day history of myalgia and headache. He has also had
some nausea and diarrhea. He is a heavy smoker.
On physical examination, he is slightly disoriented. Temperature is 38.9 0C (102 0F),
pulse rate is 110/min, respiration rate is 32/min. Chest radiograph shows fluffy
infiltrates to the right upper and lower lobes.
Results of laboratory testing show serum sodium of 128 meq/L, blood urea nitrogen
of 42 mg/dL, serum creatinine of 2.2 mg/dL, and serum creatine kinase of 250 U/L.
Which one of the following is best next step in the management of this patients
pneumonia?
A. Order direct fluorescent antibody testing of the sputum for Legionella
B. Order serologic testing for Legionella
C. Send a urine specimen for measurement of Legionella antigen
D. Initiate empiric antibiotic therapy forLegionella

Pulmonary Medicine and Critical Care:Question 49


A 28-year-old man, a suspected cocaine abuser, is found in a parking lot throwing
himself against cars. En route to the hospital, he has a generalized tonic-clonic
seizure that lasts approximately 3 minutes.
His temperature is 38.8 C (101.8 F), pulse rate is 120/min, respiration rate is
18/min, and blood pressure is 170/98 mm Hg. He is agitated, diaphoretic, and
voicing paranoid thoughts. Restraints are applied and intravenous access is obtained.

Which one of the following should be initially administered to this patient?


A. Labetalol
20

B. Haloperidol
C. Lorazepam
D. Phenytoin
E. Acetaminophen

Pulmonary Medicine and Critical Care:Question 50


A 72-year-old man with chronic obstructive pulmonary disease is evaluated because
he is planning a vacation in Hawaii and wants to ensure he can safely make the trip.
He has a history of dyspnea when climbing more that one flight of stairs.
On physical examination, his pulse rate is 76/min, respiration rate is 16/min, and
blood pressure is 122/84mm Hg. He has mildly diminished breath sounds bilaterally,
but otherwise the examination is unremarkable. There is no clubbing and no
evidence of peripheral edema. Results of arterial blood gas measurements and
spirometry (measured at ambient temperature with the patient breathing room air at
a barometric pressure of 760 mm Hg) are as follows.
PaO2 62 mm Hg
PaCO2 40 mm Hg
pH 7.40
What is the most appropriate recommendation for this patient?
A. He should not travel by a commercial airline.
B. He should be pretreated with carbonic anhydrase inhibitors before the flight.
C. He should be pretreated with dexamethasone before the flight.
D. He should use supplemental oxygen during the flight.
E. The cabin pressure on commercial airlines is sufficient for this patients needs.

Pulmonary Medicine and Critical Care:Question 51


A 37-year-old man with asthma is evaluated because he continues to have frequent
attacks and now feels his short-acting β-agonist is not providing relief. He states he
is using his medications, including a long-acting β-agonist inhaler, inhaled high-dose
corticosteroids, and a short-acting β-agonist inhaler as rescue medication. He has
symptoms daily and nocturnal symptoms about twice per week.
On physical examination, he is in mild respiratory distress. He is afebrile. Pulse rate
is 90/min and regular, respiration rate is 18/min, and blood pressure is 140/85 mm
Hg. He has bilateral wheezing.
Spirometry shows a forced expiratory volume in 1 sec (FEV1) 65% of predicted; it
improves with bronchodilators to 85% of predicted. He has no history of recent viral
upper respiratory infections or rhinitis or symptoms of gastroesophageal reflux
disease.
Which of the following is the best next step in this patients management?
A. Add a leukotriene inhibitor.
B. Switch to an oral β-agonist and have the patient return for a pill count.
C. Observe the patient using the metered-dose inhaler.
D. Initiate oral prednisone therapy and have the patient return for a pill count.
E. Have the patient return with a symptom and treatment log.

Pulmonary Medicine and Critical Care:Question 52


A 57-year-old woman with systemic lupus erythematosus is evaluated because the
gradual onset of shortness of breath on exertion. She takes low-dose corticosteroids
and nonsteroidal anti-inflammatory agents. Her current disease manifestations are
primarily small-joint arthralgias and anemia. She has had no fever, chest pains, or
21

cough.
On physical examination, there is no peripheral edema or abnormal jugular venous
distention. There are no crackles or rub on lung auscultation. Chest radiograph is
normal except for low lung volumes and bibasilar platelike atelectasis. Pulmonary
function testing shows:
Forced expiratory volume in 1 sec (FEV1) 75% of predicted
Forced vital capacity (FVC)73% of predicted
FEV1/FVC 90%
Total lung capacity 72%
Residual volume 95%
Diffusing lung capacity for carbon monoxide (DLco) 80%
CT scan of the chest reveals symmetrical lower lobe volume loss without pulmonary
infiltrates or fibrosis.
Which of the following best characterizes this patients restrictive lung disease?
A. Diffuse alveolar hemorrhage
B. Small airways disease secondary to systemic lupus erythematosus
C. Interstitial pneumonitis
D. Respiratory muscle weakness

Pulmonary Medicine and Critical Care:Question 53


A 32-year-old black man is evaluated because of a 4-month history of progressive
dyspnea on exertion.
On physical examination, the patient has elevated jugular venous pressure to the
angle of the jaw with large V waves, a holosystolic murmur at the left lower sternal
border, and an accentuated pulmonic valve component. The lungs are clear to
auscultation. The abdomen is distended, with shifting dullness. The patient has 1+
peripheral edema. High-resolution CT scan reveals mosaic perfusion.
Pulmonary function test results are listed below.
Forced expiratory volume in 1 sec (FEV1) 2.06 L (84% of predicted)
Forced vital capacity (FVC) 2.74 L (83% of predicted)
FEV1/FVC 75%
Forced expiratory flow (FEF 25%-75%) 1.58 L/sec (57% of predicted)
Total lung capacity 4.01 L (76% of predicted)
Functional residual capacity 2.13 L (72% of predicted)
Residual volume 1.42 L (73% of predicted)
Diffusing lung capacity for carbon monoxide (DLco) 4.30 mL/min per mm Hg (22%
of predicted)
Which of the following is the most likely cause of this patients dyspnea?
A. Idiopathic pulmonary fibrosis
B. Emphysema
C. Chronic thromboembolic disease
D. Congestive heart failure
E. Cirrhosis
22

Pulmonary Medicine and Critical Care:Question 54


A 61-year-old man is evaluated in the emergency department because of a 3-day
history of cough productive of yellow sputum. He has a history of coronary artery
disease, severe diabetes, and moderate emphysema. He routinely uses supplemental
oxygen, 2 L/min. He states that he is more dyspneic than usual and is now short of
breath even at rest. He reports no hemoptysis or chest pain.
His oxygen saturation is 91% on 2 L supplemental oxygen; he is using accessory
muscles of respiration and pursed-lipped breathing. Physical examination shows a
prolonged expiratory-to-inspiratory phase on exhalation and a few scattered
wheezes. He has tachycardia and bilateral pitting edema of the extremities.
Chest radiograph shows changes consistent with emphysema but is otherwise
unchanged from baseline. Complete blood count shows leukocytosis with a left shift.
Two days ago, his arterial blood gas values were PaO2 70 mm Hg, PaCO248 mm
Hg, and pH 7.37. Today, the values are PaO2 59 mm Hg, PaCO2 64 mm Hg, and pH
7.30.
Which of the following is the best next step in this patients management?
A. Prescribe oral prednisone, and instruct him to follow up with his primary physician
in 1 to 2 weeks.
B. Prescribe oral prednisone and an oral antibiotic, and instruct him to follow up with
his primary physician in 1 to 2 weeks.
C. Hospitalize him.
D. Increase his supplemental oxygen to 3 L/min, prescribe oral prednisone and an
oral antibiotic, and instruct him to follow up with his primary physician in 1 to 2
weeks.
E. Reassure him that exacerbations are common in COPD, increase his supplemental
oxygen to 3 L/min, and instruct him to follow up with his primary physician within 1
week.

Pulmonary Medicine and Critical Care:Question 55


A 65-year-old man is evaluated because of a 2-year history of shortness of breath on
exertion, fatigue of fairly recent onset, and weight loss. Symptoms have progressed
to the point that he is dyspneic when engaged in the activities of daily living. He has
had a dry cough for more than 1 year, but over the past 3 months, he has had a
cough productive of yellow sputum with occasional streaks of blood. He has no fever
or night sweats. He has a 20-pack-year history of cigarette smoking; he quit
smoking more than 20 years ago. He is a retired surface miner. On physical
examination, he is thin and appears chronically ill. He has fine inspiratory crackles
and clubbing. Chest radiograph shows diffuse, small nodular densities with a 5-cm
mass in the left upper lobe and a 3-cm mass with cavitation in the right upper lobe.
Report of a chest radiograph taken 2 years ago describes bilateral nodular infiltrates
but does not mention upper lobe masses.
Which of the following would be the most appropriate next step in this patient’s
management?
A. Begin oral clindamycin.
B. Schedule a positron emission tomography scan.
C. Measure the serum rheumatoid factor level.
D. Collect sputum for an acid-fast bacillus stain and culture.
E. Schedule fiberoptic bronchoscopy.

Pulmonary Medicine and Critical Care:Question 56


A 22-year-old previously healthy man is evaluated in the emergency department
because of fever, productive cough, and shortness of breath. His temperature is 40 C
(104 F), pulse is 120/min, respiration rate is 32/min, and blood pressure is 100/70
23

mm Hg. Measurement of arterial blood gases with the patient breathing room air
shows PO2 of 55 mm Hg, PCO2 of 30 mm Hg, pH of 7.41. Chest radiograph reveals
bilateral alveolar infiltrates with no effusions. Gram stain of the sputum reveals
gram-positive diplococci.
Which of the following is the most appropriate for this patient?
A. Treat him as an outpatient with oral therapy.
B. Treat him as an outpatient with intravenous therapy.
C. Hospitalize him.
D. Hospitalize the patient in the intensive care unit.

Pulmonary Medicine and Critical Care:Question 57


A 52-year-old man is evaluated because of a 2-month history of nonproductive
cough, myalgias, and low-grade fever. When his illness began, a chest radiograph
showed bilateral alveolar infiltrates, and a presumptive diagnosis of community-
acquired pneumonia was made. He was treated with oral azithromycin without effect,
followed by a 10-day course of levofloxacin, also without effect. During the course of
his illness he has lost 4.5 kg (10 Ib) without significant anorexia. He is a lifetime
nonsmoker and works as an office manager. He has no pets and no unusual hobbies.
On physical examination, his vitals signs are normal, except for a respiration rate of
20/min. He is in mild respiratory distress on exertion. On examination of the chest,
bilateral crackles are audible, without wheezing. Chest radiograph shows bilateral
alveolar infiltrates, which are changed in location from those seen on his original
radiographs. Pulmonary function tests show forced expiratory volume in 1 sec (FEy1)
75% of predicted, forced vital capacity (FVC) 72% of predicted, total lung capacity
80% of predicted, and diffusing lung capacity for carbon monoxide (DLCO) 65% of
predicted. Arterial blood gas values, with the patient breathing room air, are PO2 62
mm Hg, PCO 242 mm Hg, and pH 7.39.

Which of the following is the most likely diagnosis?


A. Hypersensitivity pneumonitis
B. Cryptogenic organizing pneumonitis
C. Resistant pneumococcal pneumonia
D. Chlamydia pneumonia
E. Bronchoalveolar cell carcinoma

Pulmonary Medicine and Critical Care:Question 58


A 54-year-old man with known severe emphysema is evaluated because of a 6-week
history of poor appetite, cough with intermittent production of blood-tinged sputum,
and increased dyspnea on exertion. The patient has recently completed a 2-week
course of oral antibiotics, but his symptoms have not abated. Recent pulmonary
function testing shows severe obstructive lung disease with a forced expiratory
volume in 1 sec (FEV1) of less than 20% of predicted on repeat studies. Chest
radiograph shows a 2-cm left peripheral lung mass that was not present on a
radiograph 18 months ago. CT scan of the chest confirms the mass and shows no
evidence of mediastinal lymphadenopathy. Needle aspiration of the mass yields cells
that show adenocarcinoma of the lung. Which one of the following statements is true
in the consideration of this patient for lung transplantation?
A. This patient should be referred immediately for lung transplantation.
B. Lung transplantation may be considered if the tumor is removed by surgical
wedge resection and no recurrence is observed after 2 years.
C. The presence of cancer is an absolute contraindication to lung transplantation.
D. This patient is too old to be considered for lung transplantation.
24

Pulmonary Medicine and Critical Care:Question 59


A 68-year-old woman with hypertension, degenerative joint disease, and depression
is hospitalized in the intensive care unit with altered mental status. No information
about the patients medications is available.
The patients temperature is 38.3 C (101 F), pulse rate is 118/min, respiration rate is
24/min, and blood pressure is 120/70 mm Hg. No focal neurologic abnormalities are
noted, but she is obtunded. Her reflexes are normal. Electrocardiogram shows only
sinus tachycardia with no acute changes, and chest radiograph shows bilateral
interstitial infiltrates.
Laboratory studies:
Blood urea nitrogen 30 mg/dL
Plasma glucose 65 mg/dL
Serum sodium 148 meq/L
Serum potassium 4.5 meq/L
Serum chloride 108 meq/L
Serum bicarbonate 18 meq/L
Serum creatinine 1.6 mg/dL
Serum osmolarity 302 mosm/L
Which one of the following is the most likely toxin?
A. Ephedrine
B. Salicylate
C. Ethylene glycol
D. Lithium
E. Acetaminophen

Pulmonary Medicine and Critical Care:Question 60


A 68-year-old man with severe chronic obstructive pulmonary disease (forced
expiratory volume in 1 sec 32% of predicted) is evaluated because of severe
dyspnea and the inability to carry out his activities of daily living. He is on maximal
bronchodilator and oxygen therapy.
Which of the following might pulmonary rehabilitation improve?
A. Exercise tolerance
B. Forced expiratory volume in 1 sec
C. Oxygenation
D. Survival

Pulmonary Medicine and Critical Care:Question 61


A 40-year-old man is evaluated because of a positive tuberculin skin test after a pre-
employment physical examination. There is a local reaction with induration measured
at 22 mm. Chest radiograph is essentially normal. He is asymptomatic except for an
occasional dry cough. He has no shortness of breath.
Which of the following is the best next step in this patients management?
A. Treatment for latent tuberculosis
B. Sputum induction
C. Isoniazid therapy for 9 months
D. Rifampin and pyrazinamide therapy for 6 months
E. CT scan of the chest

Pulmonary Medicine and Critical Care:Question 62


A 46-year-old man is evaluated because of ankle edema and exertional dyspnea
25

without angina. For several years, he has had fatigue, dysphagia with symptomatic
gastroesophageal reflux, and pain with blanching in his fingers while shoveling snow.
He takes a β-blocker for mild hypertension.
On physical examination, he has perioral skin tightening, numerous telangiectasias
on both sun-exposed and protected surfaces, and ‘sausage shaped” fingers without
digital ulcerations. His lungs are clear. On cardiac examination, he has a right
parasternal lift with a loud pulmonic valve component. The S2 does not split on
inspiration. He has pitting edema to the mid-shin bilaterally and 8 cm of jugular
venous distention while seated at 45 degrees from horizontal. Left ventricular
systolic and diastolic functions are normal on transthoracic echocardiogram. Both the
right atrium and right ventricle are dilated with moderate tricuspid regurgitation.
Pulmonary artery pressure is estimated to be 52 mm Hg by echocardiography.
Which of the following statements is true about patients with these symptoms?
A. Pulmonary hypertension is a frequent cause of death.
B. Pulmonary hypertension indicates an overlap syndrome.
C. Pulmonary hypertension occurs only in the setting of an associated interstitial lung
disease.
D. Recurrent venous thromboembolism due to an associated hypercoagulable state is
the usual cause of pulmonary hypertension.

Pulmonary Medicine and Critical Care:Question 63


A 55-year-old man is evaluated in the emergency department because of an acute,
severe asthma attack; he is hospitalized in the intensive care unit for aggressive
medical therapy and monitoring. He is expectorating thick greenish sputum.
His medical history includes hypertension, cholecystectomy, and glaucoma. Chest
radiograph reveals hyperinflation only. Medical therapy in the emergency department
included repeated doses of aerosolized albuterol and ipratropium, as well as
methylprednisolone, 125mg administered intravenously.
Peak expiratory flow rate is unimproved at 80 L/min.
Which of the following is the most appropriate next step in this patient’s
management?
A. Nebulized ipratropium bromide administered by face mask
B. Intravenous magnesium sulfate
C. Broad-spectrum antibiotics targeting community-acquired respiratory pathogens
D. Inhaled corticosteroids

Pulmonary Medicine and Critical Care:Question 64


A 62-year-old woman with moderate emphysema is evaluated during a routine visit.
She has chronic dyspnea on exertion but has no cough or sputum production. She
uses supplemental oxygen, 2 L/min, when sleeping and on exertion. She currently
uses albuterol and ipratropium four times per day, and salmeterol and theophylline
twice per day. She is currently enrolled in a pulmonary rehabilitation program and is
concerned about ‘catching a cold” from other people enrolled in the pulmonary
rehabilitation program.
What is the best advice for this patient?
A. Take a daily antibiotic (long-term suppressive antibiotic therapy) to prevent
pneumonia.
B. Practice good hand washing, attempt to avoid close prolonged contact with ill
persons, and take pneumococcal and annual influenza vaccine.
C. Avoid any social functions where there will be large crowds.
D. Discontinue attendance at the pulmonary rehabilitation program.
26

Pulmonary Medicine and Critical Care:Question 65


A 35-year-old man is evaluated because of a 2-week history of low-grade fevers,
fatigue, cough, pleuritic chest pain, and increasing dyspnea on exertion. He is a
construction worker and is having difficulty performing his usual tasks. He has a 10-
pack-year history of cigarette smoking.
On physical examination, he has right chest pain but no respiratory distress at rest.
Temperature is 38.2 C (100.8 F), pulse rate is 112/min and regular, and respiration
rate is 20/min. There is evidence of a right pleural effusion and no other
abnormalities.
Peripheral blood leukocyte count is 9,000/μL, with 80% neutrophils and 15%
lymphocytes. Liver function test results are normal. Chest radiograph shows a
moderate right pleural effusion with minimal contralateral shift and no parenchymal
infiltrates.
Thoracentesis yields minimally turbid yellow fluid with results as follows :
Pleural fluid nucleated cell count 3000/1iL with 5% neutrophils, 85% lymphocytes,
and 1% macrophages
Pleural fluid total protein 5.2 g/dL
Pleural fluid serum lactate dehydrogenase 230 U/L
Pleural fluid glucose 80 mg/dL
Pleural fluid pH 7.36
Pleural fluid Gram and acid-fast bacilli stains are negative. Tuberculin skin test is
negative. Cytologic evaluation for malignant cells is negative.
What is the most likely diagnosis?
A. Tuberculous pleurisy
B. Lung cancer
C. Parapneumonic effusion
D. Pulmonary embolism
E. Benign asbestos pleural effusion

Pulmonary Medicine and Critical Care:Question 66


A 72-year-old man is evaluated because of difficulty maintaining sleep once he goes
to bed. He often awakens at around 2 AM. He reports no history of snoring, has no
daytime symptoms, and has no difficulty initiating sleep. He has end-stage renal
disease and undergoes outpatient dialysis three times per week. His medications
include epoetin alfa, stress vitamins, lisinopril, and sertraline.
On physical examination, blood pressure is 150/70 mm Hg. The pharynx is normal,
chest is clear, and the heart is normal. There is no peripheral edema. Serum ferritin
is 65 mg/dL. During a 5-hr sleep recording, the patient had 265 periodic limb
movements, with 156 associated with arousal from sleep. One epoch from the
patients sleep study is shown.
27

Which of the following medications is most effective for the treatment of this
patient’s sleep disorder?
A. Clonazepam
B. Pramipexole
C. Oxycodone
D. Gabapentin
E. Ketorolac

Pulmonary Medicine and Critical Care:Question 67


A 40-year-old woman, a nurse, is evaluated because of worsening asthma
symptoms. She has had mild, intermittent asthma since college, for which she has
been using an albuterol inhaler as needed, usually less than once a month.
During the past 3 months, she has experienced cough, tightness of the chest, and
wheezing, which improve after the use of inhaled albuterol. She uses the inhaler
twice a day on average and has awakened at least twice a week with nocturnal
cough.
She works three consecutive 12-hour day shifts, and the cough is regularly worse at
the end of each shift. During her days off, she has fewer asthma symptoms and feels
significantly better by the time she returns to work. She has a history of allergic
rhinitis that has also recently become more symptomatic. Approximately 6 months
ago, she acquired a kitten that sleeps in the bedroom. She has lived in her home for
6 years, and it is carpeted and has heavy draperies.
Chest examination is notable for good air entry. There are scattered end-expiratory
wheezes.
In addition to treatment with inhaled corticosteroids, which of the following
interventions is most likely to benefit this patient?
A. Treatment with an oral antihistamine
B. Getting rid of the kitten
C. Avoiding exposure to latex products
D. Removing the carpets and draperies from her home
E. Treatment with a leukotriene-modifying drug

Pulmonary Medicine and Critical Care:Question 68


A 60-year-old woman is hospitalized for an exacerbation of chronic obstructive
pulmonary disease. She is treated with ipratropium bromide by nebulizer every 4
hours; intravenous azithromycin, 500 mg/d; methylprednisolone, 125 mg
28

intravenously every 6 hours; and oxygen by nasal cannula. During the first 2 hospital
days, her condition remains unchanged. On hospital day 3, she develops increased
dyspnea and a cough productive of sputum.
On physical examination, she is awake and alert and in moderate respiratory
distress. Her temperature is 36.7 C (98 F), pulse rate is 110/min, respiration rate is
20/min, and blood pressure is 150/90 mm Hg. Her lungs are hyperresonant to
percussion, with accessory muscle use, poor air movement, mild wheezing, and no
crackles. Chest radiograph demonstrates hyperinflation, with no other abnormalities.
Leukocyte count is 16,000/μL. Arterial blood gas measurements, with the patient
breathing 1.5 L oxygen, show PO2 of 55 mm Hg, PaCO2 of 55 mm Hg, and pH of
7.32. She is transferred to the intensive care unit for close observation and possible
assisted ventilation.
Which of the following is most appropriate additional management for this patient?
A. Discontinue azithromycin and begin levofloxacin.
B. Begin intravenous aminophylline.
C. Increase methylprednisolone to 250 mg every 6 hours.
D. Add albuterol to ipratropium bromide by nebulizer every 4 hours.
E. Increase oxygen by nasal cannula to 3 L/min.

Pulmonary Medicine and Critical Care:Question 69


A 22-year-old man is evaluated in the emergency department after being rescued
from a house fire. The paramedics administered 100% oxygen by nonrebreather
mask, and his oxygen saturation is 99% by pulse oximetry. Pulse rate is 115/min,
respiration rate is 30/min, and blood pressure is 120/70 mm Hg. There is no stridor,
no use of the accessory muscles of respiration, and the patient’s mental status is
clear.
There are mild facial burns as well as singed nasal hairs and soot in the posterior
oropharynx. Lungs are clear with good air entry bilaterally.
Which one of the following is most correct about this patient’s condition?
A. The pressure of facial burns reliably predicts the presence of airway injury.
B. Direct laryngoscopy or bronchoscopy should be performed for airway injury.
C. This patient is at low risk for delayed 2 days postinhalation) injury to the lower
airways.
D. Dexamethasone should be empirically administered.

Pulmonary Medicine and Critical Care:Question 70


A 19-year-old competitive female swimmer with no history of asthma is evaluated
because of dyspnea and cough after any exercise, symptoms she has had since she
was accidentally exposed to chlorine powder that was being added to a swimming
pool in the university 6 weeks ago. Within hours of the exposure, she and several
teammates noted burning of the eyes and mucous membranes, as well as briefly
incapacitating chest tightness and cough. Physical examination shows no
abnormalities, and results of chest radiograph and spirometry are also normal.
Which of the following statements regarding this case is true?
A. Spirometry is invariably abnormal.
B. Inhaled corticosteroids and β-agonists are usually ineffective.
C. Methacholine challenge testing is likely to show an abnormality.
D. Symptoms generally resolve within 3 months of exposure.
E. The patient must avoid further exposure to chlorinated water.

Pulmonary Medicine and Critical Care:Question 71


A 23-year-old woman is evaluated in the emergency department because of
29

generalized pruritic rash and difficulty breathing, both of which developed minutes
after she ate at a nearby restaurant. The patient developed a rash some years ago
after eating in a restaurant, but she is unsure whether she has food allergy.
On physical examination, the patient is very anxious and in significant respiratory
distress. Her temperature is 36.7 C (98 F), pulse rate is 120/min and regular,
respiration rate is 30/min, and blood pressure is 85/50 mm Hg. Periorbital edema is
present. Diffuse wheezing and prolonged expiratory phase are noted on auscultation.
There is diffuse erythema over the trunk and extremities.
The patient is promptly treated with subcutaneous epinephrine, intravenous fluids,
inhaled β-agonist
s, corticosteroids, and antihistamines. After 2 hours she feels better, her vital signs
have returned to normal, and there is no wheezing on minimal prolongation of
expiratory phase.
Which of the following is the most appropriate next step in this patient’s
management?
A. Observe her in the emergency room for another hour, and discharge her if she
remains stable.
B. Discharge her and prescribe a tapering dose of corticosteroids for 1 week.
C. Hospitalize her for further observation.
D. Discharge her with an epinephrine and instructions to consult an allergist.

Pulmonary Medicine and Critical Care:Question 72


A 74-year-old male nursing home resident with a history of mild dementia and
chronic obstructive pulmonary disease is hospitalized because of a 2-day history of
upper respiratory symptoms and severe dyspnea. He is intubated and mechanical
ventilation is begun. Therapy with intravenous corticosteroids, antibiotics, and
bronchodilators is initiated.
He soon becomes hypotensive and hypoxemic with rising peak airway pressures.
Chest radiograph reveals a left-sided tension pneumothorax, and a chest tube is
placed with immediate improvement in vital signs and oxygenation. Five minutes
later, he becomes severely agitated, hypertensive, and diaphoretic, despite good
oxygenation, acceptable airway pressures, and a functioning chest tube.
Which one of the following is the best next step in this patients management?
A. Vecuronium
B. Fentanyl
C. Midazolam
D. Propofol
E. Haloperidol

Pulmonary Medicine and Critical Care:Question 73


A 60-year-old man with chronic obstructive pulmonary disease is evaluated because
of persistent cough and sputum production. He reports no fevers or weight loss. He
has had increased sputum production over the past 6 to 9 months and moderately
severe reduction in forced expiratory volume in 1 sec (FEV1 55% of predicted). His
symptoms have been unresponsive to antibiotics. He has an 80-pack-year history of
cigarette smoking.
Chest radiograph reveals multiple small nodules in the left upper lung zone without
infiltrate. Serologic testing for HIV is negative. Sputum cultures are negative for
bacteria. A test for acid-fast bacillus is negative, but culture grows Mycobacterium
30

avium-intracellulare with 1+ growth.


Which of the following is the best next step in this patients management?
A. Treat him for Mycobacterium avium-intracellulare without further testing.
B. Perform bronchoscopy with bronchioalveolar lavage and transbronchial biopsy.
C. Order a HRCT scan of the chest.
D. Order a sputum culture for Mycobacterium avium-intracellulare.

Pulmonary Medicine and Critical Care:Question 74


A 21-year-old man presents with a 4-hour history of cough and dyspnea after
spending the day moving stored hay. A college student, he lives with his parents and
two younger siblings on the dairy farm where he was born and raised. No one else in
the family has similar symptoms. He actively participates in small grain production,
soybean grinding for animal feed, and the cutting, baling, and storage of hay. He has
had similar episodes the past two summers; each time he was hospitalized for
atypical pneumonia, and his symptoms resolved quickly with antibiotics and inhaled
bronchodilators.
On physical examination, his temperature is 38.4 C (101 .1 F), respiration rate is
22/min, and blood pressure is 142/84 mm Hg. He has a few bibasilar crackles. There
is no jugular venous distention or peripheral edema. Chest radiograph reveals patchy
infiltrates in the lower lobes and right middle lobe interpreted by the radiologist as
compatible with bronchopneumonia. Complete blood count shows a leukocyte count
of 14,500/μL with 11,400 polymorphonuclear cells, 200 band forms, and 150
eosinophils. What additional information would most strongly support the diagnosis?
A. A precipitin band toThermoactinomyces sacchari on a farmers lung panel
B. Immediate skin test reactivity to ‘mold’ extract
C. Improvement with observation alone and recurrence of symptoms after returning
to the farm
D. Bedside spirometry demonstrating reversible airflow obstruction after
bronchodilator administration

Pulmonary Medicine and Critical Care:Question 75


A 50-year-old woman is evaluated on a winter morning because of a 1-week history
of daily headaches with some nausea and flu-like symptoms. Other people in her
house have had similar headaches. She typically awakens with a headache but feels
better after going to work without any specific therapy. The headache recurs several
hours after she arrives home. She is a nonsmoker. Her vital signs, physical
examination, leukocyte count, and hemoglobin are normal.
Which of the following tests is most likely to help make the diagnosis?
A. Pulse oximetry
B. Lactate level
C. Electrocardiogram
D. Carboxyhemoglobin level

Pulmonary Medicine and Critical Care:Question 76


A 56-year-old construction worker, a heavy smoker, sustains severe trauma to his
left chest. Chest pain is severe for several minutes but subsides over the next hour.
Because the chest pain does not resolve completely, he is evaluated in the
emergency department 2 hours later, where results of a chest radiograph and
complete blood count with differential are normal. The next day, he leaves on a
week-long vacation to South America. During that time, he has intermittent chest
discomfort and gradually increasing dyspnea with exertion.
Upon returning home, he sees his physician because of dyspnea. Chest radiograph
31

shows a large left-sided pleural effusion with minimal contralateral shift. At


thoracentesis, 500 cc of brownish-colored fluid is removed and analyzed with the
following results.
Pleural fluid nucleated cell count 4000/mL with 10% neutrophils, 30% lymphocytes,
15% macrophages, and 45% eosinophils
Pleural fluid hematocrit 10%
Pleural fluid total protein 4 g/dL
Pleural fluid serum lactate dehydrogenase 200 UIL
Pleural fluid glucose 80 mg/dL
Pleural fluid pH 7.35
Cytology test results are negative.
Pain medication is prescribed for the patient. When he returns 14 days later,
complete blood count shows a leukocyte count of 9000/μL with 20% eosinophils and
chest radiograph shows that the pleural effusion has decreased substantially.
Which of the following is the most likely diagnosis?
A. Benign asbestos pleural effusion
B. Paragonimiasis
C. Post-traumatic hemothorax
D. Lung cancer
E. Pulmonary infarction

Pulmonary Medicine and Critical Care:Question 77


A 60-year-old man with a history of severe chronic obstructive pulmonary disease is
evaluated in the emergency department because of an exacerbation of his illness
characterized by increasing shortness of breath over the past 2 days. He has a 90-
pack-year history of cigarette smoking (he stopped smoking 6 months ago) and
dyspnea on exertion, with exercise tolerance of one-half block. He uses home
oxygen. His baseline forced expiratory volume in 1 sec (FEV1) is 0.80 Land forced
vital capacity (FVC) is 2.4 L. He has a cough productive of minimal clear sputum.
On physical examination, his temperature is 38.3 C (101.0 F), pulse rate is 110/min,
and respiratory rate is 30/min. He is alert and anxious, using accessory muscles; he
has decreased breath sounds bilaterally and wheezing with a prolonged expiratory
phase.
Laboratory studies:
Leukocyte count 1 0,200/μL
Serum creatinine 1.3 mg/dL
Blood urea nitrogen 30 mg/dL
Serum bicarbonate 32 meq/L
Arterial blood gases (with the patient breathing room air)
PaO2 54 mm Hg
PaCO2 56 mm Hg
pH 7.29
Chest radiograph shows no pulmonary infiltrate. Despite treatment with albuterol
and ipratropium by nebulizer, the patient remains dyspneic.
Which of the following is the best next step in this patients management?
A. Increase oxygen concentration and continue albuterol/ipratropium treatment.
B. Start noninvasive positive-pressure ventilation.
C. Start continuous positive-airway pressure ventilation.
D. Start intravenous antibiotic therapy.

Pulmonary Medicine and Critical Care:Question 78


A 47-year-old man is evaluated in the emergency department because of fatigue and
headaches, which are more frequent in the morning. He also notes nocturia and
32

night sweats.
On physical examination, his body mass index is 42. His heart and lungs are normal.
Laboratory studies yield normal results except hemoglobin of 16.8 g/dL. Trace
edema of the lower extremities is noted. Chest radiograph shows that his heart is of
normal size and that his lung fields are clear.
Which of the following is most likely to show the cause of the fatigue and headaches?

A. Echocardiogram
B. Spirometry
C. Beck Depression Inventory
D. CT scan of the brain
E. Overnight polysomnography

Pulmonary Medicine and Critical Care:Question 79


A 28-year-old man with a diffuse petechial rash is hospitalized in the intensive care
unit with a clinical diagnosis of meningococcemia and septic shock. He was intubated
in the emergency department for airway protection. Results of a lumbar puncture are
compatible with bacterial meningitis. Intravenous penicillin is initiated. Chest
radiograph shows no abnormality.
On physical examination, his temperature is 39 C (102.2 F) and pulse rate is
120/min. An arterial line is inserted, and mean arterial pressure is 68 mm Hg with
administration of intravenous norepinephrine. He is sedated and mechanically
ventilated. Pulse oximetry is 100% with fractional inspired oxygen (FiO 2 set at .5,
and positive end-expiratory pressure of 5 cm H20.
Since his arrival in the emergency department 6 hours ago he has received 3 L
normal saline. His urine output has decreased and is currently .25 mL/kg per hour
over last 2 hours. Leukocyte count is 22,000/μL and platelet count is 40,000/μL
(platelet count was 60,00/μL at the time of the lumbar puncture).
Which of the following therapeutic decisions is most appropriate at this time?
A. Transfuse platelets.
B. Increase norepinephrine to achieve a mean arterial pressure of 75 mm Hg or
higher.
C. Switch from norepinephrine to dopamine.
D. Administer a 1000-mL bolus normal saline.
E. Administer furosemide.

Pulmonary Medicine and Critical Care:Question 80


A 55-year-old man is evaluated in the emergency department because of a 5-day
history of increased dyspnea and cough productive of yellow-green mucus. Nine
months ago, he required prolonged mechanical ventilation for an exacerbation of
chronic obstructive pulmonary disease. His medical history includes hypertension.
On recent pulmonary function testing, the forced expiratory volume in 1 sec (FEV1)
was 38% of predicted. His temperature is 38.1 C (100.5 F), pulse rate is 135/min
and irregular, respiration rate is 25/min, and blood pressure is 90/65 mm Hg. He is
mildly lethargic but arousable and oriented. He has a weak cough with pooling of
secretions in the oral cavity and hypopharynx. Electrocardiogram demonstrates
multifocal atrial tachycardia. Chest examination reveals accessory muscle use,
coarse rhonchi, and decreased breath sounds at the right base. Leukocyte count is 1
7,000/μL. Chest radiograph shows right lower lobe consolidation. With the patient
breathing 4 L oxygen, arterial blood gases show a FO2 of 50 mm Hg, a PCO2 of 65
mm Hg, and a pH of 7.25. Therapy with methylprednisolone sodium succinate,
125mg intravenously every 6 h, is intiated, along with nebulized albuterol and
ipratropium bromide every 4 h, and azithromycin, 500 mg administered
33

intravenously daily.
Which of the following is the most appropriate additional management?
A. Intubate and begin mechanical ventilation.
B. Initiate mucloytic therapy, chest physiotherapy, and oral-tracheal suctioning.
C. Administer a helium-oxygen mixture of 70%:30%, delivered by face mask.
D. Initiate bilevel noninvasive positive-pressure ventilation by face mask
E. Administer 40% oxygen by Venturi mask

Pulmonary Medicine and Critical Care:Question 81


A 70-year-old man is ready to be discharged in October from the hospital after
treatment of an exacerbation of his chronic obstructive pulmonary disease. His only
other medical problems include stable angina and hypertension. He has normal renal
function and is well nourished. He quit smoking 2 years ago. He received pneumonia
vaccine 2 years ago and influenza vaccine 1 year ago.
Which of the following is true of his vaccination status?
A. He should receive pneumonia vaccine and influenza vaccine.
B. He should receive pneumonia vaccine.
C. He should receive influenza vaccine.
D. He should receive neither pneumonia vaccine nor influenza vaccine.

Pulmonary Medicine and Critical Care:Question 82


A 25-year-old man is evaluated approximately 12 hours after a suicide attempt in
which he ingested 30 pills of an extra-strength acetaminophen product (500 mg/pill),
10 ibuprofen tablets (200 mg), and 7 pills of alprazolam (dosage unknown). He is
sleepy but arousable and oriented. He has nausea and vomiting. His temperature is
37 C (98.6 F), pulse rate is 90/min, respiration rate is 16/min, and blood pressure is
120/76 mm Hg.
In addition to obtaining an acetaminophen level, which of the following is the most
important initial intervention?
A. Administer oral N-acetylcysteine.
B. Administer oral activated charcoal.
C. Administer intravenous naloxone.
D. Administer intravenous flumazenil.

Pulmonary Medicine and Critical Care:Question 83


A 53-year-old woman with myasthenia gravis is hospitalized because of a 3-day
history of progressive dyspnea. She ran out of her medications 5 days ago.
On admission she is breathing shallowly and having difficulty speaking. She has mild
weakness of the extremities. Her lungs and heart are normal. A chest radiograph is
normal. Bedside testing shows a vital capacity of 1.5 L (34% of predicted) and a
maximum negative inspiratory pressure of 60 cm H2O.
Prednisone and pyridostigmine therapy is initiated. Over the next 24 hours, the
weakness in the extremities is unchanged. The patient is noted to be having difficulty
managing secretions and aspirates them on one occasion.
What is the best next step in this patient’s management?
A. Administer noninvasive ventilation through a nasal mask.
B. Order chest physical therapy with postural drainage.
C. Initiate orotracheal intubation.
D. Order that she take nothing by mouth and administer medications intravenously.

Pulmonary Medicine and Critical Care:Question 84


A thin, 19-year-old woman is evaluated because of chronic cough productive of
34

green sputum. She reports a long-standing history of loose bowel movements and
intermittent abdominal bloating. Physical examination is notable for bilateral nasal
polyps, crackles in the upper lung zones bilaterally, and early digital clubbing.
Which of the following is the best initial test to evaluate this patient for cystic
fibrosis?
A. Gene mutation test (buccal swab or blood test)
B. Quantitative pilocarpine iontophoresis sweat chloride test
C. 72-hour fecal fat test
D. Measurement of nasal potential difference
E. Sputum culture

Pulmonary Medicine and Critical Care:Question 85


A 34-year-old man is evaluated because of recurrent sleepiness in the late afternoon,
especially at work. His excessive sleepiness dates back to his high school years and
seems unrelated to activity or schedule. He has no history of snoring or witnessed
apneas, but he does awaken frequently at night, especially in the early morning. He
reports that he has recurrent jaw weakness after laughing at a joke or becoming
angry. Particularly troubling to the patient are recurrent nightmares occurring shortly
after he falls asleep.
On physical examination, the patient is 178 cm (70 in) tall and weighs 70.3 kg (155
Ib). His blood pressure is 120/75 mm Hg. His lungs are clear on auscultation; heart
sounds are regular and there are no gallops or murmurs. Neurologic examination
reveals no abnormalities. An overnight polysomnogram reveals no evidence of
obstructive sleep apnea. On a multiple sleep latency nap test, mean sleep latency is
3.5 mm, and two sleep-onset periods of rapid eye movement are observed.
Which of the following is the most likely diagnosis?
A. Temporal lobe epilepsy
B. Panic disorder
C. Narcolepsy
D. Amphetamine abuse
E. Parasomnia

Pulmonary Medicine and Critical Care:Question 86


A 48-year-old previously healthy man has a CT scan of the thorax with intravenous
contrast. After administration of the contrast material the patient feels tightness in
the throat and diffuse itching. He also feels light-headed.
On physical examination, he is diaphoretic and anxious. His pulse rate is 125/min,
his respiration rate is 28/min, and his blood pressure is 70/50 mm Hg. There is mild
wheezing on auscultation. A rash is
noted on the patient’s chest and upper extremities.
Which one of the following is the most important immediate treatment?
A. Dobutamine
B. Dopamine
C. Corticosteroids
D. Epinephrine

Pulmonary Medicine and Critical Care:Question 87


A 65-year-old hospitalized man, develops a rapidly progressing bilateral
bronchopneumonia requiring mechanical ventilation. His fractional inspired oxygen
level (nO2) is 0.6, and positive end-expiratory pressure is 12 cm H2O. He was
agitated before being intubated and is now being treated with sedative agents.
During the past 12 hours, he has also developed a moderate increase in creatinine
35

concentration with normal urine output. His hemodynamic status has remained
stable. Prothrombin time and partial thromboplastin time are mildly elevated. Platelet
count is mildly depressed, and his leukocyte count is elevated. His blood pressure is
normal.
In addition to the proper antibiotics, which of the following is likely to benefit this
patient?
A. Antithrombin III
B. A single large dose of methylprednisolone
C. Low-dose (stress dose) corticosteroids
D. Drotrecogin alfa (activated)
E. Low-dose dopamine (2 μg/kg of body weight/mm)

Pulmonary Medicine and Critical Care:Question 88


A 23-year-old woman is evaluated in the emergency department because of right-
sided hemiplegia and depressed mental status. Her temperature is 39.2 C (102.5 F).
Laboratory studies show a platelet count of 22,000/μL, serum creatinine of 2.5
mg/dL, serum lactate dehydrogenase of 458 U/L, hemoglobin of 8.3 g/dL, normal
coagulation parameters, and schistocytes on her peripheral blood smear. She is
intubated and mechanical ventilation is begun. Plasmapheresis is initiated for
thrombotic thrombocytopenic purpura.
Her condition improves, and after 8 days she can follow simple commands and is
extubated. Three hours later she seizes and becomes hypoxemic and is reintubated.
Chest radiograph shows diffuse infiltrates consistent with neurogenic pulmonary
edema or acute respiratory distress syndrome. She is severely agitated and
lorazepam, 4 mg/hr, is begun.
Ten days later, the thrombotic thrombocytopenic purpura is in remission and her
respiratory status has improved, but she cannot be weaned because she remains
unresponsive 2 days after stopping the lorazepam. A CT scan of the brain is
unremarkable. Electroencephalogram shows “diffuse slowing.”
Which of the following is the best next step in this patient’s management?
A. Lumbar puncture
B. Resume plasmapheresis
C. Perform a tracheostomy and place a percutaneous gastrostomy tube
D. Monitor the patient off sedatives and wait for her mental status to improve
E. Measure serum levels of propylene glycol

Pulmonary Medicine and Critical Care:Question 89


A 60-year-old man, a farmer, is evaluated because of severe, progressive shortness
of breath and cough, such that he is no longer able to work. He has no history of
cardiac or pulmonary disease but states that approximately 6 weeks ago he
experienced self-limited chest tightness, wheezing, and cough that began a few
hours after he checked the silage level in his silo. He “took it easy” for a couple of
days and then felt well enough to return to work. He was well until 2 weeks ago
when his presenting symptoms began.
On physical examination, he has tachypnea and early inspiratory crackles. Chest
radiograph shows only hyperinflation. Spirometry is notable for severe obstruction,
with forced expiratory volume in 1 sec (FEV1) 35% of predicted, forced vital capacity
(FVC) 90% of predicted, and FEV1/FVC 45% of predicted.
What is the most likely cause of this patient’s symptoms?
A. Hypersensitivity pneumonitis
B. Bronchiolitis obliterans
C. Occupational asthma
D. Reactive airways dysfunction syndrome
36

E. Acute respiratory distress syndrome

Pulmonary Medicine and Critical Care:Question 90


A 66-year-old woman of normal body weight is evaluated in the emergency
department because of weakness. She has diabetes mellitus and hypertension and 1
week ago underwent cholecystectomy.
On physical examination, she weighs 55 kg (121.3 Ib). Her temperature is 38.1 C
(100.5 F), pulse rate is 128/min, respiration rate is 28/min, and blood pressure is
80/60 mm Hg. Her abdomen is tender and bowel sounds are present. She has no
chest pain but says she is dyspneic. Pulse oximetry is 88%, with the patient
breathing room air.
Supplemental oxygen is initiated. Electrocardiogram reveals T-wave inversion
laterally. Hemoglobin is 13.5 g/dL. The patient remains hypotensive after 1 L of
normal saline is infused, with worsening oxygenation.
Which of the following diagnostic studies is most appropriate at this time?
A. CT scan of the abdomen and chest
B. Ventilation/perfusion lung scan
C. Transthoracic echocardiography
D. Transesophageal echocardiography

Pulmonary Medicine and Critical Care:Question 91


A 64-year-old man who was diagnosed with amyotrophic lateral sclerosis 2 years ago
is evaluated because of weakness in the legs that prevents him from standing and
walking. He also reports that dyspnea forces him to sleep on his side. He has had no
problems swallowing.
On physical examination, he has significant wasting of the thenar and hypothenar
eminences and weakness of the hands. His cranial nerve function is intact. His vital
capacity is 40% of predicted and negative inspiratory pressure is -45 cm H20. Peak
cough expiratory flow is 400 L/min.
Which of the following interventions has been shown to prolong life in patients like
this one?
A. Prophylactic use of antibiotics to prevent pneumonia
B. Daily home chest percussion therapy and postural drainage
C. Nocturnal noninvasive ventilation through a face mask
D. Daily mucolytic therapy

Pulmonary Medicine and Critical Care:Question 92


A 68-year-old Lithuanian man with limited command of English presents for an initial
outpatient visit. A letter from a doctor in another city indicates that the man has
advanced idiopathic pulmonary fibrosis confirmed by surgical lung biopsy. Lung
disease has progressed over 2 years despite an extended trial of prednisone and
cyclophosphamide. Three months ago, pulmonary function studies showed a forced
vital capacity (FVC) 59% of predicted. A sample of arterial blood obtained at that
time showed a PCO2 of 49 mm Hg and a PO2 of 51 mm Hg, with the patient
breathing room air.
The office nurse reports that the man arrived in a wheelchair and has obvious
peripheral edema. The patients daughter privately asks the physician to take on the
care of her father, but to limit treatment to long-term oxygen supplementation only
and to avoid any mention of prognosis. ‘He would not want to be told that he is
dying, she says.
Which of the following is most appropriate response to the daughters request?
A. Agree to respect the patients wishes as conveyed by the daughter.
37

B. Respectfully decline to see the patient.


C. Agree to evaluate the patient once to determine if he has urgent health care
needs, and advise the patients daughter that any questions the patient asks will be
answered honestly and completely.
D. See the patient, advise him of his diagnosis and prognosis, and explore with him
in English his personal preferences for continuing care.

Pulmonary Medicine and Critical Care:Question 93


A 19-year-old woman is evaluated in the emergency department because of
hypoxemia, respiratory depression, seizures, and confusion. She was in a crowded
convention hall when she (and more than 50 other people) suddenly became ill.
Initial symptoms included restlessness, excessive salivation, vomiting, diarrhea, and
diaphoresis. She is intubated and mechanical ventilation is initiated.
On physical examination, her pulse rate is 50/min and regular, and her blood
pressure is 160/95 mm Hg. On mechanical ventilation, oxygen saturation is 89% on
fraction of inspired oxygen (FiO 2) of 1.0. She is minimally responsive, but muscle
fasciculation is present. She is also profusely diaphoretic.
Examination of the eyes shows miosis and excessive tearing. There are copious
pulmonary secretions in the endotracheal tube and coarse breath sounds bilaterally
with wheezing.
Which of the following statements is most correct about the management of this
patient?
A. Decontamination of the patient is not necessary in this kind of exposure, because
the agent is
inhaled, rather than absorbed through the skin.
B. Health care workers are not at risk when in close contact with this patient.
C. Atropine administration will reverse the respiratory depression.
D. Pralidoxime (2-PAM) should be administered.

Pulmonary Medicine and Critical Care:Question 94


A 60-year-old man is evaluated because of a 6-week history of progressive dyspnea
on exertion, fatigue, a decrease in appetite, and a weight loss of 1.8 kg (4 Ib). He
has a 30-pack-year history of cigarette smoking and drinks two or three cocktails
every evening. He has no gastrointestinal complaints and no history of a febrile
illness. On physical examination, he is afebrile with normal vital signs. The only
abnormalities noted on chest examination are findings compatible with a right pleural
effusion. Chest radiograph confirms a pleural effusion occupying 40% of the right
hemithorax without evidence of loculation. There are no obvious parenchymal lesions
and no mediastinal adenopathy. Results of pleural fluid analysis are as follows.
Pleural fluid nucleated cell count 2800/μL with 10% neutrophils, 50% lymphocytes,
30% macrophages, and 10% mesothelial cells
Pleural fluid total protein 3.8 g/dL (pleural fluid/serum ratio 0.60)
Pleural fluid serum lactate dehydrogenase 210 U/L (ratio of pleural fluid to upper
limits of normal serum lactate dehydrogenase 0.72)
Pleural fluid amylase 30 mg/dL (pleural fluid/serum ratio 0.5)
Pleural fluid glucose 50 mg/dL
Pleural fluid pH 7.26
Which of the following is the most likely diagnosis?
A. Complicated parapneumonic effusion
B. Esophageal rupture
C. Rheumatoid pleurisy
D. Acute pancreatitis
38

E. Malignant effusion

Pulmonary Medicine and Critical Care:Question 95


A 29-year-old woman with active systemic lupus erythematosus is hospitalized in the
intensive care unit with a diagnosis of community-acquired pneumonia. She had
been taking prednisone, 30 mg/d, but was weaned off it 6 months ago.
She is febrile with an elevated leukocyte count. Antibiotic therapy was started in the
emergency department. Laboratory studies obtained in the emergency department
revealed a leukocyte count of 6000/μL. with a left shift, hemoglobin of 10 g/dL, and
a platelet count of 20,000/μL. Mild renal insufficiency is present. Following arrival in
the intensive care unit, she has become hypotensive with a blood pressure of 70/40
mm Hg. In addition to fluid resuscitation and norepinephrine therapy as needed to
maintain adequate blood pressure, which of the following is the most appropriate
next step in this patient’s management?
A. Perform an adrenocorticotropic hormone stimulation test, and initiate
corticosteroid therapy if the results are abnormal.
B. Initiate therapy with fludrocortisone.
C. Administer methylprednisolone, 2 g intravenously.
D. Administer intravenous dexamethasone, and perform an ACTH stimulation test

Pulmonary Medicine and Critical Care:Question 96


Which of the following is a potential beneficial effect of lung-volume-reduction
surgery for properly selected patients?
A. It may significantly decrease lung elastic recoil.
B. It may significantly increase pulmonary hyperinflation.
C. It may be used as a bridge to lung transplantation in patients too ill to wait on a
transplantation waiting list.
D. It may significantly increase the work of breathing.

Pulmonary Medicine and Critical Care:Question 97


An 18-year-old woman is evaluated in the emergency department because of a 6-
day history of illness characterized by low-grade fever and dry, hacking cough. Early
this morning, she developed a shaking chill and fever of 40 C (104 F). A sample of
grossly purulent sputum shows intracellular gram-positive cocci in pairs, and a chest
radiograph shows diffuse bilateral infiltrates. A diagnosis of pneumococcal
pneumonia is made. Her respiration rate is 36/min, she is experiencing some mild
respiratory distress, and she is admitted to the intensive care unit. There she
requires emergent endotracheal intubation and mechanical ventilation.
What size should her tidal volurne be, and what additional inforrnation is necessary
to calculate the appropriate tidal volume?
A. Tidal volurne should be 6 rnL/ kg of ideal body weight; the patient’s height is
needed to calculate it.
B. Tidal volurne should be 12 mL/kg of ideal body weight; the patient’s height is
needed to calculate it.
C. Tidal volurne should be 6 mL/kg of ideal body weight; the patient’s actual weight
is needed to calculate it.
D. Tidal volurne should be 12 mL/kg of ideal body weight; the patient’s actual weight
ratio is needed to calculate it.

Pulmonary Medicine and Critical Care:Question 98


Which of the following is a recognized complication of obstructive sleep apnea?
A. Chronic renal insufficiency
39

B. Seizures
C. Gastric aspiration
D. Systemic hypertension
E. Anxiety disorder

Pulmonary Medicine and Critical Care:Question 99


A 67-year-old normal-weight woman is improving after intubation and mechanical
ventilation, which were initiated because of community-acquired pneumonia
involving both lungs.
Her leukocyte count is normal. She is afebrile and chest radiograph shows marked
improvement. Her current ventilator settings are pressure support ventilation with 16
cm H2O applied pressure, fractional inspired oxygen (FiO 2) .4, and positive end-
expiratory pressure 10 cm H2O. Her vital signs are stable, with a respiration rate of
12 and a pulse oximetry reading of 94%. Her maximal negative inspiratory pressure
is -35 cm H2O, and her vital capacity is 840 mL.
Which of the following is the most appropriate next step in this patient’s
management?
A. Perform a T-piece trial (humidified oxygen delivered through endotracheal tube
without application of
positive pressure).
B. Extubate and institute humidified oxygen.
C. Extubate and institute noninvasive ventilation.
D. Decrease positive end-expiratory pressure to 5 cm H2O.
E. Switch to synchronized intermittent mandatory ventilation.

Pulmonary Medicine and Critical Care:Question 100


A 62-year-old man is hospitalized because of an exacerbation of chronic obstructive
pulmonary disease. Noninvasive ventilation (expiratory pressure, 3 cm H2O, and
inspiratory pressure, 10 cm H2O) is initiated after the following values are obtained
for arterial blood gases during administration of supplemental oxygen: PaO2 68 mm
Hg; PaCO2 74 mm Hg; pH 7.13; and oxygen saturation 96%.
On physical examination, the patient appears sleepy but is arousable and oriented.
The nurse reports that the patient had an episode of emesis since he was last
examined. Pulse oximetry saturation has decreased to 88%.
Which of following is the best next step in this patients management?
A. Increase oxygen to maintain saturation at 92% or higher.
B. Orally intubate the patient, and provide mechanical ventilator support.
C. Measure arterial blood gases.
D. Increase the inspiratory and expiratory pressure setting by 3 to 5 cm H2O.

Pulmonary Medicine and Critical Care:Question 101


Which of the following surgeries pose the greatest risk to patients with advanced
chronic obstructive pulmonary disease?
A. Ophthalmologic procedures
B. Upper extremity surgical procedures
C. Upper abdominal and thoracic surgical procedures
D. Urologic surgical procedures using local anesthesia
E. Endoscopic sinus surgery

Pulmonary Medicine and Critical Care:Question 102


A 60-year-old man is evaluated because of a 1-year history of progressively severe
40

shortness of breath on exertion and a 3-month history of persistent, nonproductive


cough. He smoked one to two packs of cigarettes a day for 30 years but stopped
smoking 3 years ago.
On physical examination, he has pain in both knees without stiffness or swelling.
Crackles are audible over both lungs at the bases posteriorly. He has no peripheral
edema. Arterial oxygen saturation, measured by pulse oximetry with the patient at
rest and breathing room air is 94%. Oxygen saturation is 84% when measured as he
walks down the hall.
Chest radiograph shows abnormal interstitial markings in both lower lung zones.
High-resolution CT scan of the chest obtained without contrast reveals reticular
infiltrates most prominent in the periphery of the lower lobes, accompanied by
subpleural cysts and patchy, ground-glass opacities. Centrilobular emphysema is also
seen, primarily in the apices of the lungs.
Pulmonary function testing shows a forced expiratory volume in 1 sec (FEV1) 84% of
predicted, a forced vital capacity (FVC) 82% of predicted, and a diffusing lung
capacity for carbon monoxide (DLCO) 39% of predicted. Antinuclear antibody titer is
1:160 (one dilution above the upper limit of normal).
Which of the following is the most likely diagnosis?
A. Emphysema with “smoker’s lung”
B. Systemic lupus erythematosus with pulmonary involvement
C. Idiopathic pulmonary fibrosis
D. Idiopathic pulmonary fibrosis and emphysema
E. Systemic sclerosis (scleroderma) with lung involvement

Pulmonary Medicine and Critical Care:Question 103


Which of the following diseases has an increasing mortality rate in the United States?

A. Atherosclerotic heart disease


B. Stroke
C. Chronic obstructive pulmonary disease
D. Gastric cancer

Pulmonary Medicine and Critical Care:Question 104


A 45-year-old man is hospitalized in the intensive care unit with a diagnosis of
pneumococcal pneumonia. He is intubated and mechanically ventilated and is
receiving vasopressor therapy for septic shock. A pulmonary artery catheter is in
place. The most recent measurements include right atrial pressure of 10 mm Hg,
pulmonary artery pressure of 48/28 mm Hg, pulmonary artery occlusion pressure of
18 mm Hg, and cardiac index of 4.0. Mean arterial pressure is measured at 55 mm
Hg.
Which of the following sepsis-induced physiologic alterations should be targeted in
order to overcome the patient’s persistent hypotension?
A. Decreased arteriolar resistance
B. Increased pulmonary artery pressure
C. Diffuse capillary leak
D. Decreased contractility
E. Increased venous capacitance
41

ANSWERS

Pulmonary Medicine and Critical Care:Question 1


The correct answer is B
Educational Objectives
Recognize common drug interactions with medications used to treat chronic
obstructive pulmonary disease.
Critique
Ciprofloxacin and other quinolones inhibit hepatic metabolism of theophylline and can
cause a clinically important increase in serum theophylline levels. Elevated serum
theophylline levels commonly cause nausea or vomiting and occasionally seizures.
Because the therapeutic-to-toxic window is narrow for theophylline and many agents
interact with this drug, it is necessary to exercise caution when adding a new
medication to the regimens of patients already taking theophylline. Ciprofloxacin can
cause nausea but alone is not known to cause syncope, except by precipitating
anaphylaxis. Albuterol, lisinopril, oxygen, and ipratropium bromide do not
significantly interact with theophylline.

Pulmonary Medicine and Critical Care:Question 2


The correct answer is C
Educational Objectives
Diagnose exercise-induced asthma.
Critique
The most likely diagnosis is exercise-induced asthma. Although some patients with
this condition may also have an allergic component, there is no clear correlation
between the development of exercise-induced asthma and allergies. Exercise testing
for exercise-induced allergy should attempt to mimic the activity that induces the
shortness of breath. All patients with significant disease should also be instructed to
use prophylactic treatment 5 to 10 minutes before exercise, usually two puffs of a
medium-acting inhaled β-agonist (albuterol).
Patients with exercise-induced allergy may have equivocal results to methacholine
challenge testing that would not be helpful for diagnosis. Overnight oximetry would
provide no information regarding the onset of bronchospasm during exercise. In
addition, exercise testing can reveal exercise-induced laryngeal dysfunction that
sometimes mimics exercise-induced asthma.
The better term for exercise-induced asthma is exercise-induced bronchospasm,
because not all persons with the condition have asthma. The estimated prevalence of
exercise-induced bronchospasm ranges from 7% to more than 20% in the general
population.
Exercise-induced bronchospasm probably results from changes in airway physiology
triggered by the large volume of relatively cool, dry air inhaled during vigorous
activity. Bronchodilation is the more common first event during exercise and lasts for
1 to 3 minutes after exercise. In patients with exercise-induced bronchospasm, the
initial bronchodilation is followed by bronchoconstriction, which begins within 3
minutes, generally peaks within 10 to 15 minutes, and resolves by 60 minutes. Acute
bronchoconstriction was previously followed by late-phase bronchoconstriction in
some patients; however, the risk and severity of late-phase bronchoconstriction due
to exercise-induced bronchospasm is decreased when compared with allergen-
induced asthma.
42

Pulmonary Medicine and Critical Care:Question 3


The correct answer is D
Educational Objectives
Recognize the limitations of positron emission tomography scanning in the evaluation
of pulmonary nodules.
Critique
Positron emission tomography (PET) scanning uses 2-(fluorine-1 8) fluoro-2-deoxy-
D-glucose (FDG) as the positron emitter and measures the relative concentration of
the agent in the nodule. The concentration of the tracer is determined in part by the
metabolic activity of the tissue. Because FDG competes with glucose for uptake into
the nodule, elevated serum glucose levels can lead to a false-negative test. It is,
therefore, important that patients with diabetes mellitus have good serum glucose
control before PET scanning.
The cell type of the tumor is not important in the imaging, and lesions larger than 1
cm in diameter can be assessed by PET scanning. Blood pressure control has no
influence on PET imaging, and the study is not potentially nephrotoxic because it
does not use radiocontrast agents.

Pulmonary Medicine and Critical Care:Question 4


The correct answer is B
Educational Objectives
Recognize the importance of occupational history in evaluating patients with
interstitial lung disease.
Critique
This patient has a classic presentation of interstitial lung disease including crackles,
clubbing, and changes in the chest radiograph. Interstitial lung disease can occur in
the setting of exposure to fibrogenic dust leading to pneumoconiosis (most
commonly asbestos and silicosis/coal miners pneumoconiosis). Travel is unlikely to
be associated with increased risk. Although chronic hypersensitivity pneumonitis
might present in this way, it is not associated with classic allergy symptoms. Rare
types of interstitial lung disease are familial, but occupational lung disease is much
more common. The work-up of interstitial lung disease, therefore, requires a
thorough occupational and a vocational history.

Pulmonary Medicine and Critical Care:Question 5


The correct answer is C
Educational Objectives
Diagnose cough secondary to Bordetella pertussis infection.
Critique
The patient has Bordetella pertussis infection (whooping cough); the diagnosis would
be confirmed by enzyme-linked immunosorbent assay. Cough due to B. pertussis
qualifies as a postinfectious cough; it usually lasts 4 to 6 weeks.
Whooping cough is increasing in prevalence in the adult population. Although adults
may not display the characteristic inspiratory whoop at the end of a coughing
paroxysm, they often vomit with coughing and usually have the typical initial
catarrhal stage followed by paroxysmal and convalescent stages. Additional clues to
the diagnosis include leukocytosis and lymphocytosis in an afebrile patient with a
normal chest radiograph.
The three most common causes of chronic cough include postnasal drip syndrome,
asthma, and gastroesophageal reflux disease; CT scan of the sinuses, spirometry,
and 24-hour esophageal pH monitoring may be useful, in diagnosing cough due to
43

these diseases. Sarcoidosis is unlikely because the chest radiograph is normal.

Pulmonary Medicine and Critical Care:Question 6


The correct answer is D
Educational Objectives
Recognize the need for long-term control of mild persistent asthma.
Critique
Asthma symptoms that recur more than 2 days per week and more than twice per
month at night are classified as mild persistent disease. Any patient with asthma of
this degree of severity should receive long-term therapy to treat the ongoing
inflammation, in this case, albuterol and a low-dose inhaled corticosteroid.
All patients should be given a short acting β -agonist as a rescue medication.
Although long acting β-agonists can help with increased control in patients taking
inhaled corticosteroids, asthma at this level has significant inflammation, and long-
acting β-agonists alone do not decrease the inflammation.

Pulmonary Medicine and Critical Care:Question 7


The correct answer is A
Educational Objectives
Treat community-acquired pneumonia by the guidelines of the American Thoracic
Society and the Infectious Diseases Society of America.
Critique
Published guidelines of the American Thoracic Society and the Infectious Diseases
Society of America regarding empiric therapy for treatment of community-acquired
pneumonia recommend either cephalosporin/macrolide or
cephalosporin/fluoroquinolone combinations for patients in the intensive care unit.
Monotherapy with a fluoroquinolone has not been tested in patients with critical
illness. Alternative therapies like those suggested in options C and D might be
selected for patients thought to have specific pathogens, such as Pseudomonas
aeruginosa. Risk factors for P. aeruginosa include bronchiectasis, daily corticosteroid
therapy, recent antibiotic therapy, and malnutrition, none of which applies in this
case.

Pulmonary Medicine and Critical Care:Question 8


The correct answer is D
Educational Objectives
Diagnose and treat a solitary pulmonary nodule on chest radiograph.
Critique
This patient likely has a benign granuloma, which is growing. The characteristics of
the nodule, well-circumscribed, lower lobe location make it less likely to be a
malignancy. In addition, the patient’s young age, the fact that she has never had
cancer, and the fact that she is a nonsmoker also make cancer unlikely. The nodule
should be removed.
Transthoracic needle aspiration is unhelpful because a negative aspirate would not
rule out the clinical suspicion of cancer. A positron emission tomography (PET) scan
would be a reasonable option if the lesion were larger; PET scanning cannot
discriminate well if the lesion is less than 1 cm.
Lobectomy is too extensive if the lesion is not malignant. Removal of the node allows
for minimal resection; if the frozen section at the time shows cancer, then a formal
lobectomy can be accomplished during the same surgery.
44

Pulmonary Medicine and Critical Care:Question 9


The correct answer is D
Educational Objectives
Diagnose the cause of dyspnea.
Critique
Tracheal stenosis can be a complication of prolonged intubation. Although spirometry
suggests moderate obstruction, which could be due to chronic obstructive pulmonary
disease, the patient’s history and the flattening of this inspiratory and expiratory
limb of the flow-volume loop indicating a fixed upper airway obstruction provide a
clue to the diagnosis.
Superimposed congestive heart failure in patients with chronic obstructive pulmonary
disease can cause increased airway resistance mimicking asthma (“cardiac asthma’).
The degree of obstruction and the flattening of the flow-volume loop in this patient,
however, do not support the diagnosis of congestive heart failure alone.
Although there are a limited number of long-term follow-up studies of patients with
acute respiratory distress syndrome, the range of outcomes is wide. Patients may
recover with minimal or no abnormalities by routine lung function testing shortly
after acute lung insult, or they may remain substantially impaired for a year or
longer, if not permanently. Exertional dyspnea is the most commonly reported
symptom, although cough and wheezing also are common. A reduced single-breath
diffusing lung capacity for carbon monoxide is the most common pulmonary function
abnormality. Spirometry and lung volume tests show mixed restrictive-obstructive
abnormalities. Constrictive pericarditis is unlikely, given the presence of the flattened
flow-volume loop and the absence of right heart failure clinically.

Pulmonary Medicine and Critical Care:Question 10


The correct answer is C
Educational Objectives
Diagnose pulmonary embolism.
Critique
The pretest clinical suspicion of pulmonary embolism is high in this patient because
of the recent diagnosis of malignancy and compatible symptoms. The presence of
two anatomically defined, unmatched defects on ventilation/perfusion scanning
indicates a high probability for pulmonary embolism. The probability of venous
thromboembolism exceeds 95% when a high-probability scan is combined with a
high-probability pretest clinical assessment.
Anticoagulation is appropriate for this patient based on the available information.
Further testing is not necessary.
In the absence of hemodynamic compromise thrombolytic therapy is not indicated
because the risks exceed the known benefits of this therapy.

Pulmonary Medicine and Critical Care:Question 11


The correct answer is C
Educational Objectives
Recognize the clinical and radiographic features of asbestosis.
Critique
An important component of evaluating interstitial lung disease is taking a careful
longitudinal occupational and environmental exposure history. This patients lengthy
work as an insulation installer likely exposed him to high levels of asbestos, placing
him at risk for the development of asbestosis. The diagnosis of asbestosis requires
45

clinical evidence of interstitial fibrosis and a reliable history of asbestos exposure,


with a typical latency period from initial exposure to presentation of greater than 20
years. Chest radiograph usually shows small bilateral reticular lower lobe densities.
The presence of calcified pleural plaques is a hallmark of asbestos exposure and is
found in approximately 50% of persons exposed to asbestos.
Eggshell calcification of intrathoracic lymph nodes is seen in as many as 15% of
cases of silicosis, which occurs from inhalation of silica particles (most commonly
quartz). Upper lobe pulmonary nodules are characteristically seen in silicosis and
coal workers pneumoconiosis and are not typical features of asbestosis. Cavitating
pulmonary nodules and masses are radiographic findings of coal workers
pneumoconiosis, not of asbestosis.

Pulmonary Medicine and Critical Care:Question 12


The correct answer is A
Educational Objectives
Recognize indications for methacholine challenge testing in cases of suspected
asthma.
Critique
This patients history is consistent with asthma. Because asthma can be an episodic
disease, the baseline spirometry may be normal. Methacholine challenge testing can
be helpful in this setting. A decrease in the forced expiratory volume in 1 sec (FEV1)
of more than 20% at a dose of less than 4 mg/mL of methacholine (PD20) is
diagnostic of asthma. Intermediate doses of methacholine (<16 mg/mL) that result
in bronchial hyperreactivity are less sensitive for the diagnosis of asthma but may
indicate some degree of bronchial hyperresponsiveness. A drop in PD20 with the
highest dose (16 mg/mL) is consistent with a normal study.
Asthma is an episodic disease that can be triggered by environmental allergens or
stress or can be intrinsic. Some patients with mild intermittent asthma may present
with normal pulmonary function tests despite having symptoms; others may have
atypical symptoms that require testing in order to demonstrate bronchial
hyperreactivity. In patients with moderate disease, static airflow measurement can
be normal between acute episodes. However, these patients may still have bronchial
hyperreactivity in the presence of normal flows.
An adequate bronchoprovocation test requires that the patient have a normal
baseline spirometry. Recent use of inhaled β-agonists can affect response to the test,
as can use of β-blockers or histamine-blocking agents. Patients must have been
removed from these drugs prior to testing. The sensitivity of a positive methacholine
challenge test is in the range of 85%. False-positive results can be seen in patients
with allergic rhinitis, chronic obstructive pulmonary disease, congestive heart failure,
cystic fibrosis, or bronchitis. Contraindications to testing include moderate or severe
airflow obstruction (FEy1 <60% of predicted), pregnancy, recent myocardial
infarction or stroke, uncontrolled hypertension, or inability to perform good-quality
spirometry.

Pulmonary Medicine and Critical Care:Question 13


The correct answer is D
Educational Objectives
Recognize that neck circumference and nocturnal choking are significant predictors of
obstructive sleep apnea.
Critique
This patient constellation of symptoms and physical findings suggest that he has
obstructive sleep apnea (OSA) leading to recurrent collapse of his upper airway at
night and subsequent daytime hypersomnolence. In men, a neck circumference of 43
46

cm (17 in) or more is strongly associated with OSA. In patients who seek evaluation
at a sleep center, habitual snoring, choking arousals, and hypertension are also
predictive of the disorder.
Although excessive daytime sleepiness is common in OSA, it is not very predictive of
the disease because there are other common causes. Moreover, the presence and
severity of daytime sleepiness correlates poorly with the severity of OSA as
measured by the apnea-hypopnea index. Although ethanol ingestion has been shown
to reduce upper airway dilator function and increase the severity of OSA, ethanol use
has a poor predictive value for OSA. Nocturnal sexual dysfunction is common in OSA,
but it is not predictive of the disorder. Morning headaches are thought to occur
secondary to recurrent hypercapnia/hypoxemia with apnea and hypopnea throughout
the night, but the presence of morning headaches is not a sensitive test for OSA.

Pulmonary Medicine and Critical Care:Question 14


The correct answer is D
Educational Objectives
Diagnose distal intestinal obstruction syndrome in a patient with cystic fibrosis.
Critique
Approximately 3% of adults with cystic fibrosis have distal intestinal obstruction
syndrome, which is characterized by recurrent episodes of obstruction of the small
intestine, most commonly at the distal ileus. Obstruction is caused by inspissation of
stool. It is partially prevented by good hydration. Symptomatic obstruction is treated
by oral administration of a high-osmotic enteral solution or, if severe or refractory,
by enema.
Acute recurrent or chronic pancreatitis occurs in patients with cystic fibrosis whose
pancreatic function is sufficient. However, in approximately 85% of patients with
cystic fibrosis, pancreatic function is insufficient; the loss of functioning exocrine
tissue makes it unlikely that these patients would develop pancreatitis. Patients with
cystic fibrosis are not known to be at increased risk for diverticulitis. Chronic
intestinal pseudo-obstruction is an idiopathic disease unrelated to cystic fibrosis that
is associated with diffuse dilatation of the small bowel.

Pulmonary Medicine and Critical Care:Question 15


The correct answer is C
Educational Objectives
Recognize the role of radiographic imaging in the diagnosis of interstitial lung
disease.
Critique
This patients presentation suggests interstitial lung disease with bibasilar crackles in
the absence of cardiac dysfunction, a restrictive pattern on spirometry, and low
diffusing capacity. In this setting, further cardiac evaluation is unlikely to help
confirm the diagnosis. HRCT is more sensitive than plain chest radiography in
showing parenchymal disease. Distinctive radiographic features may aid in predicting
underlying pathologic change especially when changes on HRCT are characteristic of
established fibrosis (such as honeycombing and traction bronchiectasis) in the
absence of features suggesting active inflammation (including ground-glass
infiltrates, air bronchograms, nodules). Although many experts feel that lung biopsy
is required in interstitial lung disease, bronchoscopy biopsy specimens are of
inadequate size to characterize the lung process. The spirometry and physical
examination make pulmonary embolism less likely in this case.
47

Pulmonary Medicine and Critical Care:Question 16


The correct answer is A
Educational Objectives
Recognize the most common pathogens present in airways of patients with
bronchiectasis.
Critique
Bronchiectasis is the cause of this patients recurrent cough and purulent secretions.
The diagnosis is based on the symptoms and the features on the chest radiograph,
as well as the history of rheumatoid arthritis. The diagnosis can be further confirmed
by HRCT of the chest.
The most common organisms infecting the airways of patients with bronchiectasis
are Pseudomonas aeruginosa and Staphylococcus aureus. Testing of the sputum
would be appropriate, but empiric antibiotic treatment directed atP. aeruginosa could
include the fluoroquinolones and aminoglycosides.
Given her history of nonsmoking, it is less likely that she is having an acute
exacerbation of chronic bronchitis, in which the common organisms are Haemophilus
influenzae, Moraxella catarrhalis, and Streptococcus pneumoniae and which might be
treated with a β-lactam/1β-lactamase combination, macrolide, or respiratory
fluoroquinolone. Allergic bronchopulonary aspergillosis (ABPA) is a known cause of
bronchiectasis and might respond to corticosteroids and antifungal therapy, but this
patient does not meet clinical criteria for a diagnosis of ABPA, such as a positive skin
test and an elevated IgE level.

Pulmonary Medicine and Critical Care:Question 17


The correct answer is D
Educational Objectives
Determine the cause of dyspnea.
Critique
Pulmonary function testing in this patient shows mild restriction with a severely
reduced expiratory reserve volume (ERV) and an elevated diffusing lung capacity for
carbon monoxide (DLCO). Although the reduced forced expiratory volume (FEF25%-
75%) might suggest concomitant obstruction, the notable findings are the markedly
reduced ERV and increased DLCO. A reduced ERV out of proportion to other lung
volumes and an elevated DLCO make obesity the most likely cause of this patients
dyspnea.
An elevated DLCO is seen when pulmonary capillary blood volume is increased
(obesity, early left heart failure, left-to-right shunts, exercise, altitude, and the
supine position). It is also seen when there is an increase in erythrocytes
(polycythemia and alveolar hemorrhage).
The lack of a predominant obstructive ventilatory defect makes bronchiolitis
obliterans unlikely. Interstitial lung disease would cause a reduced DLCO. Although
patients with congestive heart failure may have restriction (due to an enlarged
heart) and an elevated DLCO, the normal A-a gradient on the arterial blood gas
measurement and the markedly reduced ERV makes obesity much more likely. The
pleuropulmonary manifestations of rheumatoid arthritis include pleural disease
(pleuritis, pleural effusion, pneumothorax, bronchopleural fistula, and empyema),
rheumatoid nodules (necrobiotic nodules, Caplan’s syndrome, and rheumatoid
nodulosis), interstitial lung disease, airway involvement (airway obstruction,
cricoarytenoid arthritis, bronchiectasis, bronchiolitis obliterans with organizing
48

pneumonia, and bronchiolitis obliterans), and pulmonary vascular disease (vasculitis


and pulmonary hypertension).

Pulmonary Medicine and Critical Care:Question 18


The correct answer is D
Educational Objectives
Initiate supplemental oxygen therapy in a hypoxemic patient with chronic obstructive
pulmonary disease.
Critique
The only therapeutic intervention that has been shown to prolong life in hypoxemic
patients with chronic obstructive pulmonary disease is supplemental oxygen therapy.

Two nonpharmacologic interventions have also been shown to prolong life for
patients with chronic airflow obstruction: smoking cessation and lung-volume-
reduction surgery in carefully selected patients.

Pulmonary Medicine and Critical Care:Question 19


The correct answer is A
Educational Objectives
Recognize left heart dysfunction as a cause of pulmonary hypertension.
Critique
This patient with symptomatic pulmonary hypertension has an elevated pulmonary
capillary wedge pressure (PCWP), indicating left atrial hypertension. Left ventricular
systolic function and left heart valves appear normal on echocardiography, but she
does have left ventricular hypertrophy and a history of hypertension. The most likely
diagnosis is left ventricular diastolic dysfunction secondary to systemic hypertension.

Primary pulmonary hypertension and pulmonary hypertension secondary to chronic,


organized pulmonary emboli do not explain the elevated PCWP. Indeed, the
pulmonary vascular resistance is normal: [(pulmonary artery mean pressure - PCWP)
I cardiac output] * 80 = [(30- 26)/3.1] * 80 = 103 dyne-sec-1-cm-5). Pulmonary
veno-occlusive disease is a rare disorder that causes narrowing and obliteration of
pulmonary veins; furthermore the PCWP can be difficult to obtain and is frequently
normal or decreased in this condition because the inflated catheter balloon stops
downstream blood flow and thus creates a static ‘column’ of blood that reflects
pressure in the left atrium. Constrictive pericarditis generally causes equalization
(within 5 mm Hg) of diastolic and PCWP.

Pulmonary Medicine and Critical Care:Question 20


The correct answer is A
Educational Objectives
Recognize the strengths of MRI as an imaging modality of the chest.
Critique
This patient has a superior sulcus or Pancoast tumor, which can invade local
structures, including the vertebral bodies and vascular structures. Its invasiveness
can complicate if not obviate surgical treatment. MRI has not yet found significant
application to chest disease, but MRI imaging of the tissue planes and vascular
invasion is much superior to that provided by CT scanning. Because of anatomic
relationships, superior sulcus tumors may invade neurologic and vascular structures
locally, and MRI is the imaging modality of choice in this setting.
High-resolution CT scan provides better lung parenchymal imaging but not better
tissue discrimination. Surgery may still be possible in this patient, but the surgeon
49

will first need better anatomic information about possible tumor invasion.
CT scan of the brain should be limited to such patients who have neurologic signs or
symptoms. Bone scan will not help define the anatomic issues raised in this case and
is unlikely to be helpful in the absence of symptoms or laboratory abnormalities
suggesting bone involvement.

Pulmonary Medicine and Critical Care:Question 21


The correct answer is D
Educational Objectives
Manage exercise-induced asthma.
critique
This patient’s history is most consistent with exercise-induced asthma with no
symptoms between periods of exercise. The most appropriate management at this
point would be inhaled β-agonists prior to exercise.
Theophylline and ipratropium bromide have no role in this setting, and
corticosteroids and leukotriene inhibitors are best used in the background setting of
persistent asthma.

Pulmonary Medicine and Critical Care:Question 22


The correct answer is D
Educational Objectives
Recognize cough related to the use of an angiotensin-converting enzyme inhibitor.
Critique
The history of newly diagnosed hypertension should prompt a review of the patients
medications for an angiotensin-converting enzyme (ACE) inhibitor. The frequency of
cough associated with ACE inhibitors has been reported to vary widely, from 0.2% to
33%. Cough has been reported to appear within a few hours of taking a first dose in
many patients, but it may not become apparent for weeks or even months.
The pathogenesis of ACE inhibitor-induced cough is unknown. It is likely that the
cough is related to an accumulation of proinflammatory mediators, bradykinin,
substance P, thromboxanes, and prostaglandins. There is no laboratory test that will
predict who will experience cough related to ACE inhibitor therapy. The diagnosis is
confirmed when cough disappears after the drug is discontinued.
There is nothing in the patients history to suggest that he has postnasal drip
syndrome, which would be ruled out by CT scan of the sinuses, or asthma, which
would be ruled out by methacholine challenge testing (its negative predictive value
of nearly 100%, in the context of cough, makes it is extremely useful in ruling out
asthma). Although the patient has a history of gastroesophageal reflux disease, the
disease is well controlled and not likely to be the cause of his cough.

Pulmonary Medicine and Critical Care:Question 23


The correct answer is A
Educational Objectives
Recognize when circulating compounds may alter the results of pulse oximetry.
Critique
The hypoxemia in this case is most likely related to smoke inhalation. Inhaled carbon
monoxide would result in carboxyhemoglobin elevation. Carboxyhemoglobin absorbs
light at the same wavelength as oxyhemoglobin; therefore, pulse oximetry cannot
distinguish carboxyhemoglobin from oxyhemoglobin and will provide misleading
results regarding the actual oxygen content of hemoglobin.
Methemoglobinemia does not alter pulse oximetry, although the treatment for
methemoglobinemia may result in falsely depressed oximetry readings. The presence
50

of elevated levels of carbon dioxide or the presence of anemia would not affect
oximetry readings. Smoke inhalation is responsible for most accidental cases of
carbon monoxide poisoning. Carbon monoxide diffuses rapidly across the pulmonary
capillary membrane and is able to bind the iron moiety of heme (and other
porphyrins) with approximately 240 times the affinity of oxygen. The result is
deformation and leftward shift of the oxyhemoglobin dissociation curve and
impairment of tissue oxygen delivery.
Mild or moderate carbon monoxide intoxication can cause constitutional symptoms
including headache (the most common presenting symptom), malaise, nausea, and
dizziness. Severe toxicity can produce seizures, syncope, or coma, accompanied by
myocardial ischemia, ventricular arrhythmias, pulmonary edema, and profound lactic
acidosis.

Pulmonary Medicine and Critical Care:Question 24


The correct answer is B
Educational Objectives
Recognize contraindications to lung-volume-reduction surgery.
Critique
Patients with a forced expiratory volume in 1 sec (FEV1) <20% of predicted and
either a diffusing lung capacity for carbon monoxide (DLCO) <20% of predicted or
homogeneously distributed emphysema had a 16% 30-day mortality rate, compared
with a medically treated control group in the National Emphysema Treatment Trial.
Patients with these combinations of characteristics are not considered eligible for
lung-volume-reduction surgery.

Pulmonary Medicine and Critical Care:Question 25


The correct answer is B
Educational Objectives
Diagnose latent tuberculosis.
Critique
Patients with latent tuberculous infection are identified by tuberculin skin testing.
Those with a positive skin test should undergo chest radiography looking for
evidence of active tuberculosis. For those with a normal chest radiograph, sputum
examination is not necessary. The magnitude of the skin test reaction is not
indicative of whether there is active tuberculosis. It is likely that he received a bacille
Calmette-Guerin (BCG) vaccination as a child, which brings about a 60% to 80%
reduction in the incidence of tuberculosis. The effect of BCG vaccination on reactivity
to tuberculin skin testing is dependent upon the age of the person at vaccination and
the length of the interval between vaccination and skin testing. A false-positive
reaction after previous BCG vaccination occurs in less than 10% of those vaccinated
before the age of 1 year, and as many as 25% of persons vaccinated after the age of
5 years. It is unlikely that the reaction will exceed 20 mm induration.

Pulmonary Medicine and Critical Care:Question 26


The correct answer is D
Educational Objectives
Recognize and treat poorly controlled asthma.
Critique
This patients symptoms are classified as moderate persistent asthma. Adding long-
acting β-agonists to an adequate inhaled corticosteroid dose is the recommendation
of the National Asthma Education and Prevention Program Expert Panel. The
improvement in control afforded by adding the long-acting β-agonist appears to be
51

greater than that of doubling the dosage of inhaled corticosteroids. Antibiotics have
not been shown to improve asthma control. Nebulizers can increase the dose of β-
agonist, usually equivalent to four to six puffs of a metered-dose inhaler, but
nebulizers do not lead to increased asthma control. Adding ipratropium bromide is
recommended in an acute exacerbation in an emergency situation but has not been
shown to add to the long-term control of asthma. Adding a leukotriene receptor
antagonist is an alternative approach but not the preferred approach in this setting.

Pulmonary Medicine and Critical Care:Question 27


The correct answer is B
Educational Objectives
Recognize the limitations of CT scanning in the staging of lung cancer.
Critique
The most important element in staging this patients lung cancer is determining
whether the mediastinal nodes are involved in the lung cancer. Although the
enlarged nodes at 1 .5 cm are suspicious, the CT scan has neither the sensitivity nor
specificity to allow accurate staging of a patient; specificity ranges between 79% and
86%. Appropriate staging requires that the patient undergo nodal sampling by
mediastinoscopy or median sternotomy. Resection of the mass, chemotherapy,
radiation therapy, and palliative care are not appropriate options because they are
treatments that should be based on staging information, which CT scanning cannot
provide.

Pulmonary Medicine and Critical Care:Question 28


The correct answer is A
Educational Objectives
Choose the appropriate test to evaluate a patient with chronic obstructive pulmonary
disease.
Critique
The patients history is consistent with chronic obstructive pulmonary disease (COPD)
and does not suggest coexistent cardiac disease. Because of this patient’s early work
history, which would have exposed him to asbestos, he is at risk for combined COPD
and restrictive (asbestosis) lung disease. The lung volumes and diffusing capacity
would help more fully evaluate his potential diagnosis.
The spirometric flows reveal a decrease in forced expiratory volume in 1 sec (FEV1)
and forced vital capacity (FVC). Although this pattern can be seen in obstructive
airways disease, it can also represent either restrictive disease or a mixed process.
In COPD, which encompasses both emphysema and chronic bronchitis, the FVC is
often well preserved early in the disease, although the FEV drops, resulting in a
decrease in the FEV1/FVC ratio. In some patients, however, the forced expiratory
maneuver may lead to decreased FVC, although the ratio of FEV1 to FVC is low.

Pulmonary Medicine and Critical Care:Question 29


The correct answer is D
Educational Objectives
Diagnose the cause of dyspnea.
Critique
The patient has myasthenia gravis (electromyography would show electrical
decrements on repetitive stimulation). Reduced maximum voluntary ventilation and
maximum inspiratory and expiratory mouth pressures suggest muscle weakness. A
history of symptoms that worsen during the course of the day also suggests this
diagnosis. Corticosteroids alleviate myasthenic weakness in many patients.
52

Spirometry and lung volumes reveal mild restriction. Chronic thromboembolic


disease and idiopathic pulmonary fibrosis are unlikely in the setting of a normal
diffusion capacity. Chronic obstructive pulmonary disease is unlikely in someone who
has never smoked.

Pulmonary Medicine and Critical Care:Question 30


The correct answer is D
Educational Objectives
Diagnose and treat high-altitude pulmonary edema.
Critique
This patient has high-altitude pulmonary edema (HAPE), a life-threatening form of
altitude sickness that occurs in unacclimatized persons after ascent to high altitudes
(usually higher than 2500 m [8,000 feet]). Supplemental oxygen and descent to a
lower altitude (which also increases arterial oxygen content) are the treatment of
choice for HAPE. Patients with HAPE respond dramatically to treatment with
supplemental oxygen, which reduces pulmonary artery pressure by as much as 50%.
Use of a portable hyperbaric chamber, which simulates descent, is also helpful.
If supplemental oxygen and descent are not available treatment options, then
administration of oral nifedipine is recommended. Although nifedipine can cause
systemic hypotension, the potential life-saving effect of lowering pulmonary artery
pressure is felt to outweigh the risk. Dexamethasone is indicated only if there is
evidence of concomitant cerebral edema.
HAPE is a form of noncardiogenic pulmonary edema associated with elevated
pressures in the pulmonary microvasculature. The pathophysiology centers on an
exaggerated pulmonary vasoconstrictive response to hypoxia, but impaired clearance
of fluid from the alveolar space may also be important. Early symptoms include
decreased performance, dyspnea, and dry cough. Pink, frothy sputum and frank
respiratory distress are late symptoms in the illness. Concomitant high-altitude
cerebral edema may occur and is a common finding at autopsy in persons who die of
HAPE.
Recently published studies suggest a role for inhaled β-agonists (which increase
clearance of alveolar fluid) in the prevention and treatment of HAPE. Risk factors for
the development of HAPE include the rate of ascent, absolute altitude reached,
exertion, cold, and individual susceptibility. Persons who develop HAPE have a
significantly increased risk of recurrence on future climbs. Slow, gradual ascent,
avoidance of overexertion, and nifedipine prophylaxis are recommended to reduce
the risk of future episodes of HAPE.

Pulmonary Medicine and Critical Care:Question 31


The correct answer is D
Educational Objectives
Recognize the manifestations of lung disease associated with rheumatoid arthritis.
Critique
Pulmonary manifestations in rheumatoid arthritis range from common (such as
pleural effusions) to rare (such as bronchiolitis obliterans). Interstitial lung disease,
commonly referred to as rheumatoid arthritis-associated interstitial lung disease (RA-
ILD), may be the most common form of pulmonary involvement. Because RA-ILD
often persists or progresses despite aggressive therapy for rheumatoid arthritis, it
may be difficult to differentiate progression of underlying disease from toxicity of a
53

therapeutic agent (methotrexate, for example).


Rheumatoid pleuritis may present with fever and pleuritic chest pain or may be
asymptomatic. Effusions are unilateral in more than 75% of cases and as many as
one third of cases may be associated with other rheumatoid lung diseases. The
pleural fluid is characteristically exudative with a very low glucose level (<40 mg/dL
in 70%). There is no need to evacuate these effusions if cultures are negative.

Pulmonary Medicine and Critical Care:Question 32


The correct answer is D
Educational Objectives
Manage venous thromboembolism.
Critique
The history of venous thromboembolism (VTE) combined with a history of
malignancy and compatible presenting symptoms puts this patient at considerable
risk for acute pulmonary embolism based on initial evaluation. It is appropriate to
measure theD-dimer level and obtain Doppler ultrasound imaging of the lower
extremities if it is elevated.
When the pretest probability of VTE is intermediate or high, a low-probability
ventilation-perfusion (V/Q) scan does not reasonably exclude the possibility of
pulmonary embolism. Indeed, the probability of VTE exceeds 15% when a low-
probability scan is combined with an intermediate pretest probability of disease.
Measurement of arterial blood gases is not particularly helpful for the diagnosis of
pulmonary embolism, and there is little reason to expect a clinically important
abnormality of pH or blood gases in this instance. Although a normal D-dimer in
combination with either a negative lower extremity ultrasound or nondiagnostic V/Q
scan can be used to exclude VTE when there is a low clinical suspicion, this patients
history suggests at least an intermediate clinical probability of VTE. Many conditions
in addition to VTE are associated with elevated D-dimer levels. Consequently,
anticoagulation should not be initiated or continued on the basis of an elevated D-
dimer level alone. Instead, an elevated level combined with a low- or intermediate-
probability V/Q scan should prompt a second imaging study, such as Doppler
ultrasound imaging of the lower extremities. Current evidence supports
anticoagulation therapy for 3 to 6 months following a first episode of VTE
complicating trauma. Embolic events do sometimes recur after a full course of
therapy.

Pulmonary Medicine and Critical Care:Question 33


The correct answer is C
Educational Objectives
Recognize risk factors for specific pathogens causing community-acquired
pneumonia.
Critique
This patient has risk factors that suggest additional coverage is necessary for drug-
resistant Streptococcus pneumoniae and enteric gram-negative organisms. She does
not need additional coverage for anaerobes or Pseudomonas aeruginosa.
Treatment guidelines for community-acquired pneumonia are based on the likelihood
of specific pathogens. In general, there should be coverage for the most common
organisms such as Streptococcus pneumoniae, Haemophilus influenzae, and
Mycoplasma pneumoniae. For some patients with specific risk factors, there should
be consideration given for other specific pathogens.
Risk factors for drug-resistant S. pneumoniae include age greater than 65 years, use
54

of β-lactam antibiotics within the past 3 months, immunosuppression, multiple


medical comorbidities, and regular exposure to a child in day care. Risk factors for
anaerobic infections include significant witnessed or suspected aspiration. Risk
factors for enteric gram-negative organisms include all of those for S. pneumoniae
listed above, as well as regular alcohol consumption and residence in a nursing
home. Risk factors for P. aeruginosa include bronchiectasis, daily corticosteroid
therapy, recent broad-spectrum antibiotic therapy, and severe malnutrition.

Pulmonary Medicine and Critical Care:Question 34


The correct answer is B
Educational Objectives
Manage hemoptysis in a patient with cystic fibrosis.
Critique
The appropriate next step should be urgent bronchoscopy. Even if the bleeding has
stopped, the bronchscopist can often identify at least the lobe of the lung that is
bleeding. The ability to identify the bleeding site decreases with the duration of time
since the last episode. Therefore, timely bronchoscopy is required.
This patients significant airway bleeding is associated with bronchiectasis. Sentinel
bleeding can often presage life-threatening massive hemoptysis. After ensuring
clinical stability, it is necessary to identify the site of bleeding because future
therapeutic steps, such as patient positioning (bleeding lung down), bronchoscopic
balloon tamponade, selective intubation, embolization, and surgery, require some
knowledge about the bleeding site.
This patient is unlikely to have an elevated fibrinolytic state and will not benefit from
&epsilon;-aminocaproic acid. Although appropriate antibiotic therapy may help
manage mild hemoptysis, this patients degree of bleeding requires a more
aggressive approach. Both bronchial artery embolization and selective airway
intubation may be helpful, but they cannot be effective until the site of bleeding has
been determined.

Pulmonary Medicine and Critical Care:Question 35


The correct answer is B
Educational Objectives
Diagnose vocal cord dysfunction.
Critique
Vocal cord dysfunction can mimic asthma and present as asthma resistant to
treatment. Diagnosis is difficult. Indications of vocal cord dysfunction include a
history of difficult-to-treat asthma with normal findings on spirometry; minimal
night-time symptoms; a flow-volume loop with flattening of the inspiratory loop,
suggesting a variable extrathoracic obstruction; and a patients feeling that it is
harder to breathe in than out. Diagnosis is confirmed by showing adduction of the
cords throughout the respiratory cycle. Vocal and speech therapy are used in
treatment.
Chronic obstructive pulmonary disease (COPD) is usually unresponsive to
corticosteroid therapy and rarely causes nighttime symptoms. A patient with COPD
would not have normal spirometry results.
Patients with corticosteroid-resistant asthma or asthma with bronchopulmonary
aspergillosis usually respond to corticosteroid therapy but require ever-increasing
dosages. Furthermore, spirometry would be abnormal at the time of attacks. A
patient with corticosteroid-resistant asthma would have nighttime symptoms.

Pulmonary Medicine and Critical Care:Question 36


The correct answer is D
55

Educational Objectives
Treat cough that is due to gastroesophageal reflux disease.
Critique
The patient has cough secondary to gastroesophageal reflux disease (GERD), which
is among the three most common causes of chronic cough in all age groups. A
diagnosis of GERD should be considered when patients experience heartburn, a sour
taste, regurgitation, or globus hystericus (frequent throat clearing secondary to a
sensation that something is stuck in the back of the throat). The most sensitive and
specific test for GERD is 24-hour esophageal pH monitoring. The objective of therapy
is to decrease the frequency and duration of reflux events and decrease the irritative
nature of gastric secretions. Therapy includes conservative dietary and lifestyle
measures, prokinetic agents, and acid-suppressing drugs. It may take as long as 3
months for symptoms to resolve.
Because cough may be simultaneously caused by more than one condition as much
as 93% of the time, therapy should not be stopped if partially successful; instead,
other therapies should be added sequentially. Patients who do not respond to
medical therapy may benefit from antireflux surgery including fundoplication, but it is
too early to consider surgery for this patient.

Pulmonary Medicine and Critical Care:Question 37


The correct answer is D
Educational Objectives
Recognize the importance of smoking cessation in patients with chronic obstructive
pulmonary disease.
Critique
This patient has chronic stable chronic obstructive pulmonary disease (COPD).
Medical management at this time appears adequate. She does not exhibit
oxyhemoglobin desaturation with exertion and therefore does not require
supplemental oxygen therapy. Her physical examination and chest radiograph
findings are consistent with COPD. She does not have a tension pneumothorax. The
best management strategy at this time is to encourage smoking cessation and to
discuss options to help her stop smoking.

Pulmonary Medicine and Critical Care:Question 38


The correct answer is B
Educational Objectives
Recognize the clinical importance of a transudative pleural effusion.
Critique
The finding of a transudative pleural effusion limits the differential diagnosis to
diseases that are usually apparent from the patients clinical presentation, physical
findings, and laboratory evaluation. The most common causes of transudative
effusions in order of decreased frequency include congestive heart failure, hepatic
hydrothorax, nephrotic syndrome, other causes of low-albumin states, and
atelectasis. The finding of a right-sided transudative effusion in the setting of known
cirrhosis with clinical ascites is presumptive evidence of hepatic hydrothorax. If
simultaneous paracentesis and thoracentesis are accomplished, the clinician should
find that the protein and lactate dehydrogenase values are similar with minimally
higher levels found in pleural fluid. The diagnosis of hepatic hydrothorax can be
confirmed when radionuclide uptake in the right pleural space occurs 1 to 2 hours
after it is injected into the ascitic fluid.
Patients with pleural effusions from congestive heart failure mostly have orthopnea
and paroxysmal nocturnal dyspnea; fine basilar crackles and an S3 at the apex; and
cardiomegaly, bilateral pleural effusions (right >left), and evidence of pulmonary
56

edema on chest radiograph. Approximately 20% of patients with nephrotic syndrome


have small, bilateral transudative pleural effusions. Urinalysis was normal in this
patient, with no evidence of a protein-losing enteropathy. For a patient to develop
pleural effusions from a low albumin state, the serum albumin is generally <1.8
mg/d L. Patients with lung cancer have an exudative pleural effusion and usually
have evidence of a primary lesion on chest radiograph. Ipsilateral shift toward the
side of the effusion is commonly noted.

Pulmonary Medicine and Critical Care:Question 39


The correct answer is A
Educational Objectives
Diagnose obstructive sleep apnea.
Critique
The polysomnogram shows a typical obstructive apneic event. The ventilation
channel shows no airflow (apnea) until a sudden burst of ventilation occurs after the
man arouses and opens his upper airway. The electromyogram, measured at the
chin, shows a burst of lower facial muscle activity at this same instant. The thoracic
strain gauge indicates that ventilatory effort continues through the period of apnea,
then increases after arousal.
In central sleep apnea, the thoracic strain gauge shows no activity during apnea.
Arterial oxygen saturation drifts downward during the period of apnea, then
improves rapidly after ventilation is restored. The electro-oculogram is used to detect
the rapid eye movements that characterize rapid-eye-movement sleep. This patient
may have any of the other sleep disorders listed in addition to sleep apnea, but there
is no evidence for a second sleep disorder in this one brief polysomnography
segment.

Pulmonary Medicine and Critical Care:Question 40


The correct answer is B
Educational Objectives
Recognize the limitations of positron emission tomography (PET) scanning.
Critique
The patient has a known central lung cancer with a postobstructive pneumonia. The
diagnostic tests performed thus far confirm the tumor. Immediate mediastinoscopy
is warrranted.
The positron emission tomography (PET) scan shows uptake in the tumor site and
the mediastinal lymph nodes. The uptake in the mediastinal lymph nodes could be a
false-positive result. PET scanning has specificity in the mediastinum of only about
80%. A false-positive PET scan could occur with infection, which is likely coexisting in
this patient. Surgical resection is not indicated in this patient yet; it is not the correct
treatment option if the patient has tumor deposits in the mediastinum. Radiation
therapy alone is not the correct treatment for stage 3A lung cancer.
If the mediastinal lymph nodes are negative at mediastinoscopy, then surgical
resection is the treatment of choice. If they are positive, the treatment of choice is
chemotherapy and radiation therapy.

Pulmonary Medicine and Critical Care:Question 41


The correct answer is B
Educational Objectives
Recognize the role of corticosteroid therapy in pulmonary sarcoidosis.
Critique
Sarcoidosis is an idiopathic systemic illness characterized by tissue infiltration with
57

well-formed noncaseating granulomata. More than 90% of patients have pulmonary


involvement manifesting as hilar and mediastinal lymphadenopathy, with or without
interstitial lung disease. Many patients are asymptomatic and the disease is
identified on routine chest radiographs. Treatment for sarcoidosis remains
controversial and is generally reserved for those with disabling symptoms, evidence
for radiographic or physiologic progressive lung disease, or symptomatic
extrapulmonary disease. The cornerstone of therapy is corticosteroids for 6 to 12
months. Although such treatment reduces symptoms and manifests improvement on
chest radiography and pulmonary function tests, it is of unproven benefit in altering
long-term outcome. Most patients experience a durable spontaneous remission.
Patients with radiographic stage 1 disease (hilar and/or mediastinal
lymphadenopathy without infiltrates) and no systemic symptoms to suggest an
alternative disease have a spontaneous remission rate exceeding 70%. Patients with
anterior uveitis are best treated first with topical corticosteroids rather than systemic
corticosteroids. Lofgrens syndrome is defined by fever, erythema nodosum, and
bilateral hilar lymphadenopathy. Patients with this syndrome also have a benign
course and may be managed with observation alone.

Pulmonary Medicine and Critical Care:Question 42


The correct answer is C
Educational Objectives
Recognize the clinical indications for the use of long-term supplemental oxygen.

Pulmonary Medicine and Critical Care:Question 43


The correct answer is D
Educational Objectives
Recognize the dangers of inhaled corticosteroid therapy and the appropriate step-
down therapy.
Critique
The patient is doing well, but is taking significant amounts of inhaled corticosteroid.
Maintaining the patient on his present therapy exposes him to the risk of side
effects. A step-wise decrease in the dosage of inhaled corticosteroids affords the best
chance of maintaining control while minimizing side effects. The best response in
most patients with severe asthma is effected by a combination of inhaled
corticosteroids and a long-acting β-agonist, which improve lung function and
symptoms, along with a short-acting β-agonist as a rescue medication. Maintaining
the patient on a low-dose inhaled corticosteroid alone may lead to renewed
exacerbation of symptoms.

Pulmonary Medicine and Critical Care:Question 44


The correct answer is A
Educational Objectives
Recognize the importance of pulmonary rehabilitation.
Critique
Pulmonary rehabilitation is a critical adjunct to pharmacologic therapy in patients
with chronic obstructive pulmonary disease. Severe dyspnea leads to a sedentary
lifestyle and subsequent deconditioning. Pulmonary rehabilitation increases strength,
quality of life, sense of well-being, and exercise tolerance. There is no role for high-
dose corticosteroids in the treatment of this patient. She does not appear to be
depressed; therefore prescribing antidepressant medication would be inappropriate
at this time.
58

Pulmonary Medicine and Critical Care:Question 45


The correct answer is B
Educational Objectives
Recognize the clinical presentation and treatment of cyanide poisoning.
Critique
This patient is suffering from smoke inhalation with cyanide poisoning. Clinical clues
include the findings of coma, hypotension, cardiac irritability, and profound anion gap
metabolic acidosis (due to lactic acidosis) in the setting of adequate volume
resuscitation and oxygen administration. Combustion of natural fabrics (such as wool
and silk) is known to produce hydrogen cyanide gas. The patient should be
empirically treated for cyanide poisoning with intravenous sodium thiosulfate.
Treatment with hyperbaric oxygen is indicated on the basis of the patients comatose
state with an elevated carboxyhemoglobin level.
Cyanide causes tissue hypoxia by binding to mitochondrial cytochrome oxidase and
inhibiting cellular respiration. Sodium thiosulfate converts cyanide to the much less
toxic thiocyanate ion, which is then excreted in the urine. The full currently available
cyanide antidote kit in the United States contains inhaled amyl nitrite and
intravenous sodium nitrite, which oxidize hemoglobin to methemoglobin. Cyanide
has a greater affinity for the ferric ion of methemoglobin than for the iron molecule
of cytochrome oxidase. Because methemoglobin does not carry oxygen, nitrites
should be avoided or used cautiously in the setting of significant
carboxyhemoglobinemia.
Carbon monoxide poisoning can also cause coma and hypotension, but the fact that
this patient does not respond to treatment with 100% oxygen and has only
moderately elevated levels of carboxyhemoglobin mandates consideration of other
causes of coma. The finding of unexplained lactic acidosis refractory to fluid
resuscitation and administration of 100% oxygen in the setting of smoke inhalation
should prompt consideration of cyanide poisoning.
Evidence of an anion-gap metabolic acidosis is probably related to lactic acidosis, but
bicarbonate therapy in this setting is unlikely to be helpful and may lead to a
metabolic alkalosis once the underlying problem is treated.
Although a CT scan of the brain and cervical spine series might be useful in
evaluating unexplained coma in a patient with smoke inhalation (the patient could
have been injured in an explosion), these tests would not help account for the
patients profound metabolic acidosis.
Methylene blue is the antidote for methemoglobinemia and has no role in the
treatment of this patient. The finding of an elevated osmolal gap in the setting of an
anion-gap metabolic acidosis is a clue to poisoning by ethylene glycol or methanol,
neither of which pertains in this case.

Pulmonary Medicine and Critical Care:Question 46


The correct answer is D
Educational Objectives
Diagnose cough due to cough-variant asthma.
Critique
The patient has cough-variant asthma. The three most common causes of chronic
cough include postnasal drip syndrome, asthma, and gastroesophageal reflux disease
(GERD). Bronchoscopy and chest CT play no role in diagnosing cough due to these
three causes.
The diagnosis of cough-variant asthma is suggested by the presence of airway
hyperresponsiveness and confirmed when cough goes away with asthma
medications. Cough-variant asthma (cough is the predominant symptom) occurs in
up to 57% of asthmatics. Because methacholine challenge testing has a negative
59

predictive value of 100%, in the context of cough, this test is extremely useful in
ruling out asthma.
Most patients with postnasal drip syndrome will have symptoms or evidence of one
or more of the following: postnasal drainage, throat clearing, nasal discharge,
cobblestone appearance of the oropharyngeal mucosa, or mucus in the oropharynx.
The patient had none of these symptoms or findings and empiric therapy for
postnasal drip syndrome failed to alleviate his symptoms.
There is nothing about the character and timing of chronic cough due to GERD that
distinguishes it from other conditions; in addition, it can be “silent” from a
gastrointestinal standpoint up to 75% of the time. However, the patient failed 3
months of empiric therapy for GERD. It is logical to rule out cough-variant asthma,
especially with symptoms coinciding with the addition of a β-blocker, before pursuing
24-hour esophageal pH testing.

Pulmonary Medicine and Critical Care:Question 47


The correct answer is C
Educational Objectives
Recognize the importance of nutritional management in severe chronic obstructive
pulmonary disease.
Critique
Weight loss is a common complication of severe chronic obstructive pulmonary
disease. The cause can be multifactorial and include poor caloric intake because of
dyspnea and increased work of breathing. The most important element appears to be
the catabolic state, which is due in part to increased cytokine production in the
lungs. Simple nutritional management is ineffective at reversing this process unless
it is accompanied by an anabolic stimulus such as exercise. Pulmonary rehabilitation
with exercise and nutritional counseling would be the best alternative for this patient.

Some data suggest that anabolic steroids may be beneficial, but the absence of
comprehensive and long-term studies as well as the toxicity of these drugs make
such an approach investigational at this time. The patient does not meet the criteria
for oxygen supplementation, and there are no data to suggest that either oxygen or
inhaled corticosteroids would help reverse his weight loss.

Pulmonary Medicine and Critical Care:Question 48


The correct answer is D
Educational Objectives
Diagnose Legionella-caused community-acquired pneumonia.
Critique
Legionella pneumophila should be considered a potential causative organism in all
patients with community-acquired pneumonia and appropriate risk factors and
presenting symptoms. The most common risk factors include cigarette smoking;
underlying diseases such as renal failure, cancer, diabetes mellitus, or chronic
obstructive pulmonary disease; and immunosuppression as a result of
chemotherapy. Other factors include corticosteroid therapy, alcoholism, and being
middle-aged or older. Also at risk are patients on long-term ventilation.
Diagnostic testing also has a very low sensitivity. Though highly specific, direct
fluorescent antibody testing for Legionella has a sensitivity of <30%. The urinary
antigen test has a greater sensitivity (>80%) but will detect only serotype 1,
accounting for only 70% of Legionella infections. Serologic testing may be the most
sensitive but results will not be available for weeks following presentation. Thus, the
occurrence of Legionella cannot be known at the time of diagnosis, and empiric
therapy is appropriate for hospitalized patients with community-acquired pneumonia.
60

The signs and symptoms of Legionella pneumonia are nonspecific and can range in
severity from mild illness (walking pneumonia) to fatal multilobar pneumonia.
Patients may have high, unremitting fever and cough with little sputum production;
the time between exposure and onset of illness is approximately 2 to 10 days.
Extrapulmonary symptoms include gastrointestinal disturbances and involvement of
the central nervous system, liver, and kidney; pericarditis and endocarditis are rare.
The presence of such abnormalities as diarrhea and elevated serum creatine kinase
levels has a sensitivity of about 30%.

Pulmonary Medicine and Critical Care:Question 49


The correct answer is C
Educational Objectives
Manage cocaine intoxication.
Critique
This patients sympathomimetic syndrome is consistent with cocaine intoxication.
Clinical findings often include tachycardia, hypertension, hyperthermia, mydriasis,
agitation, and psychosis. Other possible complications of cocaine abuse may be
difficult to evaluate in the agitated patient. The initial treatment in this patient should
include sedation with lorazepam, administered intravenously or intramuscularly.
Control of agitation usually brings about a decrease in heart rate, blood pressure,
and temperature.
Intravenous fluids should be administered to establish adequate urine output for
possible rhabdomyolysis, and an electrocardiogram should be obtained to assess for
myocardial ischemia. Laboratory studies should include measurement of serum
electrolytes and serum creatine kinase and evaluation of liver function. A CT scan of
the brain may be indicated to rule out intracranial injury.
Labetalol might be an appropriate antihypertensive agent due to the combined ct-
and 3-blocking effects of cocaine in a patient who has sustained severe
hypertension. Hypertension is rarely severe in cocaine abuse, however, and usually
responds to control of agitation. Other β-blockers are not recommended because of
the potential concern for worsening of vasoconstriction due to unopposed ct-blocking
effects.
Haloperidol is not initially indicated for control of agitation in a patient who abuses
cocaine. Haloperidol has the potential to lower the seizure threshold and would not
be an initial treatment in this patient who has already had a seizure. After agitation
is controlled, haloperidol can be considered if the patient manifests psychotic
features.
Drug-induced seizures, which are usually self-limited, do not respond well to
phenytoin. The treatment of choice for drug-induced seizures is benzodiazepines.
Although the patient is febrile, acetaminophen would not be considered until
agitation is controlled.

Pulmonary Medicine and Critical Care:Question 50


The correct answer is D
Educational Objectives
Recognize when supplemental oxygen is necessary during airline travel.
Critique
One researcher was able to predict the expected PaO2 by the formula (0.238* PaO2
at sea level) + (20.098*FEV1/FVC) + 22.258. This would yield a predicted PaO2 of
48mm Hg in this patient. Patients with a predicted PaO2 of less than 55 mm Hg meet
61

criteria for supplemental oxygen. An arrangement for supplemental oxygen while


traveling on a commercial airline is individualized and will require significant planning
by both the physician and the patient.
Commercial airlines that travel above 10,000 feet are required to pressurize their
cabins, but each airline pressurizes the cabin to a slightly different barometric
pressure. For purposes of calculating the PaO2 an altitude of 8000 feet is used as the
calculated airplane cabin pressure. Carbonic anhydrase inhibitors and
dexamethasone are used for high-altitude pulmonary edema (HAPE), but HAPE does
not occur below 8000 feet. Pressurized cabins prevent high-altitude pulmonary
edema.

Pulmonary Medicine and Critical Care:Question 51


The correct answer is C
Educational Objectives
Recognize barriers to the control of asthma.
Critique
Patient education is a key component in asthma care. Studies have shown that
patient education by the physician decreases the number of visits to the emergency
department and improves asthma control. Poor technique in the use of a metered-
dose inhaler (MDI) is a major reason patients do not respond well to medications.
Use of a spacer with good technique will lead to improved delivery of the medication
and decreased side effects.
Adding a leukotriene inhibitor would be appropriate if the patient is effectively using
the current medications; it should not be added, however, until it is certain that the
patient is using good MDI technique.
Administration of β-agonists by mouth or by nebulizer is done at the expense of
increased side effects. Neither method is as effective for rescue medication purposes
because oral medications have a delayed onset and nebulizers are not always be
available for patients when they need them.
Oral prednisone would be more appropriate for an exacerbation or poorly controlled
severe persistent asthma. It would improve asthma control, but without proper
education in the use of the MDI, symptoms would mostly likely return when the
corticosteroids are tapered. Furthermore, oral corticosteroids have increased side
effects.

Pulmonary Medicine and Critical Care:Question 52


The correct answer is D
Educational Objectives
Recognize the ‘shrinking lung syndrome of systemic lupus erythematosus.
Critique
Many pulmonary syndromes have been described in patients with systemic lupus
erythematosus, including recurrent pleuritis with or without effusions, alveolar
hemorrhage, acute lupus pneumonitis, chronic interstitial disease, and the ‘shrinking
lung syndrome.” In this patient with exertional dyspnea, physiologic testing reveals a
restrictive ventilatory defect (low FEV1 and FVC, high FEV1/FVC, and low total lung
capacity). Any of the disorders described above could present with these findings.
The low lung volumes with normal parenchyma on CT scan, the normal DLCO, and
the preserved residual volume in the face of a reduced total lung capacity favor
restrictive physiology due to respiratory muscle weakness.
This “shrinking lung syndrome” associated with systemic lupus erythematosus (SLE)
appears to be due to inspiratory respiratory muscle dysfunction. Although some
investigators have confirmed diaphragmatic weakness or abnormal phrenic nerve
recruitment, others have suggested that the defect lies in abnormalities in chest wall
62

compliance. Whatever the cause, the syndrome is associated with low lung volumes
in the absence of parenchymal or pleural fibrotic disease. Although the disorder has
been found to respond to corticosteroid therapy, it may relentlessly progress to
hypercapnic respiratory failure.
Alveolar hemorrhage is unlikely because there is no history of hemoptysis or cough,
the DLCO is normal, and there are no infiltrates on imaging. Acute lupus pneumonitis
is a fulminant disorder presenting with fever, hypoxemia, and pulmonary infiltrates
and is not associated with small-airways disease. Chronic interstitial lung disease is
uncommon in SLE and may develop independently of acute pneumonitis or as its
sequela. It is more common in patients with SLE whose symptoms overlap those of
systemic sclerosis. As in any form of fibrosing lung disease, CT scan would likely
show reticular infiltrates. A reduction in both total lung capacity and residual volume
due to increased elastic recoil of the scarred pulmonary parenchyma would also be
likely.

Pulmonary Medicine and Critical Care:Question 53


The correct answer is C
Educational Objectives
Determine the cause of dyspnea.
Critique
The patients physical examination is suggestive of pulmonary hypertension (tricuspid
regurgitation, accentuated pulmonic heart sound) with right-sided heart failure
(jugular venous distention, ascites, and peripheral edema). Spirometry and lung
volumes show mild restriction, and the decreased forced expiratory flow (FEF 25%-
75% ) suggests concomitant obstruction. The reduced diffusing lung capacity for
carbon monoxide (DLCO) is out of proportion to findings on spirometry and lung
volume studies. An isolated decreased DLCO may reflect destruction of lung
parenchyma (as in emphysema and interstitial lung disease), loss of pulmonary
vascular area (as in chronic pulmonary embolism and pulmonary hypertension), or
anemia.
The ventilation/perfusion scan is the standard study for chronic thromboembolic
disease. A mosaic perfusion pattern seen on HRCT scan, however, is also suggestive
of chronic thromboembolic disease with resultant pulmonary hypertension. The
mosaic pattern describes well-defined geographic areas of ground-glass attenuation
often conforming to individual or groups of secondary pulmonary lobules and is
usually associated with marked disparity in size of segmental vessels. The patient’s
age, the absence of crackles and clubbing, and the absence of intralobular fibrosis
and honeycombing on HRCT scan make idiopathic pulmonary fibrosis unlikely. The
patient’s age, ethnicity, and absence of emphysematous/bullous changes on HRCT
scan make emphysema unlikely. In early congestive heart failure, cephalization of
flow results in an increased DLCO. Once pulmonary edema occurs, DLCO will be
reduced, and crackles should be audible on examination. Although ascites occurs in
patients with cirrhosis, the other findings on examination are more suggestive of
right-sided heart failure.

Pulmonary Medicine and Critical Care:Question 54


The correct answer is C
Educational Objectives
Recognize indications for hospitalization in patients with chronic obstructive
pulmonary disease.
Critique
This patient should be hospitalized for further treatment and evaluation. He has
many comorbid conditions; his hypoxemia and hypercapnia have worsened, and he
63

is now acidemic. Furthermore, the patient is showing new signs of cor pulmonale.
These symptoms are all indications for hospitalization. It would not be appropriate to
discharge him.

Pulmonary Medicine and Critical Care:Question 55


The correct answer is D
Educational Objectives
Recognize the increased risk of tuberculosis associated with complicated silicosis.
Critique
This patient very likely has silicosis with progressive massive fibrosis now
complicated by tuberculosis. The development of a productive cough, hemoptysis,
constitutional symptoms, and a cavitary lesion in a patient with silicosis mandates
evaluation for tuberculosis, which should begin with submission of a sputum sample
for acid-fast bacillus stain and culture.
Surface mining is a high-risk occupation for the development of silicosis, and the
patient’s chest radiograph taken 2 years ago showed nodular infiltrates compatible
with silicosis. The patient’s most recent radiographic finding of upper lobe masses is
compatible with the development of progressive massive fibrosis. The association
between silicosis (especially advanced cases) and mycobacterial infection (both
tuberculous and nontuberculous) is well established and related to macrophage
dysfunction and distortion of the lung architecture; therefore, a high probability for
tuberculosis is necessary in this case.
Clindamycin is the treatment of choice for lung abscess, for which silicosis is not a
risk factor. Although a cavitary neoplasm is a possibility (especially in a former
cigarette smoker), bilateral upper lobe masses in the setting of silicosis more likely
represent progressive massive fibrosis.
Positron emission tomography scan or fiberoptic bronchoscopy would not be the
procedures of choice. Tuberculosis can be diagnosed by bronchoscopy, but the
sensitivity of expectorated sputum samples is sufficiently high that bronchoscopy is
usually not necessary. The rheumatoid factor level might be elevated in Caplan’s
syndrome (coexistent coal worker’s pneumoconiosis and rheumatoid arthritis
characterized by the formation of pulmonary rheumatoid nodules), but measuring it
is of no value in the setting of progressive massive fibrosis from silicosis.

Pulmonary Medicine and Critical Care:Question 56


The correct answer is C
Educational Objectives
Recognize the limitations of the pneumonia Patient Outcomes Research Team
Severity Index Score in hospitalization decisions.
Critique
A prediction rule has been developed with the intent of identifying patients with
pneumonia whose risk of dying within 30 days is low. Points are assigned on the
basis of various risk factors, physical findings, and laboratory test results, and are
totaled to provide an estimate of the risk of mortality. The patient in this scenario
would have a score of 67 points (22 for age, 20 for tachypnea, 15 for fever, 10 for
hypoxia). This score would put him in risk class II, which has a mortality rate of
<1%. It is suggested that patients in this risk class could be treated on an outpatient
basis.
There are, however, also risk factors known to be associated with serious outcome
failures where hospitalization would be most appropriate. These would include
obvious factors such as the need for mechanical ventilation, but also tenuously low
blood pressure, and multilobar pneumonia with hypoxia. This patient should be
hospitalized, possibly even in the intensive care unit, although the need for
64

intubation is not clearly indicated.

Pulmonary Medicine and Critical Care:Question 57


The correct answer is B
Educational Objectives
Recognize a classic presentation for cryptogenic organizing pneumonia.
Critique
This is a relatively classic presentation of cryptogenic organizing pneumonia
(previously known as idiopathic bronchiolitis obliterans with organizing pneumonia).
The presentation is like that of community-acquired pneumonia, but the symptoms
do not resolve with antibiotics. Most patients are symptomatic for 1 to 2 months
before diagnosis. Patients have migratory alveolar infiltrates, restrictive physiology,
and oxygenation problems.
Hypersensitivity pneumonia is unlikely in this case because the patient has had no
exposure and the symptoms have persisted for 2 months. Both resistant
pneumococcal pneumonia and chlamydia pneumonia should have responded to the
treatment. Bronchoalveolar cell carcinoma is not likely because he has no risk factors
for it and because of the migratory nature of the infiltrates.

Pulmonary Medicine and Critical Care:Question 58


The correct answer is C
Educational Objectives
Recognize that the presence of lung cancer is a contraindication to lung
transplantation.
Critique
Lung cancer is considered an absolute contraindication to lung transplantation for at
least 5 years after curative resection. The likelihood of recurrence after wedge
resection of a stage 1 lung cancer is approximately 40%. The administration of post-
transplant immunosuppression may further inhibit the patients ability to combat the
cancer.
This patient is not too old to be considered for lung transplantation. Most centers will
consider appropriate patients as old as 65 years for single lung transplantation. Lung
transplantation has not reliably resulted in a survival advantage for patients with
emphysema. Improvement in quality of life, however, has been consistently shown
for patients who survive the surgery.

Pulmonary Medicine and Critical Care:Question 59


The correct answer is B
Educational Objectives
Diagnose salicylate intoxication.
Critique
Salicylates should be highly considered in this patient with multiple clinical findings
consistent with intoxication. The effects of salicylate toxicity include respiratory
alkalosis, anion gap metabolic acidosis, and hyperthermia. The increase in insensible
water losses and hyperthermia often result in intravascular volume depletion.
Salicylates are also associated with a depressed level of consciousness and
noncardiogenic pulmonary edema. Other symptoms can include coagulation
abnormalities (prolonged prothrombin time), hepatic toxicity, and hypoglycemia.
Chronic salicylate ingestion should be suspected in this patient with a history of
degenerative joint disease. Chronic ingestors develop toxicity at lower blood levels
than patients with a single acute ingestion. Management includes alkalinization of
urine to enhance excretion of salicylates and hemodialysis for severe toxicity.
65

Ephedrine can produce a sympathomimetic syndrome with tachycardia,


hyperthermia, and altered mental status, but metabolic acidosis and pulmonary
edema would not be expected. Ethylene glycol ingestion can produce many of the
symptoms this patient is manifesting, including depressed level of consciousness,
metabolic acidosis, and pulmonary edema. Ethylene glycol ingestion often results in
an osmolar gap, however, and would not be associated with fever.
Lithium can also cause altered mental status and other neurologic findings, as well as
tachycardia due to intravascular volume depletion, but it would not be expected to
cause fever, metabolic acidosis, or pulmonary edema. Acetaminophen ingestion
alone would not account for the severe depression of consciousness, hyperthermia,
metabolic acidosis, or pulmonary edema.

Pulmonary Medicine and Critical Care:Question 60


The correct answer is A
Educational Objectives
Recognize indications for pulmonary rehabilitation.
Critique
Pulmonary rehabilitation is indicated for patients with chronic respiratory impairment
who, despite optimal medical management, are dyspneic, have reduced exercise
tolerance, or experience a restriction in activities.
Symptoms, disability, and handicap, rather than the severity of lung impairment,
dictate the need for pulmonary rehabilitation. Although pulmonary rehabilitation may
improve symptoms and exercise tolerance, often there is no improvement in
pulmonary function.

Pulmonary Medicine and Critical Care:Question 61


The correct answer is C
Educational Objectives
Treat latent tuberculous infection.
Critique
The patient likely has latent tuberculosis. He should be treated for latent tuberculous
infection with isoniazid.
The overall rate of adverse reaction to isoniazid therapy is less than 1% and is
associated with age greater than 50 years; it is felt that the risks of developing
tuberculosis outweigh those risks, and age alone is no longer considered an
appropriate contraindication to therapy. There may be better compliance with a
shorter duration of therapy, as can be done with combination rifampin and
pyrazinamide, but there is also a greater risk of hepatotoxicity with these agents
than with isoniazid. It is unlikely that the patient has active tuberculosis, given the
absence of suspicious findings on chest radiograph. A CT scan of the chest is not
likely to provide additional information in a patient with a normal chest radiograph.
Sputum examination is not necessary because the chest radiograph is negative.

Pulmonary Medicine and Critical Care:Question 62


The correct answer is A
Educational Objectives
Recognize the association between pulmonary hypertension and systemic sclerosis.
Critique
The patient has systemic sclerosis and associated pulmonary hypertension with cor
pulmonale. Systemic sclerosis, also referred to as scleroderma, can be subdivided
into diffuse and limited variants. Limited systemic sclerosis is also known as CREST
syndrome (calcinosis, Raynaud’s disease, esophageal dysmotility, sclerodactyly, and
66

telangiectasia). Pulmonary disease is estimated to occur in 70% to 85% of patients


with systemic sclerosis. Primary interstitial lung disease occurs frequently and is
indistinguishable clinically or radiographically from idiopathic interstitial pneumonia.
Pulmonary hypertension is the cause of death in as many as 60% of patients with
either the diffuse or the limited forms of the disease. This pulmonary hypertension
may occur in the absence of interstitial lung disease and has been associated with
peripheral Raynauds phenomenon, suggesting an abnormality in pulmonary vascular
reactivity as a primary event. There is no evidence at this time for a defined
hypercoagulable state in scleroderma, and pulmonary hypertension most often
develops without evidence of overt pulmonary embolism. The benefit of
anticoagulation in primary pulmonary hypertension, however, has led many
caregivers to try long-term warfarin therapy in these patients.

Pulmonary Medicine and Critical Care:Question 63


The correct answer is B
Educational Objectives
Recognize important initial medical treatment of status asthmaticus.
Critique
This patient is likely to have significant physiologic improvement when intravenous
magnesium sulfate (typically 2 g intravenously) is added to the regimen. Magnesium
is a smooth-muscle relaxer. In a patient with glaucoma, face-mask nebulization of
ipratropium bromide would be contraindicated, and a hand-held nebulizer would be
the appropriate means of delivery. Broad-spectrum antibiotics are indicated for acute
asthma attacks only if pneumonia is documented by infiltrates on chest radiograph
(not present in this case). Methylprednisolone is also a central component of
therapy, but administered intravenously; 40 to 60 mg every 6 hours would be an
appropriate dosage.

Pulmonary Medicine and Critical Care:Question 64


The correct answer is B
Educational Objectives
Recognize the importance of immunization for patients with advanced lung disease.
Critique
Common-sense measures such as hand washing are important in prevention of
infection and infectious complications. Immunization with the influenza and
pneumococcal vaccine is important to prevent infectious respiratory complications.
Although the vaccines are not entirely protective, they are likely to benefit patients
with emphysema. This patient should continue her social activities and should
continue with the rehabilitation program to maintain her strength and conditioning.
There is no evidence that long-term antibiotic use will prevent pneumonia.

Pulmonary Medicine and Critical Care:Question 65


The correct answer is A
Educational Objectives
Diagnose tuberculous pleurisy.
Critique
Tuberculous pleurisy occurs in as many as 10% of untreated PPD converters. At the
time of diagnosis, as many as 30% of patients may have a negative tuberculin skin
test because of mononuclear suppressor cells present in the peripheral circulation
but not in the pleural space. If the test is repeated in 6 to 8 weeks, it will be positive.

A tuberculous pleural effusion develops from a cellular-mediated immune response to


67

tuberculous antigens, which rupture into the pleural space from a peripheral focus of
active disease. These effusions may develop 3 to 6 months following the primary
infection or may reactivate anytime thereafter, usually caused by an impairment in
the immune system. The typical clinical presentation includes cough, chest pain, and
fever. Radiography shows a unilateral, small-to-moderate effusion; only one third of
patients have an associated infiltrate. On CT scan of the chest, however, a lesion is
found 75% of the time.
It is rare for a patient younger than 40 years and with a <20-pack-year history of
smoking to develop lung cancer. In patients with a malignant pleural effusion, the
percentage of lymphocytes in the pleural fluid is usually 50% to 70% and is rarely
higher than 80%. Furthermore, patients with lung cancer typically have ipsilateral
shift or an absence of contralateral mediastinal shift with a moderate pleural
effusion.
Patients with pneumonia develop parapneumonic effusions several days after the
onset of symptoms of pneumonia. The effusions are classically neutrophil-
predominant exudates, which become lymphocyte-predominant only in the resolution
phase after treatment with appropriate antibiotics. Patients with pulmonary
embolism typically have obvious risk factors for the development of deep venous
thrombosis (although some patients have hereditary thrombophilia that is unknown
at the time of presentation). The typical presentation is ipsilateral chest pain and a
small (less than one third of the hemithorax) exudative pleural effusion, which does
not have a high lymphocyte count. If the patient is treated with anticoagulation
therapy and does not have a pulmonary infarction, the effusion should peak by 72
hours and resolve in 7 to 10 days. If there is evidence of a radiographic infarction,
resolution time may be as long as 2 to 3 weeks.
Patients with benign asbestos pleural effusions have a known history of heavy
asbestos exposure and a latency period that averages 20 to 30 years from time of
initial asbestos exposure. In the acute phase, the effusion is neutrophil-predominant
and, in a later stage, may be lymphocyte-predominant, with percentages <80%.
There is a 30% incidence of pleural fluid eosinophilia.

Pulmonary Medicine and Critical Care:Question 66


The correct answer is B
Educational Objectives
Recognize the most effective treatment for the periodic limb movement disorder.
Critique
This patient has periodic limb movement disorder (PLMD), which is characterized by
periodic rhythmic extensions of the toes and dorsiflexion of the ankle and sometimes
by flexions of the knee and hip during the night. These movements last 0.5 to 3.0
sec and occur regularly (every 20 to 40 sec). They often occur in clusters and can
lead to sleep disruption.
This disorder can lead to hypersomnia or insomnia. It is more commonly seen in
patients with renal failure, iron deficiency, or peripheral neuropathy, and in patients
taking antidepressants. The most effective treatment is a dopaminergic agonist such
as pramipexole, carbidopa-levodopa, or ropinirole.
Treatment with iron (for ferritin <45 ngldL), an opioid such as oxycodone, or a
benzodiazepine such as clonazepam may provide limited benefit. Although
dopaminergic medications are prescribed for this disorder, there is no clear
relationship between PLMD and Parkinson’s disease. Drug treatment is titrated to
improve daytime symptoms. Gabapentin has been shown effective for treatment of
restless legs syndrome (characterized by dysesthesia in the legs during wakefulness
that lead to motor restlessness, often worse in the evening hours before bedtime);
however, gabapentin is not particularly effective for treatment of PLMD. Nonsteroidal
68

anti-inflammatory drugs such as ketorolac are not particularly effective at treating


PLMD.

Pulmonary Medicine and Critical Care:Question 67


The correct answer is C
Educational Objectives
Recognize latex exposure as an important cause of occupational asthma.
Critique
The significant worsening of this patients asthma symptoms should prompt an
evaluation for aggravating conditions such as home and workplace exposures,
sinusitis or rhinitis, gastroesophageal reflux disease, and medications (such as
nonsteroidal anti-inflammatory drugs and β-blockers). Although exposure to the
kitten as well as dust mite antigen (harbored in the carpet and draperies) could
contribute to poor asthma control, these home environmental factors do not account
for the patients clinical improvement during her time away from work.
This pattern suggests instead work-aggravated asthma, a form of occupational
asthma. As a nurse, the patient is at risk for latex-induced asthma caused by natural
rubber latex gloves.
Inquiry regarding the development of a contact urticarial rash after glove use is
essential to the diagnosis, because virtually all patients with latex-induced asthma
have skin symptoms when exposed to latex. Rhinitis (which this patient has), as well
as conjunctivitis, is also a finding in latex allergy. In severe cases, sensitized persons
can develop angioedema and anaphylactic shock.
Although treatment of rhinitis with antihistamines might be helpful, the cornerstone
of treatment of occupational asthma is avoidance of the offending agent. This patient
should be advised to avoid the use of latex gloves and any other latex products.
Furthermore, the cornstarch powder used to make gloves easy to put on can act as a
carrier for latex proteins. Respiratory tract exposure to latex is possible even without
direct latex exposure. For this reason, it may be necessary for everyone in her
workplace to use either unpowdered latex gloves or nonlatex gloves. Alternatively,
sensitized workers such as this patient can be reassigned. Pharmacologic treatment
of occupational asthma is identical to treatment of non-work-related asthma.
Treatment with an anti-inflammatory agent such as an inhaled corticosteroid is
indicated. Addition of a leukotriene-modifying drug concurrent with an inhaled
corticosteroid would be less beneficial than avoiding latex exposure.

Pulmonary Medicine and Critical Care:Question 68


The correct answer is D
Educational Objectives
Manage an exacerbation of chronic obstructive pulmonary disease.
Critique
Bronchodilators are effective in acute exacerbations of chronic obstructive pulmonary
disease (COPD). Ipratropium bromide and albuterol are probably equally effective.
There is no consensus regarding which bronchodilator to use initially or whether
combination therapy is superior to either alone. However, when a single
bronchodilator is used initially, a second bronchodilator should be added if there is no
improvement. Although albuterol may not result in substantial improvement, its side
effects (tachycardia, tremor) are usually well tolerated.
There are no data to support the use of broad-spectrum antibiotics, methyixanthines,
or dosages of corticosteroids greater than 125 mg administered intravenously every
6 hours in the treatment of acute exacerbations of COPD. The patient’s oxygenation
is adequate and higher levels of oxygen may promote further hypercapnia. The other
options may cause significant side effects (resistant organisms, cardiac and
69

neurologic toxicity, hyperglycemia and immunosuppression, hypercapnia and


respiratory acidosis). Despite appropriate management, the patient has deteriorated.
She may be developing respiratory muscle fatigue and is at risk for acute
hypercapnic respiratory failure. It is important to recognize the limitations of
pharmacologic therapy, which are largely directed at bronchospasm and airway
inflammation. At this point, the major focus of treatment is to be prepared to
institute assisted ventilation if necessary.

Pulmonary Medicine and Critical Care:Question 69


The correct answer is B
Educational Objectives
Assess a patient with smoke inhalation and potential burn injury to the airway.
Critique
When assessing a patient with smoke inhalation and potential thermal burn injury to
the upper airway, the first priority is evaluation of airway patency. The presence of
facial burns, soot in the mouth, carbonaceous sputum, or singed nasal hairs may
correlate with upper airway injury and a high risk of delayed airway compromise
from edema. Direct laryngoscopy may help classify this patients risk.
Facial burns do not necessarily indicate the presence of airway injury, even though
most patients with significant airway injury do have facial burns. Administration of
corticosteroids is not recommended where there is any significant airway
involvement.
Smoke inhalation can be acutely life-threatening because of asphyxia (acute oxygen
deprivation) or the toxicity of combustion products. Smoke inhalation injury to the
lower airways, which may be delayed2 days, is the result of smoke particles
triggering a severe inflammatory response, with edema and obstruction of small
bronchi. Acute respiratory distress syndrome and pneumonia are common
complications. Atelectasis and bronchopneumonia may develop, causing
intrapulmonary shunt, respiratory insufficiency, and perhaps respiratory failure.

Pulmonary Medicine and Critical Care:Question 70


The correct answer is C
Educational Objectives
Recognize the clinical features of reactive airways dysfunction syndrome.
Critique
This patient has developed reactive airways dysfunction syndrome (RADS) as a
consequence of accidental exposure to high concentrations of chlorine gas. RADS is
defined as the onset of asthma-like symptoms within 24 hours of a single exposure
to high concentrations of irritant gases, fumes, or chemicals. Inhalational challenge
testing with methacholine is likely to show abnormalities. There is no latency period
in RADS between initial exposure to the offending agent and the onset of asthma.
The pathophysiology involves direct airway injury with resulting inflammation leading
to nonspecific bronchial hyperresponsiveness. In contrast, occupational asthma
requires a latency period during which time susceptible persons become sensitized to
the offending antigen with the eventual development of symptoms that are
exacerbated by reexposure to the antigen. Other clinical criteria for RADS include
absence of preceding respiratory complaints, persistence of symptoms for at least 3
months after exposure, and presence of airflow obstruction on pulmonary function
tests or demonstration of nonspecific bronchial hyperresponsiveness by airway
challenge testing.
The asthma-like symptoms that persist after exposure are a consequence of the
nonspecific bronchial hyperresponsiveness. Patients with RADS may have normal
spirometry results, but airway challenge testing is positive. Because symptoms are
70

not caused by exposure to a sensitizing agent, reexposure to nonirritant levels of the


offending agent does not reproduce symptoms. Therefore, the patient need not avoid
swimming in chlorinated water.
The management of RADS is identical to that of asthma. Patients should be treated
with inhaled bronchodilators and anti-inflammatory agents. The long-term outcome
for patients with RADS has not been well studied, but many persons continue to
have airway symptoms and nonspecific bronchial hyperresponsiveness for years after
the inciting exposure.

Pulmonary Medicine and Critical Care:Question 71


The correct answer is C
Educational Objectives
Recognize the potential for late-phase reactions in patients with anaphylaxis.
Critique
This patient most likely has a food allergy. The classic anaphylactic response is an
allergic reaction involving antigen- and IgE-specific effector cells. When stimulated
by the antigen-IgE compound, these effector cells release a host of mediators
producing the clinical manifestations of anaphylaxis. Common food antigens include
peanuts, soybeans, eggs, milk, shellfish, and nuts. Anaphylaxis can progress to
shock, multiple-organ dysfunction, and death. Appropriate diagnosis and treatment
in a timely fashion are essential in reducing morbidity and mortality.
Improvement with treatment may be followed by late phase-reactions, typically 6 to
12 hours after the initial reaction, as a result of migration of effector cells into areas
of antigen introduction. Patients presenting to the emergency department with
severe anaphylaxis (hypotension or airway compromise or both) should be admitted
for further observation in a monitored setting.
Observing the patient in the emergency room for another hour would not provide
sufficient time to rule out a late-response reaction. Follow-up with an allergist,
instruction on use of epinephrine, and a corticosteroid taper are important
considerations for patients with anaphylaxis. Discharging a patient from the
emergency department after presentation with severe anaphylaxis would not be
prudent.

Pulmonary Medicine and Critical Care:Question 72


The correct answer is B
Educational Objectives
Treat an agitated patient in the intensive care unit.
Critique
The most likely cause of this patient’s agitation following treatment of the tension
pneumothorax is pain related to the placement of the chest tube. The most
appropriate treatment after a rapid assessment to ensure that the tube is functioning
is to provide analgesia with a narcotic such as fentanyl or morphine. In this case,
fentanyl is an excellent choice because acts more quickly than morphine.
Vecuronium, a neuromuscular blocking agent, would be completely inappropriate.
Although it is occasionally required in the treatment of patients with severe lung
disease on mechanical ventilation, it should be used only as a last resort after
appropriate sedation and analgesia have been tried.
Midazolam is an excellent drug for the treatment of anxiety, but because it has no
analgesic properties, it would not be appropriate until pain control is adequate. It is
also important to be aware that midazolam can occasionally cause increased
agitation, particularly in elderly patients. Propofol, a powerful, rapidly acting
sedative, has no analgesic properties and would be inappropriate until pain is
managed.
71

Haloperidol, a neuroleptic agent, is particularly useful in the treatment of patients


with delirium in the intensive care unit. Although this patient is at significant risk for
delirium because of his age, history of dementia, and acute illness with respiratory
failure, it is far more likely that his agitation is due to pain, which treatment with
haloperidol would not address.

Pulmonary Medicine and Critical Care:Question 73


The correct answer is D
Educational Objectives
Diagnose nontuberculous mycobacterial disease.
Critique
The diagnosis of nontuberculous mycobacterial (NTM) disease is dependent upon
satisfying clinical, radiographic, and bacteriologic criteria. Clinical and radiographic
criteria have been met in this case, but bacteriologic criteria must be met to confirm
the diagnosis.
Sputum is a reliable source of culture material to make a diagnosis of NTM disease,
and requires either three positive cultures within 1 year, or two positive cultures and
one positive acid-fast bacillus smear. Because the patient is capable of producing
sputum, this is the best option.
Bronchoscopy can be useful in making the diagnosis, especially if patients cannot
produce sputum. A positive culture from bronchoalveolar lavage fluid can confirm the
diagnosis if 2+ or greater acid-fast bacilli are present on the smear. Otherwise the
culture must show growth of mycobacteria of 2+ or greater. Performance of a biopsy
is useful, because any growth of mycobacteria from a biopsy specimen is
confirmatory of NTM disease.
HRCT scan of the chest may demonstrate additional features, such as multifocal
bronchiectasis, but is unnecessary to fulfill the radiographic criteria for the diagnosis.

Pulmonary Medicine and Critical Care:Question 74


The correct answer is C
Educational Objectives
Recognize the clinical presentation of hypersensitivity pneumonitis.
Critique
This patient is presenting with a history most compatible with hypersensitivity
pneumonitis (HP), a specific allergic reaction in the lower respiratory tract to inhaled
low-molecular-weight antigens. The clinical presentation is typically one of recurrent
acute episodes of fever, cough, and dyspnea that begin 4 to 6 hours after antigen
exposure with spontaneous resolution 24 to 48 hours after antigen avoidance. A
diagnosis of HP is suggested by 1) compatible clinical presentation coupled with a
history of exposure to a candidate antigen, 2) demonstration of specific immunity to
that antigen, and 3) resolution of disease with antigen avoidance or recurrence of
disease with repeated exposure.
Environmental antigen challenges have been used to confirm the diagnosis of HP, but
at present there are no widely accepted protocols or standardized antigen
preparations. A clinical response to prolonged antigen avoidance or disease
recurrence on repeated exposure is as helpful where practical. A specific immune
response may be confirmed by the detection of circulating precipitating antibodies to
the antigen of interest. However, the majority of an exposed population may develop
precipitating antibodies with only a minority manifesting clinical signs of HP.
Therefore, the presence of circulating antibody is supportive but not sufficient
evidence to make the diagnosis of HP in the absence of an appropriate clinical
scenario.
72

The finding of a precipitin band in a life-long farmer does not of itself imply a
diagnosis of HP. The disease represents a specific immunologic response in
susceptible persons to repeated antigen exposure, which distinguishes it from the
nonspecific lower respiratory tract inflammation seen after a high-level organic dust
exposure (as occurs in mycotoxicosis, endotoxin inhalation, and organic toxic dust
syndrome). Thus, simultaneous disease in similarly exposed persons argues against
a diagnosis of HP. HP is not associated with immediate skin test reactivity because it
is not mediated by IgE. The demonstration of reversible airflow obstruction is a
nonspecific finding and can occur with asthma, postinfectious bronchial
hyperreactivity, endotoxin inhalation, or HP.

Pulmonary Medicine and Critical Care:Question 75


The correct answer is D
Educational Objectives
Select the best method for diagnosing carbon monoxide poisoning.
Critique
This patient’s presentation is suggestive of possible carbon monoxide poisoning for
several reasons. Faulty heating systems are among the most common causes of
carbon monoxide poisoning and are most likely to cause problems during use in cold
weather months. Headache and nausea are the most frequent symptoms associated
with carbon monoxide exposure, and the association of symptoms with a specific
environment is also suggestive.
The best confirmation of significant carbon monoxide exposure is an elevated
carboxyhemoglobin level. An initial level may not be helpful; however, if a significant
period of time has elapsed since exposure. If the carboxyhemoglobin level is normal
and the suspicion for carbon monoxide exposure is high, then the home environment
would need to be assessed for the presence of carbon monoxide. Because pulse
oximetry often overestimates oxygenation in the setting of carbon monoxide toxicity,
it would not be helpful. Although carbon monoxide toxicity is associated with
ischemia of tissues, an elevated lactate level is nonspecific and would be more likely
to occur in severe exposure. The lactate level is more reliable than the
carboxyhemoglobin level in determining the severity of toxicity. Similarly, an
electrocardiogram is unlikely to show changes of ischemia unless there is significant
exposure; ischemic changes would be nonspecific.

Pulmonary Medicine and Critical Care:Question 76


The correct answer is C
Educational Objectives
Diagnose post-traumatic hemothorax.
Critique
The patient has a hemothorax subsequent to his initial injury. The process of
increased production and transmigration of eosinophils to the pleural space typically
requires a period of 7 to 10 days following hemothorax. Subsequently, eosinophils
begin to appear in the peripheral blood. Peripheral eosinophilia resolves when the
hemothorax resolves. Knowledge of the time course and kinetics of pleural fluid
eosinophilia and subsequent peripheral eosinophilia support a clinical diagnosis of
traumatic or post-surgical hemothorax and avoids additional diagnostic testing,
which may lead to increased morbidity and expense.
73

By contrast, eosinophils move rapidly into pleural tissue and pleural fluid following a
pneumothorax. Approximately 30% of patients with benign asbestos pleural effusion,
who are typically asymptomatic, may develop pleural fluid eosinophilia. Benign
asbestos pleural effusion is usually associated with heavy asbestos exposure. Other
causes of pleural fluid eosinophilia (>10% of total nucleated cells) include pulmonary
infarction, parasitic disease, fungal disease (coccidioidomycosis and histoplasmosis),
drug-induced pleural effusion (dantrolene and nitrofurantoin), Hodgkin’s lymphoma,
carcinoma, and Churg-Strauss syndrome. Pleural fluid eosinophilia is exceedingly
rare in tuberculous pleurisy. The prevalence of malignancy has been shown to be
similar in eosinophilic and noneosinophilic pleural effusions.
Although the patient was a smoker and at increased risk for lung cancer, malignant
effusions do not resolve spontaneously. The patient has no medical risk factors for
deep venous thrombosis. He did not have spontaneous chest pain, and he has a
large pleural effusion (the effusion with pulmonary embolism is virtually always less
than a third of the hemithorax).
Although paragonimiasis is considered a tropical disease, it has been diagnosed in
North Americans. Worms migrate through and sometimes remain in the pleura.
Affected patients may have gastrointestinal and constitutional symptoms several
weeks before they develop pleurisy; most patients with paragonimiasis, however, are
not systemically ill. Patients who have predominantly pleural paragonimosis are not
likely to have ova in the sputum and are often misdiagnosed as having tuberculosis,
as the majority will have concomitant pulmonary abnormalities. The definitive
diagnostic test is demonstration of the ova in sputum or stool.

Pulmonary Medicine and Critical Care:Question 77


The correct answer is B
Educational Objectives
Recognize the need to institute noninvasive positive-pressure ventilation.
Critique
In a patient who has acute chronic respiratory acidosis, bilevel noninvasive
ventilation may provide the ventilation necessary to increase elimination of carbon
dioxide and avoid the need for intubation. The assist to inspiration is achieved with
pressure ventilation. The expiratory pressure is equivalent to positive end-expiratory
pressure (PEEP), and inspiratory pressure is equivalent to the sum of pressure
support and PEEP. Therefore a bilevel inspiratory setting of 13 cm H2O and
expiratory pressure setting of 5 cm H2O is equivalent to a standard ventilator setting
of pressure support of 8 cm H2O.

Pulmonary Medicine and Critical Care:Question 78


The correct answer is E
Educational Objectives
Diagnose obesity hypoventilation syndrome and recognize its close association with
obstructive sleep apnea.
Critique
Extreme obesity and elevated PCO2 during wakefulness are characteristic of obesity
hypoventilation syndrome. It is often associated with symptoms of sleep-disordered
breathing, including morning headaches, hypersomnolence, and sleep arousals.
Patients with severe disease can have dyspnea, hypoxemia, and pulmonary
hypertension. Overnight polysomnography is the appropriate test for this patient,
because obstructive sleep apnea almost always accompanies this disorder.
The diagnosis of obesity hypoventilation syndrome is made when the PCO2 is
elevated (>45 mm Hg) in an obese patient (usually >150% ideal body weight),
where other causes of hypoventilation such as hypothyroidism, chronic obstructive
74

pulmonary disease, and neuromuscular disease have been excluded. A body mass
index of 42 is diagnostic of morbid obesity.
An echocardiogram might show pulmonary artery hypertension but is not likely to
lead to the diagnosis. Headaches in this patient are due to carbon dioxide retention;
a CT scan of the brain will likely be unrevealing. The Beck Depression Inventory, a
measure of depression symptoms, might be abnormal in this patient but would not
diagnose the underlying cause of the symptoms.

Pulmonary Medicine and Critical Care:Question 79


The correct answer is D
Educational Objectives
Treat septic shock.
Critique
This patients urine output has significantly decreased. Even though he has received 3
L crystalloid over the last 6 hours, his increased venous capacitance and capillary
leak likely imply a continuing decrease in left ventricular preload. Inputs and outputs
are of no value in the management of early severe sepsis and septic shock because
fluid is leaving the intravascular space. Therefore, in the presence of organ
hypoperfusion, fluid should be administered until either lung auscultation indicates
volume overload or oxyhemoglobin desaturation occurs. In this patient, oxygenation
is being achieved without difficulty with 50% oxygen administration, and intubation
offers a significant safety net for further aggressive fluid resuscitation.
Although the platelet count is decreased to 40,000/μL, there is no evidence that the
patient is bleeding and no evidence that a procedure is planned. Transfusion is
typically performed to maintain platelet count of 50,000/μL or higher in the presence
of hemorrhage or if procedures are planned. In the absence of these two clinical
events, platelets are not typically transfused until the count drops below 20,000/μL.
There is no evidence that maintaining a mean arterial pressure of 75 mm Hg with
vasopressor agents offers an advantage over a mean arterial pressure of 65 mm Hg.
There is no evidence that dopamine offers an advantage over norepinephrine when
used as a vasopressor to treat septic shock. In fact, although no prospective
randomized clinical trials exist, there is more physiologic logic and literature support
for norepinephrine as a preferred agent.

Pulmonary Medicine and Critical Care:Question 80


The correct answer is A
Educational Objectives
Manage an exacerbation of chronic obstructive pulmonary disease.
Critique
This patient has developed hypercapnic respiratory failure. The choice between
intubation and noninvasive positive-pressure ventilation (NPPV) is a matter of
judgment. In this case it is best to electively intubate the patient under controlled
circumstance now rather than emergently after a likely failure of NPPV later. Given
his severe baseline degree of obstruction (FEV1 of 38% predicted) and his recent
episode of prolonged mechanical ventilation, the desire to avoid intubation is
understandable. NPPV reduces the rate of intubation in appropriately selected
patients with acute exacerbations of chronic obstructive pulmonary disease (COPD).
In this case, however, there are several contraindications to NPPV:
possible pneumonia (fever, purulent sputum, consolidation of chest radiograph,
leukocytosis), inability to protect the airway (lethargy, pooling of secretions, weak
cough), and hemodynamic instability (relative hypotension, multifocal atrial
tachycardia). Randomized trials have shown no benefit to mucolytics or mechanical
percussion of the chest in acute exacerbations of COPD. Nasotracheal suctioning can
75

result in hypoxia, vocal cord damage, and aspiration.


The major pathophysiologic abnormality is not hypoxia but hypercapnia resulting
from airway obstruction, hyperinflation, and respiratory muscle fatigue. Higher
oxygen concentrations will not improve the patients condition and may worsen it by
promoting carbon dioxide retention. Theoretically, the lower density of a helium-
oxygen mixture decreases resistance to flow, hyperinflation, and work of breathing.
There are, however, no well-controlled studies establishing the effectiveness of
helium-oxygen in this setting.

Pulmonary Medicine and Critical Care:Question 81


The correct answer is C
Educational Objectives
Follow current recommendations for vaccination for influenza and pneumonia.
Critique
He should receive the influenza vaccine. He has already received a pneumonia
vaccine, which should provide him with acceptable protection; giving additional
vaccine has not been proved to boost response. This patient is considered at risk for
pneumonia because of his health risks and age. The only reason not to give him the
influenza vaccine is allergy to eggs. His recent illness does not place him at risk for
an untoward effect to vaccination, which he should receive before he leaves the
hospital. Influenza vaccine prevents influenza illness in approximately 70% to 90%
of healthy adults older than 65 years. Older persons and persons with certain chronic
diseases might develop lower post-vaccination antibody titers than healthy young
adults and thus can remain susceptible to influenza-related upper respiratory tract
infection. There is a reported vaccine efficacy of 58% against influenza respiratory
illness for patients aged 60 years and older, but the efficacy might be lower among
those aged 70 years and older. The vaccine can also be effective in preventing
secondary complications and reducing the risk of influenza-related hospitalization
and death among adults 65 years and older with and without high-risk medical
conditions (such as chronic obstructive pulmonary disease). Among elderly persons
living outside of nursing homes or similar long-term care facilities, influenza vaccine
is 30% to 70% effective in preventing hospitalization for pneumonia and influenza.
Among elderly persons residing in nursing homes, influenza vaccine is most effective
in preventing severe illness, secondary complications, and death.

Pulmonary Medicine and Critical Care:Question 82


The correct answer is A
Educational Objectives
Manage acetaminophen poisoning.
Critique
Appropriate management of acetaminophen overdose is important to prevent hepatic
toxicity and mortality. The patient took a potentially toxic amount (15 g) 12 hours
before presentation. His acetaminophen level should be determined for comparison
with the Rumack-Matthew nomogram; results may not be available for several
hours, however. Because N-acetylcysteine is most effective within 8 hours of
ingestion, a loading dose of oral N-acetylcysteine should be administered to this
patient immediately, pending results of the blood level. If the acetaminophen level is
in the nontoxic range, no further doses need to be administered.
Activated charcoal can be administered within 4 hours of ingestion and is beneficial
in adsorbing acetaminophen, but it is unlikely to have any significant effect for this
patient 12 hours after ingestion of the drug. Flumazenil is an antidote for
benzodiazepine toxicity, but it is not necessary for this patient who is alert and
oriented and has no respiratory depression. Administration of naloxone is not
76

indicated because no narcotic was taken.

Pulmonary Medicine and Critical Care:Question 83


The correct answer is C
Educational Objectives
Recognize indications for intubation in acute respiratory failure associated with
neuromuscular disease.
Critique
The leading cause of death in neuromuscular disease is respiratory failure and
pneumonia. Although noninvasive modalities can be helpful in treating the patient
with neuromuscular respiratory disease, intubation or tracheostomy is required when
patients are unable to protect the airway because of impaired swallowing or cough
function. It is also necessary when noninvasive techniques have failed to maintain
adequate PaO2 , PaCO2, or secretion management. It might also be considered if
staff lacks expertise in the use of noninvasive devices.

Pulmonary Medicine and Critical Care:Question 84


The correct answer is B
Educational Objectives
Select the appropriate diagnostic test for cystic fibrosis.
critique
The diagnosis of cystic fibrosis is made by clinical history and documentation of
dysfunction of the cystic fibrosis transmembrane conductance regulator (CFTR) gene,
a cyclic adenosine monophosphate (cAMP)-activated chloride channel in epithelial
cells. The most appropriate initial test is the quantitative pilocarpine iontophoresis
sweat chloride test, which should be performed at an accredited laboratory.
Gene mutation testing might be helpful, but patients who present initially as adults
often have unusual genotypes that are not detected by many commercially available
test batteries. Although fecal fat testing is helpful for establishing whether the
patient has fat malabsorption due to pancreatic insufficiency, fat malabsorption is
symptomatic of disease states other than cystic fibrosis.
Although cystic fibrosis should be suspected in any outpatient with a sputum culture
positive for mucoid-strain Pseudomonas aeruginosa, this organism is now
increasingly also recovered from outpatients who have bronchiectasis without cystic
fibrosis. Measurement of nasal potential difference is sometimes useful in supporting
a diagnosis of cystic fibrosis in atypical or mild cases but is not routinely available.

Pulmonary Medicine and Critical Care:Question 85


The correct answer is C
Educational Objectives
Recognize that cataplexy is pathognomonic of narcolepsy.
critique
Narcolepsy affects one of every 2000 to 10,000 persons in the United States and is
associated with invasion of rapid-eye-movement (REM) sleep into wakefulness.
Patients report one or more of four hallmark symptoms: cataplexy, excessive
daytime somnolence, hypnagogic hallucinations, and sleep paralysis. The presence of
cataplexy is pathognomonic of this disorder, and is characterized by sudden attacks
of focal or generalized muscle weakness in response to strong emotion, such as
anger or laughter. Hypnagogic hallucinations are vivid, often frightening dreams
occurring shortly after sleep onset. Sleep paralysis refers frightening awareness of
generalized paralysis at transitions between sleep and wakefulness. The diagnosis of
narcolepsy is confirmed by documentation of sleep latency less than 5 mm on
77

multiple sleep latency test (MSLT) and early onset of REM sleep on MSLT or
polysomnography.
Temporal lobe epilepsy sometimes occurs predominantly at night but is not
associated with cataplexy. Panic disorder can also occur predominantly during sleep
but does not cause cataplexy. The parasomnias are motor disorders of sleep,
including sleep walking. They do not cause persistent daytime hypersomnolence or
cataplexy.

Pulmonary Medicine and Critical Care:Question 86


The correct answer is D
Educational Objectives
Treat anaphylactic shock.
critique
This patient is in anaphylactic shock, most likely in reaction to the contrast dye used
in the CT scan. Anaphylactic shock is the most severe presentation of anaphylaxis,
which can be triggered by immunologic or nonimmunologic mechanisms. Agents
associated with anaphylactoid reactions include radio-contrast dye, opiates, aspirin,
and other nonsteroidal anti-inflammatory drugs. The cornerstone of therapy for
anaphylactic shock is epinephrine.
Epinephrine can be administered subcutaneously, intramuscularly, or intravenously.
Subcutaneous administration is preferred, usually at a dosage of 0.3 to 0.5 mL
(1:1000 dilution). Intravenous administration is reserved for cases of severe
hemodynamic collapse; extreme caution is necessary in patients at risk for
myocardial ischemia. Aggressive intravenous fluid resuscitation (crystalloids or
colloids) is also imperative as an adjunct to epinephrine.
Dobutamine is a sympathomimetic agent with predominantly inotropic effects; it
would not correct the hypotension in anaphylactic shock. Dopamine is sometime
used as an adjunct in cases refractory to epinephrine and fluids, but it is not a first-
line treatment for anaphylactic shock. Corticosteroids play a role in the later
treatment of anaphylaxis, to prevent late-phase reactions and alleviate airway
edema and bronchoconstriction. Corticosteroids have no immediate effect on
hemodynamic abnormalities in anaphylactic shock.

Pulmonary Medicine and Critical Care:Question 87


The correct answer is D
Educational Objectives
Treat severe sepsis with recombinant activated protein C therapy.
Critique
This patient has sepsis associated with pulmonary and renal failure. Drotrecogin alfa
(activated), a recombinant human activated protein C, has been shown to improve
survival rates in such patients. The administration of this agent would be
contraindicated only in the presence of severe coagulation abnormalities.
Antithrombin Ill is another natural anticoagulant, but its beneficial effects have not
been demonstrated in severe sepsis.
Low-dose (stress dose) corticosteroids may be beneficial in septic shock, but this
hemodynamically stable patient would not qualify for low-dose treatment. A single,
large dose of methylprednisolone has not been shown to improve outcome in large
clinical trials. The routine administration of low doses of dopamine has not been
shown to be beneficial.

Pulmonary Medicine and Critical Care:Question 88


The correct answer is D
78

Educational Objectives
Recognize potential complications of continuous high-dose sedation with
benzodiazepines.
Critique
The best approach is continued observation. The patient has received large
quantities of lorazepam for more than a week, which have undoubtedly accumulated
in fat stores. It may take several days more before drug levels decrease, allowing
her to wake up.
There is no indication to resume plasmapheresis unless there are signs of recurrent
thrombotic thrombocytopenic purpura. Although a tracheostomy and percutaneous
gastrostomy tube may be necessary at some point, it is reasonable to expect that
her mental status will improve in a timely manner, making these interventions
unnecessary.
Metabolic acidosis due to propylene glycol toxicity has been described in patients
receiving high-dose lorazepam, but this is an unlikely explanation for the depressed
mental status and there is no clinical indication to measure propylene glycol levels.
Clinical sedation guidelines emphasize avoiding oversedation and drug accumulation
by providing the smallest amount of drug necessary to achieve clinical end points.
Daily interruption of sedation and avoiding continuous infusions have also been
advocated to avoid excessive sedation.

Pulmonary Medicine and Critical Care:Question 89


The correct answer is B
Educational Objectives
Recognize clinical features and late sequela of silo filler’s disease.
Critique
This patient has developed bronchiolitis obliterans as a late complication of silo filler’s
disease. Silo filler’s disease is the acute lung injury that occurs from the inhalation of
oxides of nitrogen, such as nitrogen dioxide. These gases are created by the
fermentation of plant nitrates, and they concentrate in silos just above the surface of
the silage.
Because nitrogen dioxide is relatively insoluble in water, acute exposure may cause
minimal irritation to the mucous membranes and upper airway. Therefore, patients
may be unaware of their initial exposure. Within hours, however, patients develop
cough, dyspnea, wheezing, and chest pain, which occur from direct injury to the
distal airways. Severe exposure to high concentrations of nitrogen dioxide can cause
acute respiratory distress syndrome. Patients with silo filler’s disease should be
treated with corticosteroids to minimize the late complication of bronchiolitis
obliterans.
After recovery from the acute inhalational injury, some patients develop recurrence
of respiratory symptoms 3 to 6 weeks after the initial exposure, including severe
dyspnea, nonproductive cough, wheezing, chills, and fever. Chest radiograph may be
normal or may show hyperinflation or patchy infiltrates. Spirometry reveals
irreversible airflow obstruction, and histologic evaluation shows bronchiolitis
obliterans. Treatment with corticosteroids is recommended. Patients unresponsive to
therapy may develop progressive respiratory failure with need for lung
transplantation.
Although farmers can develop hypersensitivity pneumonitis from inhalation of spores
of Thermoactinomyces harbored in moldy hay, this so-called farmer’s lung is not
caused by exposure to silage.
Agricultural workers appear to be at increased risk for occupational asthma, but this
patient’s history is not compatible with occupational asthma, which requires a
latency period between exposure and subsequent development of asthma symptoms.
79

Although reactive airways dysfunction syndrome does cause acute airway symptoms
within 24 hours of toxic inhalation, this patient’s known exposure to nitrogen dioxide,
as well as the bimodal distribution of symptoms, is most compatible with bronchiolitis
obliterans. Acute respiratory distress syndrome is an acute rather than late
complication of nitrogen dioxide inhalation.

Pulmonary Medicine and Critical Care:Question 90


The correct answer is C
Educational Objectives
Use echocardiography in the initial bedside assessment of a patient with shock of
unknown cause.
Critique
This patient is hypotensive and therefore unstable. Differential diagnosis includes
post-operative abdominal complication from the previous surgery, pulmonary
embolism, and postoperative ischemia. T-wave inversion on electrocardiogram is
nonspecific. Intra-abdominal hemorrhage is a remote possibility. It would be unwise
to move the patient to a remote testing site if suitable alternatives are available.
Such an alternative is bedside echocardiography, which can provide valuable
information concerning the differential diagnosis in this case.
Pulmonary embolism might demonstrate acute right ventricular dilation, myocardial
ischemia focal wall motion abnormalities, and sepsis global hypokinesis. Hemorrhage
would be expected to produce a hyperdynamic poorly filled left ventricle. Initially
transthoracic echocardiography would be performed in this patient who has an
aesthetic body habitus and in whom esophageal insertion would require sedation,
which is less than ideal in a hypotensive patient.

Pulmonary Medicine and Critical Care:Question 91


The correct answer is C
Educational Objectives
Use noninvasive positive-pressure ventilation to treat a patient with neuromuscular
disease.
Critique
Amyotrophic lateral sclerosis is a progressive neuromuscular disease with no known
cure. The ultimate cause of death is respiratory failure and pneumonia. Therapeutic
interventions are aimed at improving quality and length of life where possible.
Current practice recommendations by the American Academy of Neurology are to
initiate noninvasive positive-pressure ventilation by nasal cannula or face mask when
the vital capacity falls below 50% of predicted or the patient exhibits symptoms of
nocturnal or diurnal hypoventilation. Several studies have shown that patients who
tolerate this modality live 12 to 18 months longer and have improved quality of life.
Attempts to improve airway secretion mobilization or clearance or prevent infection
have not been shown to have any effect on mortality rate in amyotrophic lateral
sclerosis.

Pulmonary Medicine and Critical Care:Question 92


The correct answer is C
Educational Objectives
Manage terminal illness in a person from another culture.
Critique
In some countries and cultures, current conventional wisdom dictates that patients
not be told of a terminal diagnosis or prognosis. In North America and elsewhere, the
convention is a direct physician-patient relationship that generally includes full
80

disclosure to the patient of diagnosis and prognosis.


In this instance, the physician is asked to see a man who is unlikely to survive longer
than 1 year. Planning for health care near the end of life should begin shortly.
Decisions to initiate or limit life support might be necessary at any time. The
patient’s capacity to make decisions on his own behalf is unknown as yet and may be
difficult to determine if language translation through the daughter is necessary.
Clinicians are advised to respect cultural and religious differences but also to abide
by their own moral compass. Clinician-patient relationships are formed voluntarily
and entail expectations that are agreeable to both parties. The physician should offer
to see the patient to determine if urgent intervention is needed but should not agree
to answer the patient’s questions falsely or incompletely. After this first visit, it may
be possible to persuade the daughter to support a physician-patient relationship
based on full but compassionate disclosure.
If it is not possible to agree on mutual expectations for an ongoing physician-patient
relationship, the man and his family members should be advised to seek another
physician, perhaps an American of recent Lithuanian descent who might be more
agreeable to their expectations.

Pulmonary Medicine and Critical Care:Question 93


The correct answer is D
Educationai Objectives
Treat a patient for nerve gas exposure.
Critique
The constellation of signs and symptoms in this patient are typical for exposure to a
nerve agent.
Chemical exposure should be suspected when large numbers of patients present
from a single location with similar respiratory, dermatologic, ocular, or neurologic
problems.
Nerve agents cause a cholinergic crisis by inhibiting cholinesterase, preventing
degradation of acetylcholine at postsynaptic receptors. There are three major
components of this toxicity: muscarinic, nicotinic, and central nervous system
effects. Muscarinic effects include salivation, lacrimation, diarrhea, and potentially
life-threatening bronchorrhea/bronchospasm. N icotinic effects include fasciculation,
muscle weakness, and respiratory failure secondary to diaphragmatic paralysis.
Central nervous system effects include altered mental status and seizures. The major
life-threats are hypoxemia from bronchorrhea and ventilatory failure secondary to
diaphragmatic weakness.
Pralidoxime (2-PAM) reactivates acetylcholinesterase, and can reverse the muscle
weakness, paralysis, and respiratory depression. Benzodiazepines can be
administered to help prevent or control seizures.
Although atropine will treat the bronchorrhea, high doses and continuous infusion
may be necessary; it will not reverse the muscle weakness or respiratory depression.

Nerve agents can be absorbed through the skin. Thorough decontamination is


imperative, including removing the patient’s clothing and washing with soap and
water. Decontamination must be performed in a well-ventilated area to prevent
contamination of health care workers. Some chemicals, most notably nerve agents,
may persist in the environment even after decontamination and pose a significant
threat to medical staff.

Pulmonary Medicine and Critical Care:Question 94


The correct answer is E
Educational Objectives
81

Recognize pleural fluid pH as a tool in the diagnosis of an exudative pleural effusion.


Critique
Approximately 30% of patients with a malignant pleural effusion present with a low
pleural fluid pH. A low pleural fluid pH and glucose in malignancy are associated with
a large tumor burden on the pleural surface. The abnormal pleural membrane
inhibits end-products of glucose metabolism from being removed from the pleural
space at a normal rate and, therefore, results in a low glucose concentration and a
low pH, usually in the range of 6.90 to 7.29. Patients with malignant effusions with a
low pleural fluid pH and glucose tend to have a shorter survival, less successful
response to pleurodesis, and higher diagnostic yield on initial pleural fluid cytology
than patients with malignant effusions with a normal pH and glucose.
A complicated parapneumonic effusion (CPE) is a neutrophil-predominant exudate;
its end-stage is an empyema. CPE virtually always have a pH of <7.30, and the pH is
often as low as 6.00. If the effusion is purulent (empyema), a pH does not have to
be measured, as pleural space drainage is indicated based solely on the presence of
purulence. The low pH of a complicated parapneumonic effusion results from
increased pleural fluid metabolic activity that is due to a high glycolytic rate from
neutrophil phagocytosis and bacterial metabolism. The end-products of glucose
metabolism (CO2 and lactic acid) do not equilibrate with blood concentrations, as in
the normal state, due to an abnormal pleura, resulting in high CO2 and lactate levels
and acidosis.
The low pH of esophageal rupture develops as a result of an anaerobic empyema
that has developed because of a direct pathway of anaerobes from the mouth into
the pleural space through the mediastinum. In contrast to patients with
parapneumonic empyemas, those with esophageal rupture appear to have a constant
supply of glucose, allowing for continued acid production and a typical pH of about
6.00.
In a chronic rheumatoid pleurisy, the pleura becomes fibrotic and prevents the end-
products of glucose metabolism (CO2 and lactic acid) from exiting the pleural space
at a normal rate, resulting in both a low pH (7.00) and low glucose, often
approaching 0 mg/dL.
There is a rare report of acute pancreatitis presenting with pleural fluid acidosis of
7.28. However, this can only occur with acute pancreatitis when it is associated with
severe pleural injury from another cause, as a high amylase concentration is not
associated with significant pleural injury. The usual pleural fluid pH in acute
pancreatitis ranges from 7.32 to 7.37.

Pulmonary Medicine and Critical Care:Question 95


The correct answer is D
Educational Objectives
Diagnose and treat adrenal insufficiency.
Critique
This patient has hypotension and persistent vasopressor requirements following a
diagnosis of sepsis. A significant concern in this patient is the possibility of adrenal
insufficiency.
Empiric corticosteroid therapy is indicated in this patient. Since hydrocortisone
interferes with cortisol measurement, an initial dose of intravenous dexamethasone
followed by the adrenocorticotropic hormone (ACTH) stimulation test would be the
most appropriate response, because dexamethasone does not interfere with the
cortisol assay. Once cortisol levels after ACTH simulation are obtained, the therapy
can be switched to hydrocortisone.
There are three considerations for this patient in septic shock who has potential
adrenal insufficiency:
82

(1) Adrenal shock may present in a similar fashion as septic shock and have a similar
clinical profile including fever—unlikely in this case because the chest radiograph
supports a diagnosis of pneumonia. (2) Steroid release is the primary way that the
body handles the pro-inflammation response of severe sepsis. In patients previously
taking corticosteroids, there may be adrenal gland suppression that prevents
adequate adrenal response—the primary concern in this case. (3) Recently described
is a relative adrenal insufficiency that may occur in septic shock, which is thought to
occur when the pro-inflammatory response overwhelms the ameliorating effect of
adrenal release of corticosteroids. It is characterized by inability to raise cortisol level
in response to ACTH stimulation test in patients with septic shock.
Patients with relative adrenal insufficiency are at increased risk for mortality and
survival may be improved by administration of stress-dose steroids (hydrocortisone
50 mg intravenously every 6 hours and fludrocortisone 50 μgld administered orally,
both for 7 days).

Pulmonary Medicine and Critical Care:Question 96


The correct answer is C
Educational Objectives
Recognize the potential benefits of lung-volume-reduction surgery.
Critique
Lung-volume-reduction surgery is an appropriate surgical procedure in properly
selected patients with advanced emphysema. It is commonly used as a bridge to
transplantation in patients deemed too ill to wait the estimated 2 years on a lung
transplantation waiting list. It has been shown to significantly increase exercise
performance, quality of life, forced expiratory volume in 1 sec (FEV1), and lung
elastic recoil and decrease hyperinflation in patients with severe emphysema.

Pulmonary Medicine and Critical Care:Question 97


The correct answer is A
Educational Objectives
Recognize the safest and rnost effective mode of ventilator support for patients with
acute respiratory
distress syndrome.
Critique
This patient has acute respiratory distress syndrome (ARDS) secondary to
pneurnococcal pneumonia. The largest study of mechanical ventilation in this
condition was performed by the ARDS Clinical Trials Network. This trial showed that
tidal volurnes of 6 rnL/kg of ideal body weight are superior to the previous standard
of 12 rnL/kg of ideal body weight with regard to duration of rnechanical ventilation
and survival. The ARDS Network formula for calculation of ideal body weight is based
on sex and height, the two main predictors of lung size.

Pulmonary Medicine and Critical Care:Question 98


The correct answer is D
Educational Objectives
Recognize that obstructive sleep apnea is associated with an increased incidence in
hypertension.
Critique
83

Obstructive sleep apnea (OSA) has been strongly associated with systemic
hypertension. The degree of hypertension correlates closely with the severity of OSA
as measured by the apnea-hypopnea index. OSA is not associated with worsening
renal function, although long-standing untreated hypertension can bring about renal
dysfunction indirectly. Gastric aspiration, seizures, and anxiety are not known to
occur more commonly in patients with OSA.

Pulmonary Medicine and Critical Care:Question 99


The correct answer is D
Educational Objectives
Recognize initial oxygenation requirements that must exist before a patient is
weaned from mechanical
ventilation.
Critique
Before a patient can be extubated and taken off mechanical ventilator support, the
following criteria must be met: (1) there must be reasonable expectation that
oxygenation will be adequate, (2) the patient must be able to protect the airway, (3)
the patient should be stable and have adequate ventilatory strength. Typically, a
fractional inspired oxygen level (FF0 2) of .4 or less and a positive end-expiratory
pressure (PEEP) of 5 cm 1-120 or less are required.
In this circumstance it is possible that the PEEP of 10 cm H2O is still providing
significant therapeutic benefit for oxygenation and if the patient were extubated,
hypoxemia could occur with the highest FiO2 achievable when the endotracheal tube
is withdrawn. Although a spontaneous breathing trial with a T-piece trial or with a T-
piece-like trial (5 cm H2O PEEP and 5 cm H2O pressure support) would be a logical
next step to judge capability for extubation after the criteria are met. Although
noninvasive ventilation has been used as a bridge in select patient populations, in
this case there are no data to indicate that it is the best step for this patient.
Synchronized intermittent mandatory ventilation may also be used as a weaning
technique, but not until it is certain that the patient will be able to breathe
adequately with the endotracheal tube and mechanical ventilation removed.

Pulmonary Medicine and Critical Care:Question 100


The correct answer is B
Educational Objectives
Recognize contraindications to noninvasive positive-pressure ventilation.
Critique
Among the contraindications to noninvasive ventilation are emesis, failure of prior
attempts at noninvasive ventilation, hemodynamic instability, inability to protect the
airway, excessive secretions, an agitated or uncooperative state, and anatomic
abnormality of the face that interferes with mask fit. In this patient not only is
continued emesis a distinct possibility, but also the drop in oxyhemoglobin saturation
may already represent aspiration. A chest radiograph should be obtained to verify
that possibility.

Pulmonary Medicine and Critical Care:Question 101


The correct answer is C
Educational Objectives
Recognize surgical risk in a patient with emphysema.
Critique
Upper abdominal and thoracic surgeries pose the greatest risk to patients with
obstructive lung disease. Anesthesia- and narcotic-induced respiratory depression
84

adds to the risk.


In general, the farther the surgical site from the diaphragm, the lower the surgical
risk to patients with obstructive lung disease. Ophthalmologic procedures have a
very low mortality rate. Short endoscopic procedures, especially with the use of local
anesthesia pose a reasonably low risk.

Pulmonary Medicine and Critical Care:Question 102


The correct answer is D
Educational Objectives
Recognize the clinical and radiographic abnormalities associated with idiopathic
pulmonary fibrosis superimposed on pulmonary emphysema.
Critique
This patient’s history, the crackles heard on auscultation of the lungs, and the
radiographic abnormalities are best explained by a combination of pulmonary fibrosis
and pulmonary emphysema. The counterbalancing restrictive effect of pulmonary
fibrosis and the obstructive effect of emphysema account for the normalization of
forced expiratory volume in 1 sec (FEV1) and forced vital capacity (FVC). Pulmonary
fibrosis and emphysema decrease the diffusing lung capacity for carbon monoxide
(DLCO) and explain the disproportionately low DLCO in this case.
Several collagen vascular diseases are associated with progressive pulmonary
fibrosis. Sometimes lung involvement precedes other manifestations of these
diseases. Conversely, levels of antinuclear antibody and rheumatoid factor are often
abnormal in idiopathic pulmonary fibrosis. This patient’s age and sex and the paucity
of extrapulmonary signs or symptoms 1 year after onset of illness all point to
idiopathic pulmonary fibrosis rather than to systemic lupus erythematosus or
scleroderma.
No pharmacologic treatment has proven effective for idiopathic pulmonary fibrosis.
Because the median survival of affected patients remains dismal at 2.8 years,
referral for lung transplantation evaluation should be considered as soon as the
diagnosis is confirmed.

Pulmonary Medicine and Critical Care:Question 103


The correct answer is C
Educational Objectives
Recognize the significance of chronic obstructive pulmonary disease as a worldwide
health threat.
critique
Chronic obstructive pulmonary disease (COPD) is a growing international health
threat that currently ranks as the fourth leading cause of morbidity and mortality in
the United States. COPD is also the only major disease with an increasing mortality
rate. The World Bank and the World Health Organization predict that by the year
2020, COPD will rank fifth as a cause of worldwide disability. Coronary disease,
stroke, and gastric cancer, by contrast, are decreasing as causes of morbidity and
mortality.

Pulmonary Medicine and Critical Care:Question 104


The correct answer is A
Educational Objectives
Treat the hemodynamic instability of septic shock.
Critique
The patients primary continuing physiologic abnormality that needs addressing is
the low mean arterial pressure (MAP). A pressure lower than 60 to 65 mm Hg is
85

considered problematic because of sepsis-induced arteriolar vasodilation


produced by nitric oxide. The degree of arteriolar vasodilation can be estimated
by calculation of systemic vascular resistance index (SVRI). SVRI = MAP-RAP
divided by Cl times 80. Normal range for SVRI is 1300-2900 dyne/s/cm-5fm.
Further vasopressor therapy to raise mean arterial pressure is indicated.
Although diffuse capillary leak and increased venous capacitance may lead to
significant decreases in left ventricular preload, a pulmonary artery occlusive
pressure of 18 mm Hg in this case implies adequate left ventricular filling. The
increased pulmonary vascular resistance will offer a further impediment to left
ventricular filling but again in this case left ventricular filling is adequate.
Significant myocardial depression may occur in septic shock and in some patients
lead to a low cardiac index despite adequate left ventricular filling, boost stroke
volume through afterload reduction (induced by sepsis stimulation of no
production), and tachycardia. In this patient, however, cardiac index is slightly
elevated and therefore contractility is not a major issue.

Вам также может понравиться